Vous êtes sur la page 1sur 138

ERDIE E.

AMBROCIO Remedial Law Review 1


2014-0308 Criminal Procedure – Case Digests

1. HANNAH SERRANA VS SANDIGANBAYAN


542 SCRA

FACTS: Serana was a senior student of UP-Cebu who was also appointed by Pres. Estrada as
student regent of UP to serve a one-year term from Jan.1, 2000 to Dec. 31, 2000. On Sept. 2000,
petitioner together with her siblings and relatives, registered with the SEC the Office of the
Student Regent Foundation, Inc. (OSFRI). On of the projects of the OSFRI was the renovation of
Vinzons Hall in UP Diliman, and Pres. Estrada gave P15M as financial assistance for the said
project. The source of funds, according to the information, was the Office of the President.

However, the renovation failed to materialize. The succeeding student regent and system-wide
alliances of students consequently filed a complaint for Malversation of Public Funds and
Property with the Ombudsman. After due investigation, the Ombudsman instituted a criminal
case against Serana and her brother, charging them of Estafa.

Serana moved to quash the Information, contending that the Sandiganbayan does not have
jurisdiction over the offense charged nor over her person in her capacity as UP student regent.
She contends that Estafa falls under Crimes Against Property and not on the chapter on Crimes
Committed by Public Officers, only over which, she argues, the Sandiganbayan has jurisdiction.
Furthermore, she argues that it was not the government that was duped, but Pres. Estrada,
because the money came from the Office of the President and not from government funds. As to
jurisdiction over her person, she contends that as a UP student regent, she is not a public officer
since she merely represents her peers, in contrast to the other regents who held their positions in
an ex officio capacity.

The Sandiganbayan denied her motion for lack of merit.

ISSUE/S: Whether or not Sandiganbayan has jurisdiction over the offense charged and over
Serana

HELD: YES. Jurisdiction of Sandiganbayan; Crime of Estafa. Plainly, estafa is one of those
felonies within the jurisdiction of the Sandiganbayan, subject to the twin requirements that: 1)
the offense is committed by public officials and employees mentioned in Section 4(A) of PD No.
1606, as amended, and that; 2) The offense is committed in relation to their office.

It is well established that compensation is not an essential element of public office. At most, it is
merely incidental to the public office. Delegation of sovereign functions of the government, to be
exercised by him for the benefit of the public makes one a public officer. A UP Student Regent is
a Public Officer. A public office is the right, authority, and duty created and conferred by law, by
which for a given period, either fixed or enduring at the pleasure of the power, an individual is
interested with some portion of sovereign functions of the government, to be exercised by him
for the benefit of the public.

Jurisdiction of the Sandiganbayan covers Board of Regents. The Sandiganbayan, also has
jurisdiction over the other officers enumerated in PD No. 1606. In Geduspan v. People, the SC

1
ERDIE E. AMBROCIO Remedial Law Review 1
2014-0308 Criminal Procedure – Case Digests

held that while the first part of Sec. 4(A) covers only officials with Salary grade 27 and higher
but who are by express provisions of law placed under the jurisdiction of the Sandiganbayan as
she is placed there by express provisions of law. Sec. 4(A)(1)(g) of PD No. 1606 explicitly
vested the Sandiganbayan with jurisdiction over Presidents, directors and trustees, or manager of
government-owned or controlled corporations, state universities, or educational foundations.
Petitioner falls under this category. As the Sandiganbayan pointed out, the Board of Regents
performs functions similar to those of a board of trustee of a non-stock corporation. By express
mandate of law, petitioner is, indeed, a public officer as contemplated by PD No. 1606.

2
ERDIE E. AMBROCIO Remedial Law Review 1
2014-0308 Criminal Procedure – Case Digests

2. GARCIA VS SANDIGANBAYAN
603 SCRA

FACTS: To recover unlawfully acquired funds and properties that the Garcia’s’ had allegedly
amassed and acquired, the Republic, through the OMB filed with the SB 2 petitions for the
forfeiture of those properties. The Garcia’s’ filed motion to dismiss on the ground of SB’s lack
of jurisdiction for lack of proper and valid service of summons: (1) Forfeiture I – the
corresponding summons on the case were all issued and all served on Gen. Garcia at his place of
detention; (2) Forfeiture II – the sheriff stated giving the copies of the summons to the
OIC/Custodian of the PNP Detention Center who in turn handed them to Gen. Garcia.

The general signed his receipt of the summons, but as to those pertaining to the other
respondents, Gen. Garcia acknowledged receiving the same, but with the following qualifying
note: In receiving the copies of Clarita, Ian Carl, Juan Paolo & Timothy but these copies will not
guarantee it being served to the above-named.

ISSUE/S: Whether the Sandiganbayan acquired jurisdiction on the person of Clarita Garcia and
her three sons, considering that summons were improperly served and that the plunder case
(Crim. Case No. 28107) has already been filed and pending with another division of the SB, i.e.,
Second Division of the SB.

HELD: Forfeiture Cases and the Plunder Case Have Separate Causes of Action; the Former Is
Civil in Nature while the Latter Is Criminal.

A forfeiture case under RA 1379 arises out of a cause of action separate and different from a
plunder case, thus negating the notion that the crime of plunder charged in Crim. Case No. 28107
absorbs the forfeiture cases. In a prosecution for plunder, what is sought to be established is the
commission of the criminal acts in furtherance of the acquisition of ill-gotten wealth. In the
language of Sec. 4 of RA 7080, for purposes of establishing the crime of plunder, it is sufficient
to establish beyond reasonable doubt a pattern of overt or criminal acts indicative of the overall
unlawful scheme or conspiracy [to amass, accumulate or acquire ill-gotten wealth]. On the other
hand, all that the court needs to determine, by preponderance of evidence, under RA 1379 is the
disproportion of respondents properties to his legitimate income, it being unnecessary to prove
how he acquired said properties. As correctly formulated by the Solicitor General, the forfeitable
nature of the properties under the provisions of RA 1379 does not proceed from a determination
of a specific overt act committed by the respondent public officer leading to the acquisition of
the illegal wealth.

On the issue of lack of jurisdiction, petitioner argues that the SB did not acquire jurisdiction over
her person and that of her children due to a defective substituted service of summons. There is
merit in petitioner’s contention. Sec. 7, Rule 14 of the 1997 Revised Rules of Civil Procedure
clearly provides for the requirements of a valid substituted service of summons, thus:

SEC. 7. Substituted service. If the defendant cannot be served within a reasonable


time as provided in the preceding section [personal service on defendant], service

3
ERDIE E. AMBROCIO Remedial Law Review 1
2014-0308 Criminal Procedure – Case Digests

may be effected (a) by leaving copies of the summons at the defendants residence
with some person of suitable age and discretion then residing therein, or (b) by
leaving the copies at defendants office or regular place of business with some
competent person in charge thereof.

It is basic that a court must acquire jurisdiction over a party for the latter to be bound by its
decision or orders. Valid service of summons, by whatever mode authorized by and proper under
the Rules, is the means by which a court acquires jurisdiction over a person.

In the instant case, it is undisputed that summons for Forfeitures I and II were served personally
on Maj. Gen. Carlos Flores Garcia, who is detained at the PNP Detention Center, who
acknowledged receipt thereof by affixing his signature. It is also undisputed that substituted
service of summons for both Forfeitures I and II were made on petitioner and her children
through Maj. Gen. Garcia at the PNP Detention Center. However, such substituted services of
summons were invalid for being irregular and defective.

Requirements as laid down in Manotoc vs CA


1. Impossibility of prompt personal service, i.e., the party relying on substituted service or
the sheriff must show that defendant cannot be served promptly or there is impossibility
of prompt service within a reasonable time. Reasonable time being “so much time as is
necessary under the circumstances for a reasonably prudent and diligent man to do,
conveniently, what the contract or duty requires that should be done, having a regard for
the rights and possibility of loss, if any[,] to the other party.” Moreover, we indicated
therein that the sheriff must show several attempts for personal service of at least three
(3) times on at least two (2) different dates.

2. Specific details in the return, i.e., the sheriff must describe in the Return of Summons the
facts and circumstances surrounding the attempted personal service.

3. Substituted service effected on a person of suitable age and discretion residing at


defendant’s house or residence; or on a competent person in charge of defendant’s office
or regular place of business.

4
ERDIE E. AMBROCIO Remedial Law Review 1
2014-0308 Criminal Procedure – Case Digests

3. ESCOBAL VS GARCHITORENA
422 SCRA

FACTS: Petitioner Escobal is a graduate of the PMA, a member of the AFP and the Philippine
Constabulary, as well as the Intelligence Group of the Philippine National Police. On March 16,
1990, the petitioner was conducting surveillance operations on drug trafficking at a café bar and
restaurant in Naga City when he somehow got involved with a shooting incident that resulted to
the death of Rodney Nueca.

Escobal was preventively suspended from the service. When arraigned, he pleaded not guilty.
Thereafter, he filed a Motion to Quash the Information alleging that the court martial, not the
RTC, had jurisdiction over criminal cases involving PNP members and officers. RTC denied the
motion. Trial proceeded. The prosecution rested its case and petitioner presented his evidence.
On July 20, 1994, the petitioner filed a Motion to Dismiss the case. Citing Republic of the
Philippines v. Asuncion, et al., he argued that since he committed the crime in the performance
of his duties, the Sandiganbayan had exclusive jurisdiction over the case. The RTC dismissed the
motion but ordered the conduct of a preliminary hearing to determine whether or not the crime
charged was committed by the petitioner in relation to his office as a member of the PNP.

On July 31, 1995, the trial court issued an Order declaring that the petitioner committed the
crime charged while not in the performance of his official function. The trial court added that
nonetheless, upon the enactment of R.A. No. 7975, the issue had become moot and academic
since the amendatory law transferred the jurisdiction over the offense charged from the
Sandiganbayan to the RTC. The petitioner did not have a salary grade of "27" as provided for in
or by Section 4(a)(1), (3) thereof.

The trial court nevertheless ordered the prosecution to amend the Information pursuant to the
ruling in Republic v. Asuncion and R.A. No. 7975, and to include therein an allegation that the
offense charged was not committed by the petitioner in the performance of his duties/functions,
nor in relation to his office.

The petitioner filed a MR of the said order, reiterating that based on his testimony and those of
his witnesses, the offense charged was committed by him in relation to his official functions. He
asserted that R.A. No. 7975, which was enacted on March 30, 1995, could not be applied
retroactively.

The RTC ordered the public prosecutor to file a Re-Amended Information and to allege that the
offense charged was committed by the petitioner in the performance of his duties/functions or in
relation to his office; and, conformably to R.A. No. 7975, to thereafter transmit the same to the
Sandiganbayan. The Sandiganbayan returned the records of the case to the RTC, contending that
the latter has jurisdiction over the case.

ISSUE/S: Whether the case falls in the jurisdiction of the Sandiganbayan or of the RTC

HELD: The case is within the jurisdiction of the RTC. Under Section 4(a) of P.D. No. 1606 as

5
ERDIE E. AMBROCIO Remedial Law Review 1
2014-0308 Criminal Procedure – Case Digests

amended by P.D. No. 1861, the Sandiganbayan had exclusive jurisdiction in all cases involving
the following:
(1) Violations of Republic Act No. 3019, as amended, otherwise known as the Anti-Graft
and Corrupt Practices Act, Republic Act No. 1379, and Chapter II, Section 2, Title VII of
the Revised Penal Code;
(2) Other offenses or felonies committed by public officers and employees in relation to
their office, including those employed in government-owned or controlled corporations,
whether simple or complexed with other crimes, where the penalty prescribed by law is
higher than prision correccional or imprisonment for six (6) years, or a fine of P6,000.

For the Sandiganbayan to have exclusive jurisdiction under the said law over crimes committed
by public officers in relation to their office, it is essential that the facts showing the intimate
relation between the office of the offender and the discharge of official duties must be alleged in
the Information. It is not enough to merely allege in the Information that the crime charged was
committed by the offender in relation to his office because that would be a conclusion of law.
The amended Information filed with the RTC against the petitioner does not contain any
allegation showing the intimate relation between his office and the discharge of his duties.
Hence, the RTC had jurisdiction over the offense charged when on November 24,1995, it
ordered the re-amendment of the Information to include therein an allegation that the petitioner
committed the crime in relation to office. The trial court erred when it ordered the elevation of
the records to the Sandiganbayan. It bears stressing that R.A. No. 7975 amending P.D. No. 1606
was already in effect.

Under Sec. 2 of said law, even if the offender committed the crime charged in relation to his
office but occupies a position corresponding to a salary grade below "27," the proper Regional
Trial Court or Municipal Trial Court, as the case may be, shall have exclusive jurisdiction over
the case. In this case, the petitioner was a Police Senior Inspector, with salary grade "23." He was
charged with homicide punishable by reclusion temporal. Hence, the RTC had exclusive
jurisdiction over the crime charged conformably to Sections 20 and 32 of Batas Pambansa Blg.
129, as amended by Section 2 of R.A. No. 7691.

The petitioner’s contention that R.A. No. 7975 should not be applied retroactively has no legal
basis. It bears stressing that R.A. No. 7975 is a substantive procedural law, which may be applied
retroactively.

6
ERDIE E. AMBROCIO Remedial Law Review 1
2014-0308 Criminal Procedure – Case Digests

4. PEOPLE VS HENRY GO
March 25, 2014

FACTS: An information was filed against Secretary Arturo Enrile and Henry T. Go before the
Sandiganbayan. The said information also alleged that there was a conspiracy between Enrile
and Go. Go filed a motion to quash alleging that the operative facts adduced therein do not
constitute an offense under Section 3(g) of R.A. 3019. Respondent, citing the show cause order
of the SB, also contended that, independently of the deceased Secretary Enrile, the public officer
with whom he was alleged to have conspired, respondent, who is not a public officer nor was
capacitated by any official authority as a government agent, may not be prosecuted for violation
of Section 3(g) of R.A. 3019.

ISSUE/S: Whether the SB lost its jurisdiction over the person of the accused since he was not a
public officer.

HELD: It is true that by reason of Secretary Enrile's death, there is no longer any public officer
with whom respondent can be charged for violation of R.A. 3019. It does not mean, however,
that the allegation of conspiracy between them can no longer be proved or that their alleged
conspiracy is already expunged. The only thing extinguished by the death of Secretary Enrile is
his criminal liability. His death did not extinguish the crime nor did it remove the basis of the
charge of conspiracy between him and private respondent. Stated differently, the death of
Secretary Enrile does not mean that there was no public officer who allegedly violated Section 3
(g) of R.A. 3019. In fact, the Office of the Deputy Ombudsman for Luzon found probable cause
to indict Secretary Enrile for infringement of Sections 3 (e) and (g) of R.A. 3019.14 Were it not
for his death, he should have been charged.

The requirement before a private person may be indicted for violation of Section 3(g) of R.A.
3019, among others, is that such private person must be alleged to have acted in conspiracy with
a public officer. The law, however, does not require that such person must, in all instances, be
indicted together with the public officer. If circumstances exist where the public officer may no
longer be charged in court, as in the present case where the public officer has already died, the
private person may be indicted alone.

7
ERDIE E. AMBROCIO Remedial Law Review 1
2014-0308 Criminal Procedure – Case Digests

5. RAMISCAL VS SANDIGANBAYAN
630 SCRA

FACTS: Ramiscal Jr. (Ramiscal) was a retired officer of AFP and the former president of AFP-
Retirement and Separation Benefits System (AFP-RSBS). During his incumbency, the BOD of
AFP-RSBS approved the acquisition of 15,020 sq. m. of land for development as housing
projects. On August 1, 1997 AFP-RSBS as represented by Ramiscal Jr., and Flaviano the
attorney-in-fact of 12 individual vendors executed and signed a bilateral Deed of Sale (1st Deed)
over the subject property at the agreed price of P 10,500.00 per sq. m. After the payment @ P
10,500.00 per sq. m., Flaviano executed and signed a unilateral Deed of Sale (2nd Deed) over the
same property with a purchase price of P 3,000.00 per sq. m. Thereafter the 2 nd Deed was
presented by Flaviano for registration, which became the basis of the Certificate of Title of the
said property.

Ramiscal Jr. filed his first Motion for Reconsideration date February 12, 1999 with a
supplemental motion dated May 28, 1999 regarding the findings of the Ombudsman. With this, a
panel of prosecutors was tasked to review the records of the case, they found out that Ramiscal
Jr., indeed participated in an affixed his signature on the contracts and found probable cause. The
Ombudsman acted positively on the findings of the prosecutor and scheduled the arraignment of
Ramiscal Jr. However, Ramiscal Jr., refused to enter a plea for petitioner on the ground that there
is a pending resolution of his second Motion for Reconsideration.

ISSUE/S:
1. Whether or not the second Motion for Reconsideration is valid and should hold his
arraignment.
2. Whether or not there is probable cause to file a case for violation of Section 3 (e) of the
Anti-Graft and Corrupt Practices Act and falsification of public documents.

HELD: No, Sec 7 of Rule 11 of the Rules provides that only one motion for reconsideration or
reinvestigation of an approved order or resolution shall be allowed xxxxx the filing of a motion
for reconsideration/reinvestigation shall not bar the
filling of the corresponding information in Court on the basis of the finding of probable cause in
the resolution subject of the motion.

The arraignment may be suspended under Sec. 11 of Rule 116 of the Rules of Court are:
unsoundness of mind, prejudicial question and a pending petition for review of the resolution of
the prosecutor in the DOJ in which the suspension shall not exceed 60 days. Ramiscal Jr., failed
to show that any of the instances constituting a valid ground for suspension of arraignment
obtained in this case. With respect to the finding of probable cause, it is the Ombudsman who
has the full discretion to determine whether or not a criminal case should be filed in the
Sandiganbayan, once the case has been filed with the said court, it is the Sandiganbayan, and no
longer the Ombudsman which has full control of the case. Ramiscal Jr., failed to establish that
Sandiganbayan committed grave abuse of discretion, thus, there is probable cause in the filing of
the case.

8
ERDIE E. AMBROCIO Remedial Law Review 1
2014-0308 Criminal Procedure – Case Digests

6. PEOPLE VS BENIPAYO
586 SCRA

FACTS: Alfredo, then Chairman of the Commission of elections, was charged with libel before
the Office of the City Prosecutor by Photokina Marketing Corporation, which felt alluded to in a
speech made by Alfredo before the Bahay Kalinaw, University of the Philippines, and in an
television interview before “Point Blank”, a show hosted by Ces Orena-Drilon at ANC. The
Office of the City Prosecutor, in both instances, filed Informations for libel before the Regional
Trial Court. In both instances, Alfredo moved for the dismissal of the case, considering that at
the time he made the alleged utterances, he was an impeachable officer and the same was made
in relation to his duties, therefore, even assuming that he
can be charged with libel, the same should be lodged with the Sandiganbayan. In both instances,
the RTC ruled in his favor, thus Photokina elevated the case to the Supreme Court on question of
law on whether the utterances were made in relation to his office; and that the RTC had no
jurisdiction over the case.

ISSUE/S: Whether or not (1) utterances of Benipayo were made in relation to his office, and (2)
RTC had jurisdiction over the libel case.

HELD: The Court observes that the parties have argued at length in their pleadings on the issue
of whether the alleged criminal acts of respondent are committed in relation to his office. They
are of the conviction that the resolution of the said question will ultimately determine which
court–the RTC or the Sandiganbayan–has jurisdiction over the criminal cases filed. The Court,
however, notes that both parties are working on a wrong premise. The foremost concern, which
the parties, and even the trial court, failed to identify, is whether, under our current laws,
jurisdiction over libel cases, or written defamations to be more specific, is shared by the RTC
with the Sandiganbayan. Indeed, if the said courts do not have concurrent jurisdiction to try the
offense, it would be pointless to still determine whether the crime is committed in relation to
office. Uniformly applied is the familiar rule that the jurisdiction of the court to hear and decide a
case is conferred by the law in force at the time of the institution of the action, unless a latter
statute provides for a retroactive application thereof. Article 360 of the Revised Penal Code
(RPC), as amended by Republic Act No. 4363, is explicit on which court has jurisdiction to try
cases of written defamations, thus: The criminal and civil action for damages in cases of written
defamations as provided for in this chapter, shall be filed simultaneously or separately with the
court of first instance [now, the Regional Trial Court] of the province or city where the libelous
article is printed and first published or where any of the offended parties actually resides at the
time of the commission of the offense xxx.

More than three decades ago, the Court, in Jalandoni v. Endaya, acknowledged the unmistakable
import of the said provision: There is no need to make mention again that it is a court of first
instance [now, the Regional Trial Court] that is specifically designated to try a libel case. Its
language is categorical; its meaning is free from doubt. This is one of those statutory provisions
that leave no room for interpretation. All that is required is application. What the law ordains
must then be followed.

9
ERDIE E. AMBROCIO Remedial Law Review 1
2014-0308 Criminal Procedure – Case Digests

This exclusive and original jurisdiction of the RTC over written defamations is echoed in
Bocobo v. Estanislao, where the Court further declared that jurisdiction remains with the trial
court even if the libelous act is committed “by similar means,” and despite the fact that the
phrase “by similar means” is not repeated in the latter portion of Article 360. In these cases, and
in those that followed, the Court had been unwavering in its pronouncement that the expanded
jurisdiction of the municipal trial courts cannot be exercised over libel cases. Thus, in Manzano
v. Hon. Valera, we explained at length that: The applicable law is still Article 360 of the Revised
Penal Code, which categorically provides that jurisdiction over libel cases [is] lodged with the
Courts of First Instance (now Regional Trial Courts).

This Court already had the opportunity to rule on the matter in G.R. No. 123263, People vs.
MTC of Quezon City, Branch 32 and Isah v. Red wherein a similar question of jurisdiction over
libel was raised. In that case, the MTC judge opined that it was the first level courts which had
jurisdiction due to the enactment of RA 7691. Upon elevation of the matter to us, respondent
judge’s orders were nullified for lack of jurisdiction, as follows: “WHEREFORE, the petition is
granted: the respondent Court’s Orders dated August 14, 1995, September 7, 1995, and October
18, 1995 are declared null and void for having been issued without jurisdiction; and said Court is
enjoined from further taking cognizance of and proceeding with Criminal Case No. 43-00548,
which it is commanded to remand to the Executive Judge of the Regional Trial Court of Quezon
City for proper disposition.”

Another case involving the same question was cited as resolving the matter: “Anent the question
of jurisdiction, we ** find no reversible error committed by public respondent Court of Appeals
in denying petitioner’s motion to dismiss for lack of jurisdiction. The contention ** that R.A.
7691 divested the Regional Trial Courts of jurisdiction to try libel cases cannot be sustained.
While libel is punishable by imprisonment of six months and one day to four years and two
months (Art. 360, Revised Penal Code) which imposable penalty is lodged within the Municipal
Trial Court’s jurisdiction under R.A. No. 7691 (Sec. 32), said law however, excludes therefrom
** cases falling within the exclusive original jurisdiction of the Regional Trial Courts **. The
Court in Bocobo vs. Estanislao, 72 SCRA 520 and Jalandoni vs. Endaya, 55 SCRA 261,
correctly cited by the Court of Appeals, has laid down the rule that Regional Trial courts have
the exclusive jurisdiction over libel cases, hence, the expanded jurisdiction conferred by R.A.
7691 to inferior courts cannot be applied to libel cases.”

Conformably with [these] rulings, we now hold that public respondent committed an error in
ordering that the criminal case for libel be tried by the MTC of Bangued. For, although RA 7691
was enacted to decongest the clogged dockets of the Regional Trail Courts by expanding the
jurisdiction of first level courts, said law is of a general character. Even if it is a later enactment,
it does not alter the provision of Article 360 of the RPC, a law of a special nature. “Laws vesting
jurisdiction exclusively with a particular court, are special in character, and should prevail over
the Judiciary Act defining the jurisdiction of other courts (such as the Court of First Instance)
which is a general law.” A later enactment like RA 7691 does not automatically override an
existing law, because it is a well-settled principle of construction that, in case of conflict between
a general law and a special law, the latter must prevail regardless of the dates of their enactment.
Jurisdiction conferred by a special law on the RTC must therefore prevail over that granted by a

10
ERDIE E. AMBROCIO Remedial Law Review 1
2014-0308 Criminal Procedure – Case Digests

general law on the MTC.

Moreover, from the provisions of R.A. 7691, there seems to be no manifest intent to repeal or
alter the jurisdiction in libel cases. If there was such intent, then the amending law should have
clearly so indicated because implied repeals are not favored. As much as possible, effect must be
given to all enactments of the legislature. A special law cannot be repealed, amended or altered
by a subsequent general law by mere implication. Furthermore, for an implied repeal, a pre-
condition must be found, that is, a substantial conflict should exist between the new and prior
laws. Absent an express repeal, a subsequent law cannot be construed as repealing a prior one
unless an irreconcilable inconsistency or repugnancy exists in the terms of the new and old laws.
The two laws, in brief, must be absolutely incompatible. In the law which broadened the
jurisdiction of the first level courts, there is no absolute prohibition barring Regional Trial Courts
from taking cognizance of certain cases over which they have been priorly granted special and
exclusive jurisdiction. Such grant of the RTC (previously CFI) was categorically contained in the
first sentence of the amended Sec. 32 of B.P. 129. The inconsistency referred to in Section 6 of
RA 7691, therefore, does not apply to cases of criminal libel.

Lastly, in Administrative Order No. 104-96 issued 21 October 1996, this Court delineated the
proper jurisdiction over libel cases, hence settled the matter with finality:
“RE: DESIGNATION OF SPECIAL COURTS FOR KIDNAPPING, ROBBERY,
CARNAPPING, DANGEROUS DRUGS CASES AND OTHER HEINOUS CRIMES;
INTELLECTUAL PROPERTY RIGHTS VIOLATIONS AND JURISDICTION IN LIBEL
CASES.
xxxx
“LIBEL CASES SHALL BE TRIED BY THE REGIONAL TRIAL COURTS HAVING
JURISDICTION OVER THEM TO THE EXCLUSION OF THE METROPOLITAN TRIAL
COURTS, MUNICIPAL TRIAL COURTS IN CITIES, MUNICIPAL TRIAL COURTS AND
MUNICIPAL CIRCUIT TRIAL COURTS.” (Underscoring supplied)

As we have constantly held in Jalandoni, Bocobo, People v. Metropolitan Trial Court of Quezon
City, Br. 32,[41] Manzano, and analogous cases, we must, in the same way, declare herein that
the law, as it still stands at present, dictates that criminal and civil actions for damages in cases of
written defamations shall be filed simultaneously or separately with the RTC to the exclusion of
all other courts. A subsequent enactment of a law defining the jurisdiction of other courts cannot
simply override, in the absence of an express repeal or modification, the specific provision in the
RPC vesting in the RTC, as aforesaid, jurisdiction over defamations in writing or by similar
means. The grant to the Sandiganbayan of jurisdiction over offenses committed in relation to
(public) office, similar to the expansion of the jurisdiction of the MTCs, did not divest the RTC
of its exclusive and original jurisdiction to try written defamation cases regardless of whether the
offense is committed in relation to office. The broad and general phraseology of Section 4,
Presidential Decree No. 1606, as amended by Republic Act No. 8249, cannot be construed to
have impliedly repealed, or even simply modified, such exclusive and original jurisdiction of the
RTC.

Since jurisdiction over written defamations exclusively rests in the RTC without qualification, it

11
ERDIE E. AMBROCIO Remedial Law Review 1
2014-0308 Criminal Procedure – Case Digests

is unnecessary and futile for the parties to argue on whether the crime is committed in relation to
office. Thus, the conclusion reached by the trial court that the respondent committed the alleged
libelous acts in relation to his office as former COMELEC chair, and deprives it of jurisdiction to
try the case, is, following the above disquisition, gross error. This Court, therefore, orders the
reinstatement of Criminal Cases Nos. Q-02-109406 and Q-02-109407 and their remand to the
respective Regional Trial Courts for further proceedings. Having said that, the Court finds
unnecessary any further discussion of the other issues raised in the petitions.”

12
ERDIE E. AMBROCIO Remedial Law Review 1
2014-0308 Criminal Procedure – Case Digests

7. LACSON VS EXECUTIVE SECRETARY


301 SCRA

FACTS: Eleven persons believed to be members of the Kuratong Baleleng gang, an organized
crime syndicate involved in bank robberies, were slain by elements of the Anti-Bank Robbery
and Intelligence Task Group (ABRITG). Among those included in the ABRITG were petitioners
and petitioner-intervenors.

Acting on a media expose of SPO2 Eduardo delos Reyes, a member of the Criminal
Investigation Command, that what actually transpired was a summary execution and not a shoot-
out between the Kuratong Baleleng gang members and the ABRITG, Ombudsman Aniano
Desierto formed a panel of investigators to investigate the said incident. Said panel found the
incident as a legitimate police operation. However, a review board modified the panel’s finding
and recommended the indictment for multiple murder against twenty-six respondents including
herein petitioner, charged as principal, and herein petitioner-intervenors, charged as accessories.
After a reinvestigation, the Ombudsman filed amended informations before the Sandiganbayan,
where petitioner was charged only as an accessory.

The accused filed separate motions questioning the jurisdiction of the Sandiganbayan, asserting
that under the amended informations, the cases fall within the jurisdiction of the Regional Trial
Court pursuant to Section 2 of R.A. 7975. They contend that the said law limited the jurisdiction
of the Sandiganbayan to cases where one or ore of the “principal accused” are government
officals with Salary Grade 27 or higher, or PNP officials with rank of Chief Superintendent or
higher. Thus, they did not qualify under said requisites. However, pending resolution of their
motions, R.A. 8249 was approved amending the jurisdiction of the Sandiganbayan by deleting
the word “principal” from the phrase “principal accused” in Section 2 of R.A. 7975.

Petitioner questions the constitutionality of Section 4 of R.A. 8249, including Section 7 which
provides that the said law shall apply to all cases pending in any court over which trial has not
begun as of the approval hereof.

ISSUE/S: Whether or not the multiple murder of the alleged members of the Kuratong Baleleng
was committed in relation to the office of the accused PNP officers which is essential to the
determination whether the case falls within the Sandiganbayan’s or Regional Trial Court’s
jurisdiction.

HELD: RTC has jurisdiction. In People vs. Montejo, it was held that an offense is said to have
been committed in relation to the office if it is intimately connected with the office of the
offender and perpetrated while he was in the performance of his official functions. Such intimate
relation must be alleged in the information which is essential in determining the jurisdiction of
the Sandiganbayan. However, upon examination of the amended information, there was no
specific allegation of facts that the shooting of the victim by the said principal accused was
intimately related to the discharge of their official duties as police officers.

Likewise, the amended information does not indicate that the said accused arrested and

13
ERDIE E. AMBROCIO Remedial Law Review 1
2014-0308 Criminal Procedure – Case Digests

investigated the victim and then killed the latter while in their custody. The stringent requirement
that the charge set forth with such particularity as will reasonably indicate the exact offense
which the accused is alleged to have committed in relation to his office was not established.

Consequently, for failure to show in the amended informations that the charge of murder was
intimately connected with the discharge of official functions of the accused PNP officers, the
offense charged in the subject criminal cases is plain murder and, therefore, within the exclusive
original jurisdiction of the Regional Trial Court and not the Sandiganbayan.

Wherefore, the constitutionality of Sections 4 and 7 of R.A. 8249 is hereby sustained. The
Addendum to the March 5, 1997 Resolution of the Sandiganbayan is reversed. The
Sandiganbayan is hereby directed to transfer Criminal Cases Nos. 23047 to 23057 (for multiple
murder) to the Regional Trial Court of Quezon City which has exclusive original jurisdiction
over the said cases.

14
ERDIE E. AMBROCIO Remedial Law Review 1
2014-0308 Criminal Procedure – Case Digests

8. SANCHEZ VS DEMETRIO
227 SCRA

FACTS: Accused-appellants were found guilty beyond reasonable doubt of seven (7) counts of
rape with homicide on seven counts and sentenced each one of them to suffer the penalty of
seven reclusion perpetua. The prosecutions version of the events was based mainly on the
recollections of its star witnesses Aurelio Centeno and Vicencio Malabanan – coconspirators
turned state witnesses. Both admitted having taken part in the abduction of Eileen Sarmenta and
Allan Gomez, but denied any personal involvement in the rape of Eileen and the twin killings
that followed. In this appeal, the pith of the assigned errors and the focus of the appellants
arguments is the issue of witnesses Centeno and Malabanans credibility, whose opencourt
narrations served as principal basis for the trial courts rendition of a guilty verdict.

ISSUE/S: Whether or not the court acted properly on denying the petition of Sanchez to quash
on the grounds that he is being charged with seven homicides arising from the death of only two
persons.

HELD: The court ruled that where there are two or more offenders who commit rape, the
homicide committed on the occasion or by reason of each rape, must be deemed as a constituent
of the special complex crime of rape with homicide. Therefore, there will be as many crimes of
rape with homicide as there are rapes committed. In effect, the presence of homicide qualifies the
crime of rape, thereby raising its penalty to the highest degree. Thus, homicide committed on the
occasion or by reason of rape, loses its character as an independent offense, but assumes a new
character, and functions like a qualifying circumstance. However, by fiction of law, it merged
with rape to constitute an constituent element of a special complex crime of rape with homicide
with a specific penalty which is in the highest degree.

The petitioner and his six co-accused are not charged with only one rape committed by him in
conspiracy with the other six. Each one of the seven accused is charged with having himself
raped Sarmenta instead of simply helping Sanchez in committing only one rape. In other words,
the allegation of the prosecution is that the girl was raped seven times, with each of the seven
accused taking turns in abusing her with the assistance of the other six. Afterwards, their lust
satisfied, all seven of them decided to kill and thus silence Sarmenta. Every one of the seven
accused is being charged separately for actually raping Sarmenta and later killing her instead of
merely assisting the petitioner in raping and then slaying her. The separate informations filed
against each of them allege that each of the seven successive rapes is complexed by the
subsequent slaying of Sarmenta and aggravated by the killing of Allan Gomez by her seven
attackers. The separate rapes were committed in succession by the seven accused, culminating in
the slaying of Sarmenta.

The matter of assigning values to declarations on the witness stand is best and most competently
performed by the trial judge who had the unmatched opportunity to observe the witnesses and to
assess their credibility by the various indicia available but not reflected in the record. The trial
courts impressions of the star witnesses for the State bind this Court, for we accord great respect
if not finality, to the findings of the trial court on the credibility of witnesses. Further, all the

15
ERDIE E. AMBROCIO Remedial Law Review 1
2014-0308 Criminal Procedure – Case Digests

appellants relied on the defense of denial/alibi but positive identification by credible witnesses of
the accused as the perpetrators of the crime, demolishes the alibi.

Calauan Mayor Antonio Sanchez brought a Petition for Certiorari before this Court, challenging
the order of the respondent Judge Demetriou denying his motion to quash the Information filed
against him and six other persons for alleged rape and homicide. One of the arguments of Mayor
Sanchez was that there was discrimination against him because of the non-inclusion of two other
persons in the Information. We held that even this Court cannot order the prosecution of a
person against whom the prosecutor does not find sufficient evidence to support at least a prima
facie ] case. However, if there was an unmistakable showing of grave abuse of discretion on the
part of the prosecutors, Mayor Sanchez should have filed a Petition for Mandamus to compel the
filing of charges against said two other persons.

16
ERDIE E. AMBROCIO Remedial Law Review 1
2014-0308 Criminal Procedure – Case Digests

9. DISINI, JR. VS SECRETARY OF JUSTICE


716 SCRA

FACTS: Petitioners assail the validity of several provision of the Republic Act (R.A.) 10175, the
Cybercrime Prevention Act of 2012. Petitioners claim that the means adopted by the cybercrime
law for regulating undesirable cyberspace activities violate certain constitutional rights. The
government of course asserts that the law merely seeks to reasonably put order into cyberspace
activities, punish wrongdoings, and prevent hurtful attacks on the system.

The following provisions were questioned:


1. Section 4(a)(1) on Illegal Access;
2. Section 4(a)(3) on Data Interference;
3. Section 4(a)(6) on Cyber-squatting;
4. Section 4(b)(3) on Identity Theft;
5. Section 4(c)(1) on Cybersex;
6. Section 4(c)(2) on Child Pornography;
7. Section 4(c)(3) on Unsolicited Commercial Communications;
8. Section 4(c)(4) on Libel;
9. Section 5 on Aiding or Abetting and Attempt in the Commission of Cybercrimes;
10. Section 6 on the Penalty of One Degree Higher;
11. Section 7 on the Prosecution under both the Revised Penal Code (RPC) and R.A.
10175;
12. Section 8 on Penalties;
13. Section 12 on Real-Time Collection of Traffic Data;
14. Section 13 on Preservation of Computer Data;
15. Section 14 on Disclosure of Computer Data;
16. Section 15 on Search, Seizure and Examination of Computer Data;
17. Section 17 on Destruction of Computer Data;
18. Section 19 on Restricting or Blocking Access to Computer Data;
19. Section 20 on Obstruction of Justice;
20. Section 24 on Cybercrime Investigation and Coordinating Center (CICC); and
21. Section 26(a) on CICCs Powers and Functions

ISSUE/S: Which court has jurisdiction over cases covered by RA 10175?

HELD: Sec. 21 provides that the RTC shall have jurisdiction over any violation of the
provisions of RA 10175 including any violation committed by a Filipino national regardless of
the place of commission. ] Jurisdiction shall lie if any of the elements was committed within the
Philippines or committed with the use of any computer system wholly or partly situated in the
country, or when by such commission any damage is caused to a natural or juridical person who,
at the time the offense was committed, was in the Philippine There shall be designated special
cybercrime court manned by specially trained judges to handle cybercrime cases.

17
ERDIE E. AMBROCIO Remedial Law Review 1
2014-0308 Criminal Procedure – Case Digests

10. JIMENEZ VS SORONGON


687 SCRA 151

FACTS: In 2003, Jimenez, the president of Unlad Shipping & Management Corporation, filed a
complaint-affidavit with the Office of the City Prosecutor of Mandaluyong City against
Antzoulatos, Alamil, Gaza and Avgoustis listed incorporators of Tsakos Maritime Services, Inc.
(TMSI), for syndicated and large scale illegal recruitment, alleging that Antzoulatos, et al. falsely
represented their stockholdings in TMSI’s articles of incorporation to secure a license to operate
as a recruitment agency from the Philippine Overseas Employment Agency (POEA).

An Information was then filed before the Regional Trial Court (RTC) Mandaluyong.
Subsequently, the City Prosecutor reconsidered resolution and filed a motion with the RTC to
withdraw the information. The RTC denied the motion to withdraw information as it found the
existence of probable cause to hold Antzoulatos, et al. for trial. Thus, the RTC ordered the
issuance of warrants of arrest against Antzoulatos, et al. Antzoulatos and Gaza filed an omnibus
motion for reconsideration and for deferred enforcement of the warrants of arrest, which was
denied. Alamil filed a motion for judicial determination of probable cause with a request to defer
enforcement of the warrants of arrest., which was denied for being moot and academic. Alamil
moved for reconsideration and for the inhibition of Judge Capco-Umali, for being biased or
partial. Judge Capco-Umali voluntarily inhibited herself from the case and did not resolve
Alamil’s motion for reconsideration and the Jimenez's motion to expunge.

The case was later re-raffled to Branch 214, presided by Judge Edwin D. Sorongon.The RTC
granted Alamil’s motion for reconsideration, and treated the motion for judicial determination as
a motion to dismiss for lack of probable cause. Jimenez moved for reconsideration, stressing the
existence of probable cause to prosecute the respondents and that respondent Alamil had no
standing to seek any relief from the RTC, which was denied. The RTC ordered the motion
expunged from the records since the motion did not have the public prosecutor’s conformity.
Jimenez filed a notice of appeal. Alamil moved to expunge the Jimenez' notice of appeal since
the public prosecutor did not authorize the appeal and the petitioner had no civil interest in the
case.

The RTC denied the Jimenez's notice of appeal since Jimenez filed it without the conformity of
the Solicitor General, who is mandated to represent the People of the Philippines in criminal
actions appealed to the CA. Thus, the RTC ordered the notice of appeal expunged from the
records. Jimenez elevated his case to the CA via a Rule 65 petition for certiorari, which was
dismissed outright by the CA for Jimenez’s lack of legal personality to file the petition on behalf
of the People of the Philippines. The CA denied the motion for reconsideration that followed.

ISSUE/S: Whether or not Jimenez, the private complainant, has legal personality in assailing
the RTC Orders.

HELD: No. It is well-settled that "every action must be prosecuted or defended in the name of
the real party in interest[,]" "who stands to be benefited or injured by the judgment in the suit, or
by the party entitled to the avails of the suit." Interest means material interest or an interest in

18
ERDIE E. AMBROCIO Remedial Law Review 1
2014-0308 Criminal Procedure – Case Digests

issue to be affected by the decree or judgment of the case, as distinguished from mere interest in
the question involved. By real interest is meant a present substantial interest, as distinguished
from a mere expectancy, or a future, contingent, subordinate or consequential interest. When the
plaintiff or the defendant is not a real party in interest, the suit is dismissible.

Procedural law basically mandates that "[a]ll criminal actions commenced by complaint or by
information shall be prosecuted under the direction and control of a public prosecutor." In
appeals of criminal cases before the CA and before this Court, the OSG is the appellate counsel
of the People, pursuant to Section 35(1), Chapter 12, Title III, Book IV of the 1987
Administrative Code.

The People is the real party in interest in a criminal case and only the OSG can represent the
People in criminal proceedings pending in the CA or in this Court. This ruling has been
repeatedly stressed in several cases and continues to be the controlling doctrine.

While there may be rare occasions when the offended party may be allowed to pursue the
criminal action on his own behalf (as when there is a denial of due process), this exceptional
circumstance does not apply in the present case.

19
ERDIE E. AMBROCIO Remedial Law Review 1
2014-0308 Criminal Procedure – Case Digests

11. PEOPLE VS VALDEZ


663 SCRA 272

FACTS: On March 1, 2000, at around 8:00 in the evening, Estrella Sayson was at a canteen
(which also includes a jai alai betting station) located at 77 Corregidor St, Bago Bantay, Quezon
City. Estrella was preparing for the 2nd celebration of her 2nd husband, Wilfredo Lladones,
which was held later that evening. Estrella’s son, the deceased Moises Sayson, a former
policeman, and his wife, Susan Sayson, owned the said canteen and betting station. At about
9:00, Estrella’s other sons, Joselito and Ferdinand Sayson arrived at the canteen to greet their
stepfather.

At about 10:00 of the same evening, the celebration was interrupted by the arrival of Eduardo
and Edwin who alighted from a motorcycle in front of the jai alai betting station. Eduardo and
Edwin, agitated, asked the jai alai teller, Jonathan Rubio, to come out. Jonathan was then
attending to other customers who were buying jai alai tickets. Moises approached Eduardo and
Edwin and tried to reason with them. He advised them not to force Jonathan to go out. Estrella
then heard one of the accused appellants threaten Moises with the words “Gusto mo unahin
kita?” Moises replied “Huwag”. Successive gunshots were thereafter heard. Moises fell and was
continuously fired upon. Ferdinand immediately approached the scene to help his brother. He
was however, was also shot on the left temporal portion of his head and fell. Joselito, ran away
but was also hit at the back while running. The RTC convicted the 2 accused of 3 counts of
murder and sentenced them to suffer Resclusion Perpetua for each count of murder. On appeal,
the CA affirmed the convictions.

In this appeal, PO2 Valdez contended that the state failed to establish the qualifying
circumstance of treachery

ISSUE/S: Whether the prosecution sufficiently established the qualifying circumstance of


treachery.

HELD: For complaint or information to be sufficient, it must state the name of the accused; the
designation of the offense given by the statute; the acts or omissions complained of as
constituting the offense; the name of the offended party; the approximate time of the commission
of the offense, and the place wherein the offense was committed. What is controlling is not the
title of the complaint, nor the designation of the offense charged or the particular law or part
thereof allegedly violated, these being mere conclusions of law made by the prosecutor, but the
description of the crime charged and the particular facts therein recited. The acts or omissions
complained of must be alleged in such form as is sufficient to enable a person of common
understanding to know what offense is intended to be charged, and enable the court to pronounce
proper judgment. No information for a crime will be sufficient if it does not accurately and
clearly allege the elements of the crime charged. Every element of the offense must be stated in
the information. What facts and circumstances are necessary to be included therein must be
determined by reference to the definitions and essentials of the specified crimes. The requirement
of alleging the elements of a crime in the information is to inform the accused of the nature of
the accusation against him so as to enable him to suitably prepare his defense. The presumption

20
ERDIE E. AMBROCIO Remedial Law Review 1
2014-0308 Criminal Procedure – Case Digests

is that the accused has no independent knowledge of the facts that constitute the offense.

The averments of the informations to the effect that the two accused "with intent to kill, qualified
with treachery, evident premeditation and abuse of superior strength did x x x assault, attack and
employ personal violence upon" the victims "by then and there shooting them with a gun, hitting
[them]" on various parts of their bodies "which were the direct and immediate cause of their
deaths" did not sufficiently set forth the facts and circumstances describing how treachery
attended each of the killings. It should not be difficult to see that merely averring the killing of a
person by shooting him with a gun, without more, did not show how the execution of the crime
was directly and specially ensured without risk to the accused from the defense that the victim
might make. Indeed, the use of the gun as an instrument to kill was not per se treachery, for there
are other instruments that could serve the same lethal purpose. Nor did the use of the term
treachery constitute a sufficient averment, for that term, standing alone, was nothing but a
conclusion of law, not an averment of a fact. In short, the particular acts and circumstances
constituting treachery as an attendant circumstance in murder were missing from the
informations.

x x x. The requirement of sufficient factual averments is meant to inform the accused of the
nature and cause of the charge against him in order to enable him to prepare his defense. This
requirement accords with the presumption of innocence in his favor, pursuant to which he is
always presumed to have no independent knowledge of the details of the crime he is being
charged with. To have the facts stated in the body of the information determine the crime of
which he stands charged and for which he must be tried thoroughly accords with common sense
and with the requirements of plain justice, x x x.

21
ERDIE E. AMBROCIO Remedial Law Review 1
2014-0308 Criminal Procedure – Case Digests

12. MIGUEL VS SANDIGANBAYAN


657 SCRA 560

FACTS: Vice Mayor and other local officials of Koronadal City, South Cotabato filed a letter-
complaint with the Office of the Ombudsman-Mindanao charging the petitioner, Fernando
Miguel, of violations of R.A. 3019 (Antigraft and Corrupt Practices Act) in connection with the
consultancy services for the proposed Koronadal Public Market.

The Ombudsman directed the petitioner to submit his counter-affidavit. After moving for an
extension, the petitioner filed his counter-affidavit. Thereafter, the Ombudsman found probable
cause against the petitioner and some other private individuals for violation of RA 3019 and
against the petitioner alone for Falsification of Public Documents under Art. 171 par. 4 of the
RPC.

The Ombudsman filed the corresponding informations with the Sandiganbayan.

The Sandiganbayan ordered the Office of the Special Prosecutor (OSP) to conduct a
reinvestigation. The Sandiganbayan gave the petitioner 10 days to file his counter-affidavit with
the OSP. Instead of submitting his counter-affidavit, petitioner asked for another 3-day extension
and afterwhich another 20-day extension.

Despite the given extensions, the petitioner still failed to file his counter-affidavit prompting
prosecutor Norberto B. Ruiz to declare the petitioner to have waived his right to submit
countervailing evidence. Ombudsman approved the resolution. After several extensions sought
and granted, the petitioner filed a Motion to Quash and/or Reinvestigation for the criminal cases
against him. The Sandiganbayan denied the petitioner’s motion because of the pending OSP
reinvestigation this, despite the OSPs earlier termination of the reinvestigation for the
petitioner’s continuous failure to submit his counteraffidavit.

After the arraignment, the OSP file a Motion to Suspend Pendente Lite. The petitioner filed his
Vigorous Opposition based on the obvious and fatal defect in the information. The
Sandiganbayan promulgated the assailed resolution suspending the petitioner pendent lite.

The petitioner moved for reconsideration of his suspension order and demanded pre-suspension
hearing. The Sandiganbayan denied his motion, prompting him to file this certiorari petition to
challenge the validity of the suspension order.

ISSUE/S: Whether the absence of an actual pre-suspension hearing renders invalid the
suspension order against the petitioner.

HELD: No. In Bedruz vs Sandiganbayan, the court considered the opposition of the accused
(Motion to Suspend Pendente Lite) as sufficient to dispense with the need to actually set the
prosecution’s motion for hearing.

In the case at bar, while there was no pre-suspension hearing held to determine the validity of the

22
ERDIE E. AMBROCIO Remedial Law Review 1
2014-0308 Criminal Procedure – Case Digests

information/suspension, the court believed that the pleadings filed for and against them achieved
the goal of the procedure. The right to due process is satisfied not by just oral arguments but by
the filing and the consideration by the court of the parties’ pleadings, memoranda and other
position papers.

Since a pre-suspension hearing is basically a due process requirement, when an accused public
official is given an adequate opportunity to be heard on his possible defenses against a
mandatory suspension under RA 3019, then an accused would have no reason to complain that
no actual hearing was conducted. It is well settled that to be heard does not only mean oral
arguments in court; one may be heard also through pleadings. Where opportunity to be heard,
either through oral arguments or pleadings, has been accorded, no denial of procedural due
process exists.

23
ERDIE E. AMBROCIO Remedial Law Review 1
2014-0308 Criminal Procedure – Case Digests

13. PEOPLE VS SORIA


685 SCRA

FACTS: The Prosecution alleges that the victim, AAA, is the daughter of the accused. On
February 26, 2000, AAA and her siblings enjoyed the spaghetti their father brought home for
merienda. After eating, AAA went to the bedroom to rest. Thereafter, appellant also entered the
room and positioned himself on top of AAA, took off her clothes and “felt” that he inserted his
penis into her vagina. AAA felt intense pain from her breast down to her vagina and thus told her
father that it was painful. At that point, appellant apologized to his daughter, stood up, and left
the room. The whole incident was witnessed by AAA’s brother, BBB. The pain persisted until
AAA’s vagina started to bleed. She thus told her aunt about it and they proceeded to a hospital
for treatment. Her mother was also immediately informed of her ordeal. AAA was taken into the
custody of the Department of Social Welfare and Development.

On the contrary, the Defense alleges that the appellant admitted that he was at home on the day
and time of AAA’s alleged rape but denied committing the same. Instead, he claimed that the
filing of the rape case against him was instigated by his wife, whom he confronted about her
illicit affair with a man residing in their community. According to appellant, he could not have
molested AAA because he treated her well. In fact, he was the only one sending his children to
school since his wife already neglected them and seldom comes home.

ISSUE/S:
1. Whether the allegations of the accused is credible to cast a reasonable doubt which would
warrant his acquittal?
2. Whether the information is sufficient to justify the conviction of the accused?

HELD: Rape can now be committed either through sexual intercourse or by sexual assault. Rape
under paragraph 1 of Article 266-A is referred to as rape through sexual intercourse. Carnal
knowledge is the central element and it must be proven beyond reasonable doubt. On the other
hand, rape under paragraph 2 of Article 266-A is commonly known as rape by sexual assault.
The perpetrator commits this kind of rape by inserting his penis into another person’s mouth or
anal orifice, or any instrument or object into the genital or anal orifice of another person.

The RTC and the CA found the accused guilty of rape through sexual intercourse but It is
evident from the testimony of AAA that she was unsure whether it was indeed appellant’s penis
which touched her labia and entered her organ. AAA stated that she only knew that it was the
“bird” of her father which was inserted into her vagina after being told by her brother BBB.
Clearly, AAA has no personal knowledge that it was appellant’s penis which touched her labia
and inserted into her vagina. Hence, it would be erroneous to conclude that there was penile
contact based solely on the declaration of AAA’s brother, BBB, which declaration was hearsay
due to BBB’s failure to testify.

The court however found it inconsequential that AAA could not specifically identify the
particular instrument or object that was inserted into her genital. What is important and relevant
is that indeed something was inserted into her vagina. Moreover, the prosecution satisfactorily

24
ERDIE E. AMBROCIO Remedial Law Review 1
2014-0308 Criminal Procedure – Case Digests

established that appellant accomplished the act of sexual assault through his moral ascendancy
and influence over “AAA” which substituted for violence and intimidation. Thus, there is no
doubt that appellant raped AAA by sexual assault. It is also improbable for appellant’s wife to
have dared encourage their daughter AAA to file the charges publicly expose the dishonor of the
family unless the rape was indeed committed.

The Information in this case did not specify with certainty whether appellant committed the rape
through sexual intercourse under paragraph 1 of Article 266-A, or rape by sexual assault as
described in paragraph 2 thereof. The Information stated that appellant inserted his penis into the
genital of "AAA," which constituted rape by sexual intercourse under the first paragraph of
Article 266-A. At the same time, the Information alleged that appellant used force and
intimidation to commit an act of sexual assault. While these allegations cause ambiguity, they
only pertain to the mode or manner of how the rape was committed and the same do not
invalidate the Information or result in the automatic dismissal of the case. "[W]here an offense
may be committed in any of the different modes and the offense is alleged to have been
committed in two or more modes specified, the indictment is sufficient, notwithstanding the fact
that the different means of committing the same offense are prohibited by separate sections of
the statute. The allegation in the information of the various ways of committing the offense
should be regarded as a description of only one offense and the information is not thereby
rendered defective on the ground of multifariousness." Any objection from the appellant with
respect to the Information is held to have been waived failing any effort to oppose the same
before trial. He therefore can be convicted of rape through sexual intercourse or rape by sexual
assault, depending on the evidence adduced during trial.

In determining whether appellant is indeed guilty of rape through sexual intercourse under
paragraph 1 of Article 266-A, it is essential to establish beyond reasonable doubt that he had
carnal knowledge of "AAA". There must be proof that his penis touched the labia of "AAA" or
slid into her female organ, and not merely stroked the external surface thereof, to ensure his
conviction of rape by sexual intercourse.

We reviewed the testimony of "AAA" and found nothing therein that would show that she was
raped through sexual intercourse. While "AAA" categorically stated that she felt something
inserted into her vagina, her testimony was sorely lacking in important details that would
convince us with certainty that it was indeed the penis of appellant that was placed into her
vagina. Based on the evidence adduced, the accused is found guilty beyond reasonable doubt for
the crime of rape by sexual assault.

25
ERDIE E. AMBROCIO Remedial Law Review 1
2014-0308 Criminal Procedure – Case Digests

14. UNION BANK VS PEOPLE


667 SCRA 113

FACTS: Desi Tomas was charged in court for perjury under Article 183 of the Revised Penal
Code (RPC) for making a false narration in a Certificate against Forum Shopping.
The accusation stemmed from Union Bank’s two (2) complaints for sum of money with prayer
for a writ of replevin against the spouses Eddie and Eliza Tamondong and a John Doe, filed
before the RTC, Branch 109, Pasay City; and filed and raffled to the MeTC, Branch 47, Pasay
City. Both complaints showed that Tomas executed and signed the Certification against Forum
Shopping.

Accordingly, she was charged of deliberately violating Article 183 of the RPC by falsely
declaring under oath in the Certificate against Forum Shopping in the second complaint that she
did not commence any other action or proceeding involving the same issue in another tribunal or
agency.

Tomas filed a Motion to Quash, arguing, among others, that the venue was improperly laid since
it is the Pasay City court (where the Certificate against Forum Shopping was submitted and used)
and not the MeTC-Makati City (where the Certificate against Forum Shopping was subscribed)
that has jurisdiction over he perjury case. Second, she argued that the facts charged do not
constitute an offense because: (a) the third element of perjury – the willful and deliberate
assertion of falsehood – was not alleged with particularity without specifying what the other
action or proceeding commenced involving the same issues in another tribunal or agency; (b)
there was no other action or proceeding pending in another court when the second complaint was
filed; and (c) she was charged with perjury by giving false testimony while the allegations in the
Information make out perjury by making a false affidavit.

The MeTC-Makati City denied the Motion to Quash and subsequently denied Tomas’ motion for
reconsideration. Union Bank and Tomas filed a petition for certiorari before the RTC-Makati
City to annul and set aside the MeTC Makati City orders on the ground of grave abuse of
discretion, which was dismissed. The RTC-Makati City subsequently denied Union Bank and
Tomas’ motion for reconsideration. Hence, the review under Rule 45.

ISSUE/S: Whether or not Makati City, where the Certificate Against Forum Shopping was
notarized, is the proper venue for the case of perjury.

HELD: Yes. Venue is an essential element of jurisdiction in criminal cases. It determines not
only the place where the criminal action is to be instituted, but also the court that has the
jurisdiction to try and hear the case. The reason for this rule is two- fold. First, the jurisdiction of
trial courts is limited to well-defined territories such that a trial court can only hear and try cases
involving crimes committed within its territorial jurisdiction. Second, laying the venue in the
locus criminis is grounded on the necessity and justice of having an accused on trial in the
municipality of province where witnesses and other facilities for his defense are available.

Unlike in civil cases, a finding of improper venue in criminal cases carries jurisdictional

26
ERDIE E. AMBROCIO Remedial Law Review 1
2014-0308 Criminal Procedure – Case Digests

consequences. In determining the venue where the criminal action is to be instituted and the
court which has jurisdiction over it, Section 15(a), Rule 110 of the 2000 Revised Rules of
Criminal Procedure provides that subject to existing laws, the criminal action shall be instituted
and tried in the court or municipality or territory where the offense was committed or where any
of its essential ingredients occurred. The above provision should be read in light of Section 10,
Rule 110 of the 2000 Revised Rules of Criminal Procedure which states: Place of commission of
the offense. – The complaint or information is sufficient if it can be understood from its
allegations that the offense was committed or some of its essential ingredients occurred at some
place within the jurisdiction of the court, unless the particular place where it was committed
constitutes an essential element of the offense charged or is necessary for its identification.

Both provisions categorically place the venue and jurisdiction over criminal cases not only in the
court where the offense was committed, but also where any of its essential ingredients took
place. In other words, the venue of action and of jurisdiction are deemed sufficiently alleged
where the Information states that the offense was committed or some of its essential ingredients
occurred at a place within the territorial jurisdiction of the court.

27
ERDIE E. AMBROCIO Remedial Law Review 1
2014-0308 Criminal Procedure – Case Digests

15. SOLIDUM VS PEOPLE


718 SCRA 263

FACTS: Two days after his birth, Gerald Gercayo, who was born with an imperforate anus,
underwent colostomy, enabling him to excrete through a colostomy bag attached to the side of
his body.

In 1995, Gerald, then three years old, was admitted at the Ospital ng Maynila for a pull-through
operation. Dr. Leandro Resurreccion headed the surgical team, and was assisted by Dr. Joselito
Luceño, Dr. Donatella Valeña and Dr. Joseph Tibio. The anesthesiologists included Dr. Marichu
Abella, Dr. Arnel Razon and Dr. Fernando Solidum.

During the operation, Gerald experienced bradycardia, and went into a coma. His coma lasted for
two weeks, but he regained consciousness only after a month. He could no longer see, hear or
move. Ma. Luz Gercayo (Luz) then lodged a complaint for reckless imprudence resulting in
serious physical injuries with the City Prosecutor’s Office of Manila.

Upon a finding of probable cause, the City Prosecutor’s Office filed an information solely
against Dr. Solidum. Dr. Solidum were pronounced guilty of reckless imprudence resulting in
serious physical injuries by the Regional Trial Court (RTC) and the Court of Appeals (CA). He
then filed an appeal to the SC. The SC acquitted Dr. Solidum because circumstances, taken
together, did not prove beyond reasonable doubt that Dr. Solidum had been recklessly imprudent
in administering the anesthetic agent to Gerald.

ISSUE/S:
1. Whether or not Dr. Solidum, through his acquittal, is exempt from civil liability.
2. Whether or not Ospital ng Maynila should be adjudged jointly and severally liable with
the accused.

HELD:
1. Yes.
The circumstances that have been established do not present the factual and legal bases for
validly doing so. His acquittal did not derive only from reasonable doubt. There was really no
firm and competent showing how the injury to Gerard had been caused. That meant that the
manner of administration of the anesthesia by Dr. Solidum was not necessarily the cause of the
hypoxia that caused the bradycardia experienced by Gerard. Consequently, to adjudge Dr.
Solidum civilly liable would be to speculate on the cause of the hypoxia. We are not allowed to
do so, for civil liability must not rest on speculation but on competent evidence.

2. No.
In criminal prosecutions, the civil action for the recovery of civil liability that is deemed
instituted with the criminal action refers only to that arising from the offense charged. Ospital ng
Maynila, being an artificial entity, had not been charged along with Dr. Solidum. The lower
courts thereby acted capriciously and whimsically, which rendered their judgment against
Ospital ng Maynila void as the product of grave abuse of discretion amounting to lack of

28
ERDIE E. AMBROCIO Remedial Law Review 1
2014-0308 Criminal Procedure – Case Digests

jurisdiction.

For one, Ospital ng Maynila was not at all a party in the proceedings. Hence, its fundamental
right to be heard was not respected from the outset. The RTC and the CA should have been alert
to this fundamental defect. Verily, no person can be prejudiced by a ruling rendered in an action
or proceeding in which he was not made a party. Such a rule would enforce the constitutional
guarantee of due process of law.

Moreover, Ospital ng Maynila could be held civilly liable only when subsidiary liability would
be properly enforceable pursuant to Article 103 of the Revised Penal Code. But the subsidiary
liability seems far-fetched here. The conditions for subsidiary liability to attach to Ospital ng
Maynila should first be complied with. Firstly, pursuant to Article 103 of the Revised Penal
Code, Ospital ng Maynila must be shown to be a corporation "engaged in any kind of industry."
The term industry means any department or branch of art, occupation or business, especially one
that employs labor and capital, and is engaged in industry. However, Ospital ng Maynila, being a
public hospital, was not engaged in industry conducted for profit but purely in charitable and
humanitarian work. Secondly, assuming that Ospital ng Maynila was engaged in industry for
profit, Dr. Solidum must be shown to be an employee of Ospital ng Maynila acting in the
discharge of his duties during the operation on Gerald. Yet, he definitely was not such employee
but a consultant of the hospital. And, thirdly, assuming that civil liability was adjudged against
Dr. Solidum as an employee (which did not happen here), the execution against him was
unsatisfied due to his being insolvent.

29
ERDIE E. AMBROCIO Remedial Law Review 1
2014-0308 Criminal Procedure – Case Digests

16. CASTILLO VS SALVADOR


JULY 2014

FACTS: Phillip R. Salvador and his brother, Ramon Castillo, was charged with Estafa under
Article 315, paragraph 2 (a) of the Revised Penal Code by petitioner Cristina B. Castillo, a
businesswoman engaged in real estate business, educational institution, boutique, and trading
business. She was then enticed by Phillip and Ramon to engage in freight and remittance
business.

As petitioner had deeply fallen in love with respondent Salvador and since she trusted him very
much as he even acted as a father to her children while her annulment was ongoing, she agreed
to embark on the remittance business. She agreed with respondent and Ramon that any profit
derived from the business would be equally divided among them and that respondent would be in
charge of promotion and marketing in Hong Kong, and Ramon would take charge of the
operations of business in the Philippines and she would be financing the business.

The business has not operated yet as petitioner was still raising the amount of US$100,000.00 as
capital for the actual operation. When petitioner already had the money, she handed the same to
respondent Salvador, which was witnessed by her disabled half-brother Enrico B. Tan. However,
the proposed business never operated as respondent only stayed in Hong Kong for three days.
When she asked respondent about the money and the business, the latter told her that the money
was deposited in a bank. However, upon further query, respondent confessed that he used the
money to pay for his other obligations. Since then, the US$100,000.00 was not returned at all.

Upon their arraignment, Phillip and Ramon pleaded not guilty to the offense charged. Trial on
the merits thereafter ensued. The RTC rendered a Decision finding Phillip guilty beyond
reasonable doubt and ordered him to pay Castillo, US$100,000.00 or its equivalent in Philippine
currency. Ramon was acquitted for insufficiency of evidence. Phillip appealed his conviction to
the CA, which overturned the RTC decision. Castillo filed a petition for review on certiorari on
the civil aspect of the case, arguing that the Court should have at least retained the amount of
damages to her.

ISSUE/S: Whether or not the acquitted accused are still liable for damages.

HELD: Yes. In Manantan v. CA, we discussed the consequences of an acquittal on the civil
liability of the accused as follows. Our law recognizes two kinds of acquittal, with different
effects on the civil liability of the accused. First is an acquittal on the ground that the accused is
not the author of the actor omission complained of. This instance closes the door to civil liability,
for a person who has been found to be not the perpetrator of any act or omission cannot and can
never be held liable for such act or omission. There being no delict, civil liability ex delicto is out
of the question, and the civil action, if any, which may be instituted must be based on grounds
other than the delict complained of. This is the situation contemplated in Rule III of the Rules of
Court. The second instance is an acquittal based on reasonable doubt on the guilt of the accused.

In this case, even if the guilt of the accused has not been satisfactorily established, he is not

30
ERDIE E. AMBROCIO Remedial Law Review 1
2014-0308 Criminal Procedure – Case Digests

exempt from civil liability, which may be proved by preponderance of evidence only. This is the
situation contemplated in Article 29 of the Civil Code, where the civil action for damages is "for
the same act or omission." x x x. A reading of the CA decision would show that Phillip was
acquitted because the prosecution failed to prove his guilt beyond reasonable doubt. [S]ince the
acquittal is based on reasonable doubt, [Phillip] is not exempt from civil liability which may be
proved by preponderance of evidence only. In Encinas v. National Bookstore, Inc., we explained
the concept of preponderance of evidence as follows: x x x Preponderance of evidence is the
weight, credit, and value of the aggregate evidence on either side and is usually considered to be
synonymous with the term "greater weight of the evidence" or "greater weight of the credible
evidence." Preponderance of evidence is a phrase, which in the last analysis, means probability
of the truth. It is evidence, which is more convincing to the court as worthy of belief than that
which is offered in opposition thereto.

In discrediting [Castillo's] allegation that she gave [Phillip] US$100,000.00 in May 2002, the CA
found that: (1) [Castillo] failed to show how she was able to raise the money in such a short
period of time and even gave conflicting versions on the source of the same; (2) [Castillo]failed
to require respondent to sign a receipt so she could have a record of the transaction and offered
no plausible reason why the money was allegedly hand-carried to Hong Kong; (3) [Castillo's]
claim of trust as reason for not requiring [Phillip] to sign a receipt was inconsistent with the way
she conducted her previous transactions with him; and (4) [Castillo's] behavior after the alleged
fraud perpetrated against her was inconsistent with the actuation of someone who had been
swindled.

31
ERDIE E. AMBROCIO Remedial Law Review 1
2014-0308 Criminal Procedure – Case Digests

17. LIM VS. KOU CO PING


679 SCRA 114

FACTS: FR Cement Corporation (FRCC), owner/operator of a cement manufacturing plant,


issued several withdrawal authorities for the account of cement dealers and traders, Fil-Cement
Center and Tigerbilt. These withdrawal authorities state the number of bags that the dealer/trader
paid for and can withdraw from the plant. Each withdrawal authority contained a provision that it
is valid for six months from its date of issuance, unless revoked by FRCC Marketing
Department.

Fil-Cement Center and Tigerbilt, through their administrative manager, Gail Borja (Borja), sold
the withdrawal authorities covering 50,000 bags of cement to Co for the amount of P3.15 million
or P63.00 per bag. Co sold these withdrawal authorities to Lim allegedly at the price of P64.00
per bag or a total of P3.2 million. Using the withdrawal authorities, Lim withdrew the cement
bags from FRCC on a staggered basis. She successfully withdrew 2,800 bags of cement, and sold
back some of the withdrawal authorities, covering 10,000 bags, to Co. FRCC did not allow Lim
to withdraw the remaining 37,200 bags covered by the withdrawal authorities. Lim clarified the
matter with Co and Borja, who explained that the plant implemented a price increase and would
only release the goods once Lim pays for the price difference or agrees to receive a lesser
quantity of cement. Lim objected and maintained that the withdrawal authorities she bought were
not subject to price fluctuations. Lim sought legal recourse after her demands for Co to resolve
the problem with the plant or for the return of her money had failed.

An Information for Estafa through Misappropriation or Conversion was filed against Co. The
private complainant, Lily Lim, participated in the criminal proceedings to prove her damages.
She prayed for Co to return her money amounting to P2,380,800.00, foregone profits, and legal
interest, and for an award of moral and exemplary damages, as well as attorney’s fees.

The RTC of Pasay acquitted Co for insufficiency of evidence. After the trial on the civil aspect
of the criminal case, the Pasig City RTC also relieved Co of civil liability to Lim. Lim sought a
reconsideration of the above Order, arguing that she has presented preponderant evidence that Co
committed estafa against her. The trial court denied the motion. Lim filed her notice of appeal. A
month later, Lim filed a complaint for specific performance and damages before Branch 21 of
the RTC of Manila. The defendants in the civil case were Co and all other parties to the
withdrawal authorities, Tigerbilt, Fil-Cement Center, FRCC, Southeast Asia Cement, and La
Farge Corporation. The complaint asserted two causes of action: breach of contract and abuse of
rights.

ISSUE/S: Whether or not Lim committed forum shopping in filing the civil case for specific
performance and damages during the pendency of her appeal on the civil aspect of the criminal
case for estafa.

HELD: No. A single act or omission that causes damage to an offended party may give rise to
two separate civil liabilities on the part of the offender: (1) civil liability ex delicto, that is, civil
liability arising from the criminal offense under Article 100 of the Revised Penal Code, and (2)

32
ERDIE E. AMBROCIO Remedial Law Review 1
2014-0308 Criminal Procedure – Case Digests

independent civil liability, that is, civil liability that may be pursued independently of the
criminal proceedings. The independent civil liability may be based on “an obligation not arising
from the act or omission complained of as a felony,” as provided in Article 31 of the Civil Code
(such as for breach of contract or for tort). It may also be based on an act or omission that may
constitute felony but, nevertheless, treated independently from the criminal action by specific
provision of Article 33 of the Civil Code (“in cases of defamation, fraud and physical injuries”).

The civil liability arising from the offense or ex delicto is based on the acts or omissions that
constitute the criminal offense; hence, its trial is inherently intertwined with the criminal action.
For this reason, the civil liability ex delicto is impliedly instituted with the criminal offense. If
the action for the civil liability ex delicto is instituted prior to or subsequent to the filing of the
criminal action, its proceedings are suspended until the final outcome of the criminal action. The
civil liability based on delict is extinguished when the court hearing the criminal action declares
that “the act or omission from which the civil liability may arise did not exist.”

The independent civil liabilities are separate from the criminal action and may be pursued
independently, as provided in Articles 31 and 33 of the Civil Code, which state that:

ART. 31. When the civil action is based on an obligation not arising from the act
or omission complained of as a felony, such civil action may proceed
independently of the criminal proceedings and regardless of the result of the
latter.

ART. 33. In cases of defamation, fraud, and physical injuries a civil action for
damages, entirely separate and distinct from the criminal action, may be brought
by the injured party. Such civil action shall proceed independently of the criminal
prosecution, and shall require only a preponderance of evidence.

Because of the distinct and independent nature of the two kinds of civil liabilities, jurisprudence
holds that the offended party may pursue the two types of civil liabilities simultaneously or
cumulatively, without offending the rules on forum shopping, litis pendentia, or res judicata.

33
ERDIE E. AMBROCIO Remedial Law Review 1
2014-0308 Criminal Procedure – Case Digests

18. CASUPANAN VS LAROYA


388 SCRA 28

FACTS: Two vehicles, one driven by respondent Laroya and the other owned by petitioner
Capitulo and driven by petitioner Casupanan, figured in an accident. As a result, two cases were
filed with the MCTC. Laroya filed a criminal case against Casupanan for reckless imprudence
resulting in damage to property, while Casupanan and Capitulo filed a civil case against Laroya
for quasi- delict.

When the civil case was filed, the criminal case was then at its preliminary investigation stage.
Laroya, defendant in the civil case, filed a motion to dismiss the civil case on the ground of
forum-shopping considering the pendency of the criminal case.

The MCTC granted the motion and dismissed the civil case. On Motion for Reconsideration,
Casupanan and Capitulo insisted that the civil case is a separate civil action, which can proceed
independently of the criminal case. The MCTC denied the motion for reconsideration.
Casupanan and Capitulo filed a petition for certiorari under Rule 65 before the RTC assailing the
MCTC’s Order of dismissal.

The RTC dismissed the petition for lack of merit. The Capas RTC ruled that the order of
dismissal issued by the MCTC is a final order which disposes of the case and therefore the
proper remedy should have been an appeal. The Capas RTC further held that a special civil
action for certiorari is not a substitute for a lost appeal. Finally, the Capas RTC declared that
even on the premise that the MCTC erred in dismissing the civil case, such error is a pure error
of judgment and not an abuse of discretion. Casupanan and Capitulo filed a Motion for
Reconsideration but the Capas RTC denied the same.

ISSUE/S: Whether Casupanan and Capitulo, who are not the offended parties in the criminal
case, can file a separate civil action against the offended party in the criminal case.

HELD: Yes. Under Section 1 of the present Rule 111, the independent civil action in Articles
32, 33, 34 and 2176 of the Civil Code is not deemed instituted with the criminal action but may
be filed separately by the offended party even without reservation. The commencement of the
criminal action does not suspend the prosecution of the independent civil action under these
articles of the Civil Code. The suspension in Section 2 of the present Rule 111 refers only to the
civil action arising from the crime, if such civil action is reserved or filed before the
commencement of the criminal action. Thus, the offended party can file two separate suits for the
same act or omission. The first a criminal case where the civil action to recover civil liability ex-
delicto is deemed instituted, and the other a civil case for quasi-delict—without violating the rule
on non-forum shopping. The two cases can proceed simultaneously and independently of each
other. The commencement or prosecution of the criminal action will not suspend the civil action
for quasi-delict. The only limitation is that the offended party cannot recover damages twice for
the same act or omission of the defendant. In most cases, the offended party will have no reason
to file a second civil action since he cannot recover damages twice for the same act or omission
of the accused. In some instances, the accused may be insolvent, necessitating the filing of

34
ERDIE E. AMBROCIO Remedial Law Review 1
2014-0308 Criminal Procedure – Case Digests

another case against his employer or guardians.

Similarly, the accused can file a civil action for quasi-delict for the same act or omission he is
accused of in the criminal case. This is expressly allowed in paragraph 6, Section 1 of the present
Rule 111 which states that the counterclaim of the accused “may be litigated in a separate civil
action.” This is only fair for two reasons. First, the accused is prohibited from setting up any
counterclaim in the civil aspect that is deemed instituted in the criminal case. The accused is
therefore forced to litigate separately his counterclaim against the offended party. If the accused
does not file a separate civil action for quasi-delict, the prescriptive period may set in since the
period continues to run until the civil action for quasi-delict is filed. Second, the accused, who is
presumed innocent, has a right to invoke Article 2177 of the Civil Code, in the same way that the
offended party can avail of this remedy which is independent of the criminal action. To disallow
the accused from filing a separate civil action for quasi-delict, while refusing to recognize his
counterclaim in the criminal case, is to deny him due process of law, access to the courts, and
equal protection of the law. Thus, the civil action based on quasi-delict filed separately by
Casupanan and Capitulo is proper.

35
ERDIE E. AMBROCIO Remedial Law Review 1
2014-0308 Criminal Procedure – Case Digests

19. CATERPILLAR, INC. VS SAMSON


November 9, 2016

FACTS: Petitioner Caterpillar, Inc. is a foreign corporation engaged in the business of


manufacturing shoes, clothing items, among others. Search warrant applications were filed
against Manolo Samson (herein after referred to as Samson) for violations of unfair competition
provided under Section 168.3(a) in relation to Sections 131.3, 123(e) and 170 of Republic Act
No. 8293, otherwise known as the Intellectual Property Code. Search warrants were then issued
against respondent Samson and his other business establishments (Itti Shoes Corporation,
Kolm’s Manufacturing, and Caterpillar Boutique and General Merchandise). Pursuant to the
search warrants various merchandise garments, footwear, bags, wallets, deodorant sprays, shoe
cleaners and accessories, all bearing the trademarks "CAT," "CAT AND DESIGN,"
"CATERPILLAR," "CATERPILLAR AND DESIGN," "WALKING MACHINES" and/or
"Track-type Tractor and Design" were seized. Respondent Samson filed a motion to quash the
search warrants but was denied. However, the Court ordered the release of the articles that were
seized since there was no criminal action filed against the respondent. Petitioner then filed
Motion for Partial Reconsideration but was denied by the TC. CA also denied the petition after
noting that all the criminal complaints that were filed against the respondent were dismissed by
the investigating prosecutor and that the respondent never denied the existence of the said items.

ISSUE/S:
1) Whether or not CA erred in upholding the immediate release of the seized items on the
ground that there was no criminal action for unfair competition filed against the
respondent?

2) Whether or not the CA erred in ruling that the subsequent dismissal of the investigating
State Prosecutor of the criminal complaints against respondent justifies the return of the
seized items?

HELD:

1) No. The Joint Resolution of the DOJ has become final; therefore no criminal case was filed
against the respondent in relation with the five search warrants that were issued by the Trial
Court. There was also no criminal case filed against the articles that were seized. With these, the
seized articles should be immediately released. Also, the numerous articles of clothing, footwear
and accessories, among others, that were seized had little, if any, evidentiary value for the
criminal action of unfair competition.

An action for unfair competition is based on the proposition that no dealer in merchandise should
be allowed to dress his goods in simulation of the goods of another dealer, so that purchasers
desiring to buy the goods of the latter would be induced to buy the goods of the former. The most
usual devices employed in committing this crime are the simulation of labels and the
reproduction of form, color and general appearance of the package used by the pioneer
manufacturer or dealer. In the case at hand the respondent already admitted the existence of the
seized articles. The Court therefore ruled that the admissions of the respondent in the case at

36
ERDIE E. AMBROCIO Remedial Law Review 1
2014-0308 Criminal Procedure – Case Digests

hand are already suffiecient to establish that he used such trademarks in order to sell
merchandise at a commercial scale and that the actual products manufactured by the respondent
need not be presented to prove such fact. Also, there were already available samples from the
purchases as well as photographs of the particular parts of the merchandise where the trademark
in dispute were attached or used, therefore there is no more need for the court to take custody of
the countless articles that were seized.

2) No. In the case at hand there is no criminal action that has been filed. The Court therefore was
left with no custody of the highly depreciable merchandise that were seized. More importantly,
these highly depreciable articles would have been superfluous if presented as evidence for the
following reasons: (1) the respondent had already admitted that he is the owner of the
merchandise seized, which made use of the trademarks in dispute; (2) the court required the
respondent to execute an undertaking to produce the articles seized when the court requires and
had already in its possession a complete inventory of the items seized as secondary evidence; (3)
actual samples of the respondent’s merchandise are in the possession of the police officers who
had applied for the search warrant, and photographs thereof had been made part of the records,
and respondent did not dispute that these were obtained from his stores. Where the purpose of
presenting as evidence the articles seized is no longer served, there is no justification for severely
curtailing the rights of a person to his property. Hence, petition denied.

37
ERDIE E. AMBROCIO Remedial Law Review 1
2014-0308 Criminal Procedure – Case Digests

20. PEOPLE VS ROMERO


306 SCRA 90

FACTS: Complainant Ernesto A. Ruiz was a radio commentator of Radio DXRB, Butuan City.
He came to know the business of Surigao San Andres Industrial Development Corporation
(SAIDECOR) when he interviewed accused Martin Romero and Ernesto Rodriguez regarding
the corporation’s investment operations in Butuan City and Agusan del Norte.

Romero was the president and general manager of SAIDECOR, while Rodriguez was the
operations manager. SAIDECOR started its operation on August 24, 1989 as a marketing
business. Later, it engaged in soliciting funds and investments from the public. The corporation
guaranteed an 800% return on investment within fifteen (15) or twenty one (21) days. Investors
were given coupons containing the capital and the return on the capital collectible on the date
agreed upon. It stopped operations in September, 1989.

Complainant Ernesto A. Ruiz went to SAIDECOR office in Butuan City to make an investment,
accompanied by his friend Jimmy Acebu, and SAIDECOR collection agent Daphne Parrocho.
After handing over the amount of one hundred fifty thousand pesos (P150,000.00) to Ernesto
Rodriguez, complainant received a postdated Butuan City Rural Bank check instead of the usual
redeemable coupon. The check indicated P1,000,200.00 as the amount in words, but the amount
in figures was for P1,200,000.00, as the return on the investment. Complainant did not notice the
discrepancy. When the check was presented to the bank for payment on October 5, 1989, it was
dishonored for insufficiency of funds, as evidenced by the check return slip issued by the bank.

Both accused could not be located and demand for payment was made only sometime in
November 1989 during the preliminary investigation of this case. Accused responded that they
had no money. Daphne Parrocho testified that complainant, with his friend Jimmy Acebu,
approached her to invest the amount of P150,000.00 at SAIDECOR. As she has reached her
quota, and therefore, no longer authorized to receive the amount, she accompanied them to the
office of SAIDECOR at Ong Yiu District, Butuan City. Accused Ernesto Rodriguez accepted the
investment and issued the check signed by him and Martin Romero.

For their defense, accused Martin Romero testified that he issued a check in the amount of
P1,200,000.00 corresponding to the total of the P150,000.00 investment and the 800% return
thereon. He claimed that the corporation had a deposit of fourteen million pesos
(P14,000,000.00) at the time of the issuance of the check and four million pesos (P4,000,000.00)
at the time SAIDECOR stopped operations. Romero knew these things because he used to
monitor the funds of the corporation with the bank. He was not aware that the check he issued
was dishonored because he never had the occasion to meet the complainant again after the
September 14, 1989 transaction. He only came to know about this when the case was already
filed in court sometime in the second or third week of January 1990.

On appeal, both accused did not deny that complainant made an investment with SAIDECOR in
the amount of P150,000.00. However, they denied that deceit was employed in the transaction.
They assigned as errors: (1) their conviction under P.D. 1689 due to the prosecution’s failure to

38
ERDIE E. AMBROCIO Remedial Law Review 1
2014-0308 Criminal Procedure – Case Digests

establish their guilt beyond reasonable doubt; and (2) the trial court’s failure to consider the joint
stipulation of facts in their favor. Ernesto Rodriguez, died pending appeal.

ISSUE/S: What is the effect of the death of an accused on the case?

HELD: Pursuant to the doctrine established in People vs. Bayotas, the death of the accused
pending appeal of his conviction extinguishes his criminal liability as well as the civil liability ex
delicto. The criminal action is extinguished inasmuch as there is no longer a defendant to stand
as the accused, the civil action instituted therein for recovery of civil liability ex delicto is ipso
facto extinguished, grounded as it is on the criminal case.

Corollarily, the claim for civil liability survives notwithstanding the death of the accused, if the
same may also be predicated on a source of obligation other than delict. Thus, the outcome of
this appeal pertains only to the remaining accused-appellant, Martin L. Romero.

39
ERDIE E. AMBROCIO Remedial Law Review 1
2014-0308 Criminal Procedure – Case Digests

21. MAGISTRADO VS PEOPLE


527 SCRA 125

FACTS: Private respondent Elena M. Librojo filed a criminal complaint for perjury against
petitioner Magestrado with the Office of the City Prosecutor. After the filing of petitioner’s
counter-affidavit and the appended pleadings, the Office of the City Prosecutor recommended
the filing of an information for perjury against petitioner. Thus, Assistant City Prosecutor
Josephine Z. Fernandez filed an information for perjury with the MeTC.

Petitioner filed a motion for suspension of proceedings based on a prejudicial question. Petitioner
alleged that Civil Case No. Q-98-34349, a case for recovery of a sum of money pending before
the Regional Trial Court (RTC) of Quezon City, Branch 84, and Civil Case No. Q-9834308, a
case for Cancellation of Mortgage, Delivery of Title and Damages, pending before the RTC of
Quezon City, Branch 77, must be resolved first before Criminal Case No. 90721 may proceed
since the issues in the said civil cases are similar or intimately related to the issues raised in the
criminal action.

ISSUE/S: Whether it is proper to suspend Criminal Case No. 90721 for perjury pending final
outcome of Civil Case No. Q98-34349 and Civil Case No. Q-98-34308 on the ground of
prejudicial question.

HELD: No. Rule 111 of the Rules of Court provides that: Sec. 6. Suspension by reason of
prejudicial question.—A petition for suspension of the criminal action based upon the pendency
of a prejudicial question in a civil action may be filed in the office of the prosecutor or the court
conducting the preliminary investigation. When the criminal action has been filed in court for
trial, the petition to suspend shall be filed in the same criminal action at any time before the
prosecution rests. Sec. 7. Elements of prejudicial question.—The elements of a prejudicial
question are: (a) the previously instituted
civil action involves an issue similar or intimately related to the issue raised in the subsequent
criminal action; and (b) the resolution of such issue determines whether or not the criminal
action may proceed.

The rationale behind the principle of suspending a criminal case in view of a prejudicial question
is to avoid two conflicting decisions. A prejudial question is defined as that which arises in a
case the resolution of which is a logical antecedent of the issue involved therein, and the
cognizance of which pertains to another tribunal. The prejudicial question must be determinative
of the case before the court but the jurisdiction to try and resolve the question must be lodged in
another court or tribunal. It is a question based on a fact distinct and separate from the crime but
so intimately connected with it that it determines the guilt or innocence of the accused. For a
prejudicial question in a civil case to suspend criminal action, it must appear not only that said
case involves facts intimately related to those upon which the criminal prosecution would be
based but also that in the resolution of the issue or issues raised in the civil case, the guilt or
innocence of the accused would necessarily be determined.

Thus, for a civil action to be considered prejudicial to a criminal case as to cause the suspension

40
ERDIE E. AMBROCIO Remedial Law Review 1
2014-0308 Criminal Procedure – Case Digests

of the criminal proceedings until the final resolution of the civil case, the following requisites
must be present: (1) the civil case involves facts intimately related to those upon which the
criminal prosecution would be based; (2) in the resolution of the issue or issues raised in the civil
action, the guilt or innocence of the accused would necessarily be determined; and (3)
jurisdiction to try said question must be lodged in another tribunal.

If the resolution of the issue in the civil action will not determine the criminal responsibility of
the accused in the criminal action based on the same facts, or there is no necessity “that the civil
case be determined first before taking up the criminal case,” therefore, the civil case does not
involve a prejudicial question. Neither is there a prejudicial question if the civil and the criminal
action can, according to law, proceed independently of each other. However, the court in which
an action is pending may, in the exercise of sound discretion, and upon proper application for a
stay of that action, hold the action in abeyance to abide by the outcome of another case pending
in another court, especially where the parties and the issues are the same, for there is power
inherent in every court to control the disposition of cases on its dockets with economy of time
and effort for itself, for counsel, and for litigants.

Where the rights of parties to the second action cannot be properly determined until the questions
raised in the first action are settled, the second action should be stayed. The power to stay
proceedings is incidental to the power inherent in every court to control the disposition of the
cases on its dockets, considering its time and effort, those of counsel and the litigants. But if
proceedings must be stayed, it must be done in order to avoid multiplicity of suits and prevent
vexatious litigations, conflicting judgments, confusion between litigants and courts. It bears
stressing that whether or not the trial court would suspend the proceedings in the criminal case
before it is submitted to its sound discretion.

41
ERDIE E. AMBROCIO Remedial Law Review 1
2014-0308 Criminal Procedure – Case Digests

22. PIMENTEL VS PIMENTEL


630 SCRA 436

FACTS: On October 25, 2004, Maria Chrysantine Pimentel y Lacap filed an action for frustrated
parricide against Joselito R. Pimentel with the RTC Quezon City. Less than four months
thereafter, Joselito received summons to appear before the RTC in Antipolo City for the pre-trial
and trial of for Declaration of Nullity of Marriage between Maria and Joselito on the ground of
psychological incapacity. Petitioner filed an urgent motion to suspend the proceedings before the
RTC Quezon City on the ground of the existence of a prejudicial question. He asserted that since
the relationship between the offender and the victim is a key element in parricide, the outcome of
the civil case for Declaration of Nullity of Marriage would have a bearing in the criminal case
filed against him before the RTC Quezon City.

The RTC Quezon City denied the motion, holding that the pendency of the case before the RTC
Antipolo is not a prejudicial question that warrants the suspension of the criminal case before it.
It held that the issues in the parricide case are the injuries sustained by respondent and whether
the case could be tried even if the validity of petitioner’s marriage with respondent is in question.

Petitoner filed a petition for certiorari with application of WPI and/or TRO before the Court of
Appeals, assailing the denial of RTC Quezon City.

The CA dismissed the petition. The CA ruled that even if the marriage between petitioner and
respondent would be declared void, it would be immaterial to the criminal case because prior to
the declaration of nullity, the alleged acts constituting the crime of frustrated parricide had
already been committed. The Court of Appeals ruled that all that is required for the charge of
frustrated parricide is that at the time of the commission of the crime, the marriage is still
subsisting.

ISSUE/S: Whether the resolution of the action in annulment of marriage is a prejudicial


question;

HELD: No . The rule is clear that the civil action must be instituted first before the filing of the
criminal action. Clearly, the civil case for annulment was filed after the filing of the criminal
case for frustrated parricide. As such, the requirement of Section 7, Rule 111 of the 2000 Rules
on Criminal Procedure was not met since the civil action was filed subsequent to the filing of the
criminal action. Further, the resolution of the civil action is not a prejudicial question that would
warrant the suspension of the criminal action.

There is a prejudicial question when a civil action and a criminal action are both pending, and
there exists in the civil action an issue which must be preemptively resolved before the criminal
action may proceed because howsoever the issue raised in the civil action is resolved would be
determinative of the guilt or innocence of the accused in the criminal case. A prejudicial
question is defined as: “x x x one that arises in a case the resolution of which is a logical
antecedent of the issue involved therein, and the cognizance of which pertains to another
tribunal. It is a question based on a fact distinct and separate from the crime but so intimately

42
ERDIE E. AMBROCIO Remedial Law Review 1
2014-0308 Criminal Procedure – Case Digests

connected with it that it determines the guilt or innocence of the accused, and for it to suspend
the criminal action, it must appear not only that said case involves facts intimately related to
those upon which the criminal prosecution would be based but also that in the resolution of the
issue or issues raised in the civil case, the guilt or innocence of the accused would necessarily be
determined.”

The issue in the civil case for annulment of marriage under Article 36 of the Family Code is
whether petitioner is psychologically incapacitated to comply with the essential marital
obligations. The issue in parricide is whether the accused killed the victim. In this case, since
petitioner was charged with frustrated parricide, the issue is whether he performed all the acts of
execution which would have killed respondent as a consequence but which, nevertheless, did not
produce it by reason of causes independent of petitioner’s will.16 At the time of the commission
of the alleged crime, petitioner and respondent were married. The subsequent dissolution of their
marriage, in case the petition for Declaration of Nullity of Marriage is granted, it will have no
effect on the alleged crime that was committed at the time of the subsistence of the marriage. In
short, even if the marriage between petitioner and respondent is annulled, petitioner could still be
held criminally liable since at the time of the commission of the alleged crime, he was still
married to respondent.

43
ERDIE E. AMBROCIO Remedial Law Review 1
2014-0308 Criminal Procedure – Case Digests

23. J. M. DOMINGUEZ VS LICLICAN


764 SCRA 338

FACTS: During the annual stockholders meeting of petitioner JM Dominguez Agronomic


Company, Inc. (JMD) held at the Baguio City Country Club, the election for its new set of
directors was conducted. This event was presided by then company president respondent Cecilia
Liclican (Liclican), and attended by her co-respondents Norma Isip (Isip) and Purita Rodriguez,
and by petitioners Helen Dagdagan (Dagdagan), Patrick Pacis, Kenneth Pacis, and Shirley
Dominguez (Dominguez) as well. Conflict ensued when petitioners Patrick and Kenneth Pacis
were allegedly not allowed to vote on the ground that they are not registered stockholders of
JMD. As pointed out, it was their mother and grandmother, both deceased, who are the
stockholders in JMD, and that there is still no settlement of their respective estates to effectively
transfer their shares in the company to Patrick and Kenneth Pacis. But since the remaining
stockholders with outstanding shares constituted a quorum, the election of officers still
proceeded.

In reaction to the foregoing developments, petitioners filed a Complaint against respondents


before the RTC Baguio Branch 59 for nullification of meetings, election and acts of directors and
officers, injunction and other reliefs. After a failed mediation, was referred for appropriate
Judicial Dispute Resolution (JDR) to Branch 7 of the RTC. Meanwhile, petitioner stockholders
immediately took hold of corporate properties, represented themselves to JMD’s tenants as the
true and lawful directors of the company, and collected and deposited rents due the company to
its bank account.

Subsequently, JMD, represented by petitioners, executed an Affidavit-Complaint charging


respondents Liclican and Isip with qualified theft. Petitioners alleged in the complaint that
Liclican and Isip, without any authority whatsoever, conspired to withdraw the amount of
P852,024.19 from the corporation’s savings account with the Equitable-PCI Bank; and that the
following day, they issued a check in the amount of P200,000, payable to cash, and to be drawn
against JMD’s account with Robinson’s Savings Bank. In a separate complaint, the corporation
claimed that respondents Liclican and Isip likewise issued a check payable to one Atty.
Francisco Lava, Jr. for P200,000 to be debited from the corporation’s account. The City
Prosecutor of Baguio City recommends for approval of the Informations for Qualified Theft
against LICLICAN and ISIP. Thereafter, Judge Tiongson-Tabora of RTC Baguio found probable
cause and issued a warrant of arrest.

In due time, respondents lodged a petition for certiorari with the CA, to annul and set aside the
two Orders by the RTC, branch 7 anchored, among others, on the alleged existence of a
prejudicial question. According to respondents, petitioner stockholders, by filing the complaint-
affidavit, are already assuming that they are the legitimate directors of JMD, which is the very
issue in the intra-corporate dispute pending in the RTC, Branch 59.

The CA granted the petition, holding that Judge Tiongson-Tabora should have refrained from
determining probable cause since she is well aware of the pendency of the issue on the validity
of JMD’s elections. As the judge overseeing the JDR of the said intra-corporate dispute, she

44
ERDIE E. AMBROCIO Remedial Law Review 1
2014-0308 Criminal Procedure – Case Digests

knew that there was still doubt as to who the rightfully elected directors of JMD are and,
corollarily, who would have the authority to initiate the criminal proceedings for qualified theft.

The CA further noted that even as corporate officers, as they claim to be, petitioners Dagdagan
and Patrick Pacis cannot file the Complaint-Affidavit in the exercise of corporate powers without
authority from the board of directors under Sec. 23,18 in relation to Sec. 2519 of the Corporation
Code Section 23. The board of directors or trustees.—Unless otherwise provided in this Code,
the corporate powers of all corporations formed under this Code shall be exercised, all business
conducted and all property of such corporations controlled and held by the board of directors or
trustees to be elected from among the holders of stocks, or where there is no stock, from among
the members of the corporation, who shall hold office for one (1) year until their successors are
elected and qualified. x x x

ISSUE/S:
(1) Whether or not the civil case constituted a prejudicial question warranting the suspension of
criminal proceedings;
(2) Whether or not there was grave abuse of discretion on the part of Judge Tabora in
disregarding the pending case on the validity of JMD’s election, by issuing the warrants.

HELD:
(1) Yes. As jurisprudence elucidates, a prejudicial question generally exists in a situation where
a civil action and a criminal action are both pending, and there exists in the former an issue that
must be preemptively resolved before the latter may proceed, because howsoever the issue raised
in the civil action is resolved would be determinative juris et de jure of the guilt or innocence of
the accused in the criminal case. The rationale behind the principle is to avoid two conflicting
decisions, and its existence rests on the concurrence of two essential elements: (i) the civil action
involves an issue similar or intimately related to the issue raised in the criminal action; and (ii)
the resolution of such issue determines whether or not the criminal action may proceed.

Here, the CA aptly observed the intra-corporate dispute, posed a prejudicial question to Criminal
Case. To be sure, the Civil involves the same parties herein, and is for nullification of JMD’s
meetings, election and acts of its directors and officers, among others. Court intervention was
sought to ascertain who between the two contesting group of officers should rightfully be seated
at the company’s helm. Without resolution of the civil case, petitioners’ authority to commence
and prosecute the Criminal case against respondents for qualified theft in JMD’s behalf remained
questionable, warranting the suspension of the criminal proceedings.

(2) Yes. In the case at bar, the CA correctly ruled that Judge Tiongson-Tabora acted with grave
abuse of discretion when she ordered the arrests of respondents Isip and Liclican despite the
existence of a prejudicial question. Judge Tiongson-Tabora cannot deny knowledge of the
pendency of Civil Case No. 6623-R as the judge presiding over its JDR. As correctly held by the
CA. Judge Tiongson-Tabora is well-aware of the existence of said prejudicial question that
should have barred the filing of the criminal complaint against petitioners Liclican and Isip, for
the simple reason that a juridical person can only act through its officers, and the issue in the
main case submitted for JDR before Judge Tiongson-Tabora is one for nullification of meetings,

45
ERDIE E. AMBROCIO Remedial Law Review 1
2014-0308 Criminal Procedure – Case Digests

election and act of directors and officers, injunction and other reliefs. Thus, she knows for a fact
that there is a question as to who are the legitimate directors of JMD such that there is doubt as to
whether private respondents are in a position to act for JMD.

46
ERDIE E. AMBROCIO Remedial Law Review 1
2014-0308 Criminal Procedure – Case Digests

24. FENEQUITO VS VERGARA, JR.


677 SCRA 113

FACTS: An Information for falsification of public documents was filed with the MTC of
Manila by the Assistant City Prosecutor of Manila against herein petitioners. Herein petitioners
filed a Motion to Dismiss the Case Based on Absence of Probable Cause. After respondent’s
Comment/Opposition was filed, the MeTC issued an Order dismissing the case on the ground of
lack of probable cause. Aggrieved, respondent, with the express conformity of the public
prosecutor, appealed the case to the Regional Trial Court (RTC) of Manila. The RTC rendered
judgment setting aside the Order of the MeTC and directing the said court to proceed to trial.

Petitioners then elevated the case to the CA via a petition for review. The CA rendered its
presently assailed Resolution dismissing the petition. The CA ruled that the Decision of the RTC
is interlocutory in nature and, thus, is not appealable. Petitioners filed a Motion for
Reconsideration but the CA denied the same.

Hence, the instant petition based on the following grounds that the (1) CA erred in outright
dismissal of the petition on the ground that the remedy is improper and (2) RTC Ruling is final
and unappealable. The petitioners relies on “Strict enforcement of the Rules may be suspended
whenever the purposes of justice so require.”

ISSUE/S:
(1) Whether or not the CA erred in dismissing the petition on improper remedy;
(2) Whether or not the RTC ruling is final and not appealable.

HELD:
(1) No. The Court notes at the outset that one of the grounds relied upon by the CA in
dismissing petitioners’ petition for review is the latter’s failure to submit copies of pleadings and
documents relevant and pertinent to the petition filed, as required under Section 2, Rule 42 of the
Rules of Court. While petitioners filed a Motion for Reconsideration, they, however, failed to
comply with these requirements. Worse, they did not even mention anything about it in the said
Motion. Section 3, Rule 42 of the same Rules provides:

“Sec. 3. Effect of failure to comply with requirements.—The failure of the


petitioner to comply with any of the foregoing requirements regarding the
payment of the docket and other lawful fees, the deposit for costs, proof of service
of the petition, and the contents of and the documents which should accompany
the petition shall be sufficient ground for the dismissal thereof.”

Moreover, it is a settled rule that the right to appeal is neither a natural right nor a part of due
process; it is merely a statutory privilege, and may be exercised only in the manner and in
accordance with the provisions of law. An appeal being a purely statutory right, an appealing
party must strictly comply with the requisites laid down in the Rules of Court. Deviations from
the Rules cannot be tolerated. The rationale for this strict attitude is not difficult to appreciate as
the Rules are designed to facilitate the orderly disposition of appealed cases. In an age where

47
ERDIE E. AMBROCIO Remedial Law Review 1
2014-0308 Criminal Procedure – Case Digests

courts are bedeviled by clogged dockets, the Rules need to be followed by appellants with
greater fidelity. Their observance cannot be left to the whims and caprices of appellants. In the
instant case, petitioners had all the opportunity to comply with the Rules. Nonetheless, they
remained obstinate in their non-observance even when they sought reconsideration of the ruling
of the CA dismissing their petition. Such obstinacy is incongruous with their late plea for
liberality in construing the Rules. On the above basis alone, the Court finds that the instant
petition is dismissible.

(2) No. A final order is one that which disposes of the whole subject matter or terminates a
particular proceeding or action, leaving nothing to be done but to enforce by execution what has
been determined. Upon the other hand, an order is interlocutory if it does not dispose of a case
completely, but leaves something more to be done upon its merits. The RTC Decision is beyond
cavil interlocutory in nature. It is essentially a denial of petitioners’ motion to quash because it
leaves something more to be done x x x, i.e., the continuation of the criminal proceedings until
the guilt or innocence of the accused is determined. Specifically, the MeTC has yet to arraign the
petitioners, then proceed to trial and finally render the proper judgment.

Petitioners contend that the PNP Crime Laboratory Questioned Document Report, submitted as
evidence by respondent to the prosecutor’s office, showed that the findings therein are not
conclusive and, thus, insufficient to support a finding of probable cause.

The Court is not persuaded. It is clear from a perusal of the cited PNP Crime Laboratory
Questioned Document Report No. 048-03 that the document examiner found that the signatures
appearing in the questioned Deed of Sale as compared to the standard signatures “reveal
divergences in the manner of execution and stroke structure [which is] an indication that they
were not written by one and the same person.” The Court agrees with the prosecutor’s
pronouncement in its Resolution dated September 22, 2003, that although the findings of the
PNP Crime Laboratory were qualified by the statement contained in the Report that “no definite
conclusion can be rendered due to the fact that questioned signatures are photocopies wherein
minute details are not clearly manifested,” the fact that an expert witness already found that the
questioned signatures were not written by one and the same person already creates probable
cause to indict petitioners for the crime of falsification of public document.

In the instant case, the Court finds no justification to depart from the ruling of the RTC that the
offense charged was committed and that herein petitioners are probably guilty thereof. With
respect to respondent’s legal personality to appeal the Order of the MeTC, suffice it to say that
the appeal filed with the RTC was made with the express conformity of the public prosecutor
who handles the case. It is wrong for petitioners to argue that it is the OSG which has authority
to file an appeal with the RTC. Section 35 (l), Chapter 12, Title III of Book IV of Executive
Order No. 292, otherwise known as the Administrative Code of 1987, mandates the OSG to
represent “the Government in the Supreme Court and the Court of Appeals in all criminal
proceedings.” On the other hand, Section 11 of Presidential Decree No. 1275, entitled
“Reorganizing the Prosecution Staff of the Department of Justice and the Offices of the
Provincial and City Fiscals, Regionalizing the Prosecution Service, and Creating the National
Prosecution Service,” which was the law in force at the time the appeal was filed, provides that

48
ERDIE E. AMBROCIO Remedial Law Review 1
2014-0308 Criminal Procedure – Case Digests

the provincial or the city fiscal (now referred to as prosecutor) “shall have charge of the
prosecution of all crimes, misdemeanors and violations of city or municipal ordinances in the
courts of such province or city and shall therein discharge all the duties incident to the institution
of criminal prosecutions.” In consonance with the above-quoted provision, it has been held by
this Court that the fiscal represents the People of the Philippines in the prosecution of offenses
before the trial courts at the metropolitan trial courts, municipal trial courts, municipal circuit
trial courts and the regional trial courts. Since the appeal, in the instant case was made with the
RTC of Manila, it is clear that the City Prosecutor or his assistant (in this case, the Assistant City
Prosecutor) had authority to file the same.

Moreover, petitioners’ reliance on Presidential Decree No. 911 is misplaced, as the cited
provision refers only to cases where the assistant fiscal or state prosecutor’s power to file an
information or dismiss a case is predicated or conditioned upon the prior authority or approval of
the provincial or city fiscal or the Chief State Prosecutor. There is nothing in the said law which
provides that in cases of appeal an Assistant City Prosecutor or a State Prosecutor may file the
same only upon prior authority or approval of the City Prosecutor or the Chief State Prosecutor.
Stated differently, unless otherwise ordered, an Assistant City Prosecutor or a State Prosecutor
may file an appeal with the RTC, questioning the dismissal by the MeTC of a case for lack of
probable cause, even without prior authority or approval of the City Prosecutor or the Chief State
Prosecutor.

49
ERDIE E. AMBROCIO Remedial Law Review 1
2014-0308 Criminal Procedure – Case Digests

25. BURGUNDY REALTY CORPORATION VS REYES


687 SCRA 524

FACTS: Private respondent Josefa “Jing” C. Reyes (Reyes), sometime in 1996, offered her
services to petitioner as the latter’s real estate agent in buying parcels of land in Calamba,
Laguna, which are to be developed into a golf course. She informed petitioner that more or less
ten (10) lot owners are her clients who were willing to sell their properties. Convinced of her
representations, petitioner released the amount of P23,423,327.50 in her favor to be used in
buying those parcels of land. Reyes, instead of buying those parcels of land, converted and
misappropriated the money given by petitioner to her personal use and benefit. Petitioner sent a
formal demand for Reyes to return the amount of P23,423,327.50, to no avail despite her receipt
of the said demand. As such, petitioner filed a complaint for the crime of Estafa against Reyes
before the Assistant City Prosecutor’s Office of Makati City.

Reyes, while admitting that she acted as a real estate agent for petitioner, denied having
converted or misappropriated the involved amount of money. She claimed that the said amount
was used solely for the intended purpose and that it was petitioner who requested her services in
procuring the lots. According to her, it was upon the petitioner’s prodding that she was
constrained to contact her friends who were also into the real estate business, including one
named Mateo Elejorde.

Meanwhile, Reyes received information that her sub-broker Mateo Elejorde had been depositing
the involved money entrusted to him under his personal account. On March 28, 2000, through a
board resolution, petitioner allegedly authorized Reyes to institute, proceed, pursue and continue
with whatever criminal or civil action against Mateo Elejorde, or such person to whom she may
have delivered or entrusted the money she had received in trust from the firm, for the purpose of
recovering such money. Thus, Reyes filed a complaint for the crime of estafa against Mateo
Elejorde before the City Prosecutor’s Office of Makati City. Thereafter, an Information for the
crime of Estafa under Article 315, par. 1 (b) of the Revised Penal Code (RPC) was filed against
Reyes and raffled before the RTC Makati City. Undeterred, Reyes filed a petition for review
before the Department of Justice (DOJ), but it was dismissed by the Secretary of Justice.
Aggrieved, Reyes filed a motion for reconsideration, and in a Resolution on the said motion was
granted.

Petitioner filed a motion for reconsideration, but was denied by the Secretary of Justice.
Eventually, petitioner filed a petition for certiorari under Rule 65 of the Rules of Court with the
CA. The latter, however, affirmed the questioned Resolutions of the Secretary of Justice. Hence,
the petition for review to the Supreme Court.

ISSUE/S: Whether of not the CA erred in not finding that the element of misappropriation was
not sufficiently established in the case but instead concurred with the decision of the DOJ
Secretary;

HELD: It is not disputed that decisions or resolutions of prosecutors are subject to appeal to the
Secretary of Justice who, under the Revised Administrative Code,9 exercises the power of direct

50
ERDIE E. AMBROCIO Remedial Law Review 1
2014-0308 Criminal Procedure – Case Digests

control and supervision over said prosecutors; and who may thus affirm, nullify, reverse or
modify their rulings. Review as an act of supervision and control by the justice secretary over the
fiscals and prosecutors finds basis in the doctrine of exhaustion of administrative remedies which
holds that mistakes, abuses or negligence committed in the initial steps of an administrative
activity or by an administrative agency should be corrected by higher administrative authorities,
and not directly by courts.

In the present case, after review and reconsideration, the Secretary of Justice reversed the
investigating prosecutor’s finding of probable cause that all the elements of the crime of estafa
are present. Estafa, under Article 315 (1) (b) of the Revised
Penal Code, is committed by―

ART. 315. Swindling (estafa).― Any person who shall defraud another by any of the means
mentioned herein below:

xxxx

1. With unfaithfulness or abuse of confidence, namely:

(a) x x x

(b) By misappropriating or converting, to the prejudice of another, money, goods, or any other
personal property received by the offender in trust or on commission, or for administration, or
under any other obligation involving the duty to make delivery of or to return the same, even
though such obligation be totally or partially guaranteed by a bond; or by denying having
received such money, goods, or other property; x x x

In reversing the finding of probable cause that the crime of estafa has been committed, the
Secretary of Justice reasoned out that, [the] theory of conversion or misappropriation is difficult
to sustain and that under the crime of estafa with grave abuse of confidence, the presumption is
that the thing has been devoted to a purpose or is different from that for which it was intended
but did not take place in this case. The CA, in sustaining the questioned resolutions of the
Secretary of Justice, ruled that the element of misappropriation or conversion is wanting. It
further ratiocinated that the demand for the return of the thing delivered in trust and the failure of
the accused to account for it, are circumstantial evidence of misappropriation, however, the said
presumption is rebuttable and if the accused is able to satisfactorily explain his failure to produce
the thing delivered in trust, he may not be held liable for estafa.

It must be remembered that the finding of probable cause was made after conducting a
preliminary investigation. A preliminary investigation constitutes a realistic judicial appraisal of
the merits of a case. Its purpose is to determine whether (a) a crime has been committed; and (b)
whether there is a probable cause to believe that the accused is guilty thereof.

This Court need not overemphasize that in a preliminary investigation, the public prosecutor
merely determines whether there is probable cause or sufficient ground to engender a well-

51
ERDIE E. AMBROCIO Remedial Law Review 1
2014-0308 Criminal Procedure – Case Digests

founded belief that a crime has been committed, and that the respondent is probably guilty
thereof and should be held for trial. It does not call for the application of rules and standards of
proof that a judgment of conviction requires after trial on the merits. The complainant need not
present at this stage proof beyond reasonable doubt. A preliminary investigation does not require
a full and exhaustive presentation of the parties’ evidence. Precisely, there is a trial to allow the
reception of evidence for both parties to substantiate their respective claims.

A review of the records would show that the investigating prosecutor was correct in finding the
existence of all the elements of the crime of estafa. Reyes did not dispute that she received in
trust the amount of P23,423,327.50 from petitioner as proven by the checks and vouchers to be
used in purchasing the parcels of land. Petitioner wrote a demand letter for Reyes to return the
same amount but was not heeded. Hence, the failure of Reyes to deliver the titles or to return the
entrusted money, despite demand and the duty to do so, constituted prima facie evidence of
misappropriation.

To reiterate, probable cause has been defined as the existence of such facts and circumstances as
would excite the belief in a reasonable mind, acting on the facts within the knowledge of the
prosecutor, that the person charged was guilty of the crime for which he was prosecuted.
Probable cause is a reasonable ground of presumption that a matter is, or may be, well founded
on such a state of facts in the mind of the prosecutor as would lead a person of ordinary caution
and prudence to believe, or entertain an honest or strong suspicion, that a thing is so. The term
does not mean “actual or positive cause” nor does it import absolute certainty. It is merely based
on opinion and reasonable belief. Thus, a finding of probable cause does not require an inquiry
into whether there is sufficient evidence to procure a conviction. It is enough that it is believed
that the act or omission complained of constitutes the offense charged.

52
ERDIE E. AMBROCIO Remedial Law Review 1
2014-0308 Criminal Procedure – Case Digests

26. ABANADO VS BAYONA


677 SCRA 595

FACTS: The case sprang from Criminal Case entitled People of the Philippines v. Cresencio
Palo, Sr. City Prosecutor Abanado filed the Information2 in the MTCC, Bacolod, which was
eventually raffled to Judge Bayona.

In connection with the issuance of a warrant of arrest against accused Palo, Judge Bayona issued
an order directing complainant Abanado to present:
1. Copy of the Memorandum of Preliminary Investigation,
2. Resolution of the Investigating Prosecutor Dennis Jarder,
3. Memorandum of the transfer of case assignment from designated Investigating
Prosecutor to the City Prosecutor, and
4. Exhibit to the Court, to enable his court to evaluate and determine the existence of
probable cause.

As to item 3, complainant stated that there was no memorandum of transfer of the case from
Investigating Prosecutor Jarder to him. Judge Bayona didn’t take the explanation stating that the
Jarder Resolution dismissing the complaint was part and parcel of the official records of the case
and must form part of the records of the preliminary investigation. Because there was a conflict
between Jarder’s and complainant’s resolutions, those documents were necessary in the
evaluation and appreciation of the evidence to establish probable cause for the issuance of a
warrant of arrest against Palo. He ordered complainant to complete the records of the case by
producing the Jarder’s Resolution. The Office of the City Prosecutor again said that it is
impossible to submit the same, the Resolution was no longer part of the records of the case as it
was disapproved by complainant.

Judge Bayona did not accept the explanations made by the Office of the City Prosecutor. He
required complainant to explain why he should not be cited for contempt. Complainant requested
for a ten-day extension to comply with it but was denied. He also ordered the Clerk of Court to
issue a subpoena duces tecum ad testificandum to Jarder directing him to testify on the existence
of his resolution dismissing the case against Palo and to Office of the City Prosecutor’s Records
Officer Myrna Vañegas to bring the entire record of the preliminary investigation of the Palo
case. Complainant then filed an inhibition against the judge and a certiorari with a prayer for the
issuance of a temporary restraining order (TRO) to restrain respondent from proceeding with the
hearing of the contempt proceedings. Complainant’s prayer for a TRO was granted by Presiding
Judge Pepito Gellada of the RTC Bacolod. Judge Gellada granted the petition for certiorari
holding that when a city or provincial prosecutor reverses the investigating assisting city or
provincial prosecutor, the resolution finding probable cause replaces the recommendation of the
investigating prosecutor recommending the dismissal of the case. The result would be that the
resolution of dismissal no longer forms an integral part of the records of the case. It is no longer
required that the complaint or entire records of the case during the preliminary investigation be
submitted to and be examined by the judge.

The rationale behind this practice is that the rules do not intend to unduly burden trial judges by

53
ERDIE E. AMBROCIO Remedial Law Review 1
2014-0308 Criminal Procedure – Case Digests

requiring them to go over the complete records of the cases all the time for the purpose of
determining probable cause for the sole purpose of issuing a warrant of arrest against the
accused. What is required is that the judge must have sufficient supporting documents upon
which to make his independent judgment or at least, upon which to verify the findings of the
prosecutor as to the existence of probable cause.

Complainant executed an administrative complaint and the same was received by the Office of
the Court Administrator (OCA). He alleged that Judge Bayona was guilty of gross ignorance of
the law or procedure and gross misconduct. He asserted that respondent unduly burdened himself
by obsessing over the production of the records of the preliminary investigation, especially
Jarder’s Resolution. Judge Bayona, in his comment with Counter-Complaint for Disbarment of
Prosecutor Abanado, reiterated the importance of the Jarder’s Resolution in deciding whether to
issue a warrant of arrest. The OCA submitted its report and recommendation noting Judge
Gellada’s Order which held that the resolution of the city or provincial prosecutor finding
probable cause replaces the recommendation of the investigating prosecutor. In such case, the
resolution recommending the dismissal is superseded, and no longer forms an integral part of the
records of the case and it need not be annexed to the information filed in court.

ISSUE/S: Whether or not the conduct of a preliminary investigation is an executive function.

HELD: Yes, the conduct of a preliminary investigation is primarily an executive function. The
courts must consider the rules of procedure of the Department of Justice in conducting
preliminary investigations whenever the actions of a public prosecutor is put in question. The
Department of Justic-National Prosecution Service (DOJ-NPS) Manual states that the resolution
of the investigating prosecutor should be attached to the information only as far as practicable.
Such attachment is not mandatory or required under the rules.

54
ERDIE E. AMBROCIO Remedial Law Review 1
2014-0308 Criminal Procedure – Case Digests

27. HEIRS OF NESTOR TRIA VS OBIAS


635 SCRA 91

FACTS: On May 22, 1998, at around 10:00 o’clock in the morning at the Pili Airport in
Camarines Sur, Engr. Nestor Tria, Regional Director of the Department of Public Works and
Highways (DPWH), Region V and concurrently Officer-In-Charge of the 2nd Engineering
District of Camarines Sur, was shot by a gunman while waiting to board his flight to Manila. He
was brought to a hospital but died the following day from the lone gunshot wound on his nape.
Subsequently, the incident was investigated by the NBI.

On July 31, 1998, NBI Regional Director Alejandro R. Tenerife, Chairman of Task Force Tria,
recommended to the Provincial Prosecutor of Camarines Sur the indictment of Roberto Aclan,
Juanito, Ona and Atty. Epifania "Fanny" Gonzales- Obias, for the murder of Engr. Tria. On the
basis of statements given by 26 individuals, autopsy and ballistic examination reports, and
relevant documents gathered the NBI found that: ACLAN and ONA had been conducting an
almost daily stakeout, for about two weeks prior to the incident, at Dir. TRIA’s office. They
would observe TRIA’s arrival and departure from office and would even ask the security guard
on duty if TRIA has already arrive or left the office.

Around 8:00 o’clock in the morning of May 22, 1998, ACLAN and ONA were spotted in their
usual places at the DPWH Office. Shortly after, Administrative Officer JOSE PECUNDO
announced to those who had some documents for signature of Director TRIA to proceed to Pili
Airport where TRIA would sign them before leaving for Manila. Upon hearing this, ACLAN and
ONA left hurriedly on board a red motorcycle. Shortly after 10:00 a.m. on that day, Director
TRIA arrived at the Airport. After signing some documents at the parking lot he proceeded
towards the pre-departure area on the second floor of the airport building. ONA, who was
waiting on the stairway, immediately followed TRIA as the latter was going up the stairs. As
TRIA was approaching the pre-departure area he was met by Atty. [E]PIFANIA OBIAS who
shook his hands and started conversing with him. It was at this juncture that a gunshot rang out
and TRIA dropped like a log on the floor, bleeding profusely from a gunshot wound at the back
of his head. Atty. EPIFANIA OBIAS, on the other hand, admitted that she was with ACLAN in
the early morning of May 22, 1998; that at about 7:00 a.m. on that day she went to the residence
of Director TRIA at Liboton, Naga City, had a brief talk with the latter and left immediately after
agreeing to meet at the airport later on. She also volunteered the information that ROBERTO
ACLAN was not the gunman who had fired the fatal shot at Director TRIA. She was also the last
person seen talking with Director TRIA when the latter was gunned down. A practicing lawyer,
Atty. OBIAS also engages herself in real estate business on the side.

In 1997 she had brokered a sale of real estate between and among spouses JEREMIAS, as
Vendors, and Spouses NESTOR and PURA TRIA, as Vendees, over a land in Balatas, Naga
City. It was Atty. OBIAS who received, for and in behalf of the vendors, the full payment of
P2.8 Million of the sale but the latter deliberately avoided the TRIA family and, despite verbal
and written demands, she failed and refused, as she still fails and refuses, to fulfill her legal
obligation to the TRIA family.

55
ERDIE E. AMBROCIO Remedial Law Review 1
2014-0308 Criminal Procedure – Case Digests

During the preliminary investigation conducted by the Office of the Provincial Prosecutor,
respondent filed her Counter- Affidavit denying that she was in anyway involved with the killing
of Engr. Tria and denied most of the allegations made by the NBI. On July 2, 1999, the Office of
the Provincial Prosecutor of Camarines Sur issued a resolution8 directing the filing of an
information for murder against Aclan and Ona but dismissing the case for insufficiency of
evidence as against herein respondent, Atty. Epifania Obias. This was, however, modified on
January 25, 2000 by then Justice Secretary Serafin Cuevas directing the Provincial Prosecutor to
include respondent in the information for murder filed against Aclan and Ona.

Respondent along with Aclan and Ona filed a motion for reconsideration of the DOJ’s January
25, 2000 resolution. In the meantime, the information charging Aclan and Ona has already been
filed with the Regional Trial Court (RTC) of Pili, Camarines Sur. Upon request however, the
venue was transferred to the RTC Quezon City by resolution of this Court in A.M. No. 00-3145-
RTC.16 After so much petition and appeal, the DOJ eventually directed the Provincial
Prosecutor to forward the records of the case to the Office of the President in compliance. In his
Order dated March 24, 2004, Presidential Assistant Manuel C. Domingo granted respondent’s
motion for reconsideration and reversed the DOJ resolutions. The OP concluded there was no
interlocking circumstantial evidence of respondent’s acts before, during and after the killing of
Engr. Tria that would establish conspiracy among Aclan, Ona and respondent to commit the
crime. Accordingly, the case against respondent was dismissed for insufficiency of evidence.
Petitioners filed a series of motions, which reached the CA, who denied their petition. Hence,
this appeal.

ISSUE/S:
1. Whether non-referral by the OP to the DOJ of the appeal or motion for reconsideration filed
by the respondent had deprived them of the opportunity to confront and cross-examine the
witnesses on those affidavits belatedly submitted by the respondent. –No.
2. Whether the CA gravely abused its discretion in affirming the OP’s reversal of the ruling of
the Secretary of Justice. –Yes.

HELD: Under the procedure for preliminary investigation provided in Section 3, Rule 112 of
the Revised Rules of Criminal Procedure, as amended, in case the investigating prosecutor
conducts a hearing where there are facts and issues to be clarified from a party or witness, "[t]he
parties can be present at the hearing but without the right to examine or cross-examine. They
may, however, submit to the investigating officer questions which may be asked to the party or
witness concerned." Hence, the non-referral by the OP to the DOJ of the motion for
reconsideration of respondent, in the exercise of its discretion, did not violate petitioners’ right to
due process. The findings of the prosecutor with respect to the existence or non-existence of
probable cause is subject to the power of review by the DOJ.

Indeed, the Secretary of Justice may reverse or modify the resolution of the prosecutor, after
which he shall direct the prosecutor concerned either to file the corresponding information
without conducting another preliminary investigation, or to dismiss or move for dismissal of the
complaint or information with notice to the parties. In reversing the DOJ’s finding of probable
cause, the OP found merit in the argument of the respondent that the DOJ’s finding that she was

56
ERDIE E. AMBROCIO Remedial Law Review 1
2014-0308 Criminal Procedure – Case Digests

with Aclan when she went to the residence of Engr. Tria early in the morning of May 22, 1998,
was not sufficiently established. The OP gave more weight to the affidavit of Calayag stating that
Aclan was not around when they and respondent, among other visitors, were at Engr. Tria’s
house at that time -- than that account given by SA Eduarte, which was uncorroborated.

As to the double sale allegedly committed by the respondent from which the latter’s strong
motive to liquidate Engr. Tria was inferred, the OP found this as a mere expression of opinion by
the investigators considering that Engr. Tria’s widow, Mrs. Pura Tria, categorically admitted her
knowledge of the said transaction. Neither was the OP persuaded by the NBI’s "kiss of death"
theory since it is but a customary way of greeting a friend to shake hands and hence it cannot
imply that respondent utilized this as a signal or identification for the gunman to shoot Engr.
Tria. Respondent’s alleged indifference immediately after Engr. Tria was gunned down while
conversing with her, was also negated by the affidavit of an employee of Philippine Air Lines
based at the Pili Airport, stating that right after the incident took place he saw respondent in the
radio room in shock and was being given water by another person.

Considering the totality of evidence, the OP was convinced there was nothing suspicious or
abnormal in respondent’s behavior before, during and after the fatal shooting of Engr. Tria as to
engender a well-founded belief of her complicity with the killing of Engr. Tria. Petitioners,
however, maintain that the records are replete with abundant proof of respondent’s complicity in
the murder of Engr. Tria. Probable cause is defined as the existence of such facts and
circumstances as would excite the belief in a reasonable mind, acting on the facts within the
knowledge of the prosecutor, that the person charged was guilty of the crime for which he was
prosecuted. It is a reasonable ground of presumption that a matter is, or may be, well-founded,
such a state of facts in the mind of the prosecutor as would lead a person of ordinary caution and
prudence to believe, or entertain an honest or strong suspicion, that a thing is so.
The term does not mean "actual and positive cause" nor does it import absolute certainty. It is
merely based on opinion and reasonable belief. A finding of probable cause merely binds over
the suspect to stand trial; it is not a pronouncement of guilt.

On the other hand, conspiracy exists when two or more persons come to an agreement
concerning the commission of a felony and decide to commit it. Direct proof of previous
agreement to commit a crime is not necessary. Conspiracy may be shown through circumstantial
evidence, deduced from the mode and manner in which the offense was perpetrated, or inferred
from the acts of the accused themselves when such lead to a joint purpose and design, concerted
action, and community of interest.

We reverse the OP’s ruling that the totality of evidence failed to establish a prima facie case
against the respondent as a conspirator in the killing of Engr. Tria. To begin with, whether or not
respondent actually conspired with Aclan and Ona need not be fully resolved during the
preliminary investigation. The absence or presence of conspiracy is factual in nature and
involves evidentiary matters. The same is better left ventilated before the trial court during trial,
where the parties can adduce evidence to prove or disprove its presence.
Preliminary investigation is executive in character. It does not contemplate a judicial function. It
is essentially an inquisitorial proceeding, and often, the only means of ascertaining who may be

57
ERDIE E. AMBROCIO Remedial Law Review 1
2014-0308 Criminal Procedure – Case Digests

reasonably charged with a crime. Prosecutors control and direct the prosecution of criminal
offenses, including the conduct of preliminary investigation, subject to review by the Secretary
of Justice. The duty of the Court in appropriate cases is merely to determine whether the
executive determination was done without or in excess of jurisdiction or with grave abuse of
discretion. Resolutions of the Secretary of Justice are not subject to review unless made with
grave abuse.

After a careful evaluation of the entire evidence on record, we find no such grave abuse when the
Secretary of Justice found probable cause to charge the respondent with murder in conspiracy
with Aclan and Ona. The following facts and circumstances established during preliminary
investigation were sufficient basis to incite reasonable belief in respondent’s guilt: (a) Motive -
respondent had credible reason to have Engr. Tria killed because of the impending criminal
prosecution for estafa from her double sale of his lot prior to his death, judging from the strong
interest of Engr. Tria’s family to run after said property and/or proceeds of the second sale to a
third party; (b) Access - respondent was close to Engr. Tria’s family and familiar with his work
schedule, daily routine and other transactions which could facilitate in the commission of the
crime eventually carried out by a hired gunmen, one of whom (Aclan) she and her father
categorically admitted being in her company while she visited Engr. Tria hours before the latter
was fatally shot at the airport; (c) Suspicious Behavior -- respondent while declaring such close
personal relationship with Engr. Tria and even his family, failed to give any satisfactory
explanation why she reacted indifferently to the violent killing of her friend while they conversed
and shook hands at the airport.

Indeed, a relative or a friend would not just stand by and walk away from the place as if nothing
happened, as what she did, nor refuse to volunteer information that would help the authorities
investigating the crime, considering that she is a vital eyewitness. Not even a call for help to the
people to bring her friend quickly to the hospital. She would not even dare go near Engr. Tria’s
body to check if the latter was still alive. All the foregoing circumstances, in our mind, and from
the point of view of an ordinary person, lead to a reasonable inference of respondent’s probable
participation in the well-planned assassination of Engr. Tria.

We therefore hold that the OP in reversing the DOJ Secretary’s ruling, and the CA in affirming
the same, both committed grave abuse of discretion. Clearly, the OP and CA arbitrarily
disregarded facts on record which established probable cause against the respondent.

58
ERDIE E. AMBROCIO Remedial Law Review 1
2014-0308 Criminal Procedure – Case Digests

28. UY VS JAVELLANA
680 SCRA 13

FACTS: This administrative case arose from a verified complaint for "gross ignorance of the
law and procedures, gross incompetence, neglect of duty, conduct improper and unbecoming of a
judge, grave misconduct and others," filed by Public Attorneys Gerlie M. Uy (Uy) and Ma.
Consolacion T. Bascug (Bascug) of the (PAO), La Carlotta District, against Presiding Judge
Javellana of the MeTC, La Castellana, Negros Occidental. Public Attorneys Uy and Bascug
alleged the following in their complaint: Judge Javellana was grossly ignorant of the Revised
Rule on Summary Procedure. Public Attorneys Uy and Bascug cited several occasions as
examples: In Crim. Case No. 04-097, entitled People v. Cornelio, for Malicious Mischief, Judge
Javellana issued a warrant of arrest after the filing of said case despite Section 16 of the Revised
Rule on Summary Procedure; Crim. Case No. 04-075, entitled People v. Celeste, et al., for
Trespass to Dwelling, Judge Javellana did not grant the motion to dismiss for non-compliance
with the Lupon requirement under Sections 18 and 19(a) of the Revised Rule on Summary
Procedure, insisting that said motion was a prohibited pleading; Also in People v. Celeste, et al.,
Judge Javellana refused to dismiss outright the complaint even when the same was patently
without basis or merit, as the affidavits of therein complainant and her witnesses were all hearsay
evidence; and Crim. Case No. 02-056, entitled People v. Lopez, et al., for Malicious Mischief,
Judge Javellana did not apply the Revised Rule on Summary Procedure and, instead, conducted a
preliminary examination and preliminary investigation in accordance with the Revised Rules of
Criminal Procedure, then set the case for arraignment and pre-trial, despite confirming that
therein complainant and her witnesses had no personal knowledge of the material facts alleged in
their affidavits, which should have been a ground for dismissal of said case. Judge Javellana
violated Section 6(b), Rule 112 of the Revised Rules of Criminal Procedure and issued warrants
of arrest without propounding searching questions to the complainants and their witnesses to
determine the necessity of placing the accused under immediate custody.

As a result, Judge Javellana issued warrants of arrest even when the accused had already
voluntarily surrendered or when a warrantless arrest had been effected. Judge Javellana failed to
observe the constitutional rights of the accused as stated in Section 12(1), Article III of the
Constitution. Judge Javellana set Crim. Case No. 03-097, entitled People v. Bautista, for
preliminary investigation even when the accused had no counsel, and proceeded with said
investigation without informing the accused of his rights to remain silent and to have a counsel
Judge Javellana stressed that the charges against him were baseless and malicious; and the acts
being complained of involved judicial discretion and, thus, judicial in nature and not the proper
subject of an administrative complaint. Consequently, Judge Javellana sought the dismissal of
the instant complaint against him. The Office of the Court Administrator (OCA), in its report,
found Judge Javellana liable for gross ignorance of the law or procedure when he did not apply
the Revised Rule on Summary Procedure in cases appropriately covered by said Rule.

ISSUE/S: Whether or not Judge Javellana was grossly ignorant of the Revised Rule on
Summary Procedure.

HELD: Judge Javellana committed a blatant error in denying the Motion to Dismiss filed by the

59
ERDIE E. AMBROCIO Remedial Law Review 1
2014-0308 Criminal Procedure – Case Digests

accused in People v. Celeste, et al. and in insisting that said Motion was a prohibited pleading,
even though the case was never previously referred to the Lupong Tagapamayapa as required by
Sections 18 and 19(a) ofthe Revised Rule on Summary Procedure. A case which has not been
previously referred to the Lupong Tagapamayapa shall be dismissed without prejudice. A motion
to dismiss on the ground of failure to comply with the Lupon requirement is an exception to the
pleadings prohibited by the Revised Rule on Summary Procedure. Given the express provisions
of the Revised Rule on Summary Procedure, we find irrelevant Judge Javellana’s argument that
referral to the Lupon is not a jurisdictional requirement. The following facts are undisputed:
People v. Celeste, et al. were not referred to the Lupon, and the accused filed a Motion to
Dismiss based on this ground. Judge Javellana should have allowed and granted the Motion to
Dismiss (albeit without prejudice) filed by the accused in People v. Celeste, et al. (hindi ko sure)
Judge Javellana did not provide any reason as to why he needed to conduct a preliminary
investigation in People v. Lopez, et al. Judge Javellana cannot be allowed to arbitrarily conduct
proceedings beyond those specifically laid down by the Revised Rule on Summary Procedure,
thereby lengthening or delaying the resolution of the case, and defeating the express purpose of
said Rule. Without any showing that the accused in People v. Cornelio and People v. Lopez, et
al. were charged with the special cases of malicious mischief particularly described in Article
328 of the Revised Penal Code the appropriate penalty for the accused would be arresto mayor in
its medium and maximum periods which under Article 329(a) of the Revised Penal Code, would
be imprisonment for two (2) months and one (1) day to six (6) months. Clearly, these two cases
should be governed by the Revised Rule on Summary Procedure.

60
ERDIE E. AMBROCIO Remedial Law Review 1
2014-0308 Criminal Procedure – Case Digests

29. PEOPLE VS VALENCIA


214 SCRA 89

FACTS: Accused-appellant Alejandro Valencia appeals the Decision of the Regional Trial
Court of Manila, in 2 Criminal Cases convicting him of Homicide with the use of an unlicensed
firearm and Less Serious Physical Injuries.

Arlyn Barredo-Jimenez, her two children, Annabelle and Samuel, Jr., aged five and three,
respectively, and her mother, are residents of 2008 F. Muñoz St., Paco, Manila. At about 9:00
p.m. of March 19, 1989, as she was about to eat supper, she noticed appellant standing five steps
away from the open door of her house and holding a sumpak, a homemade shotgun. Seized with
fear, she closed the door. After a few moments, she heard a burst of gunfire. This was followed
by cries of pain from her children inside the house. Seeing her children bloodied, she
immediately went outside and shouted for help. As she did so, she saw appellant running away,
carrying the sumpak. Two neighbors assisted Jimenez in bringing the injured children to the
Philippine General Hospital. That same evening, Patrolman Renato Marquez, a homicide
investigator, interviewed Jimenez at the hospital about the shooting incident. Since she was still
experiencing shock over the incident Jimenez forgot to mention the name of appellant as the one
who shot her children. Acting on the report of a barangay tanod, Patrolmen Roberto Cajiles,
Romeo de la Peña and Carlos Castañeda, assigned at the Ong Detachment, Police Station No. 5,
conducted an investigation of the shooting incident in the house of Jimenez. At the time, Jimenez
and her injured children were already in the hospital. Nevertheless, Pat. Cajiles was able to
interview the mother of Jimenez, the barangay captain, a certain Josie, and appellant’s brother,
Rolando, who all mentioned appellant as the gunwielder. Moreover, the policemen discovered
the presence of six pellet holes and one big hole with the size of the circumference of a shotgun
bullet on the door of the house of Jimenez. Three pellets were also found at the crime scene.

Early next morning, the three policemen were led by Rolando Valencia to the residence of Sonia
Castillo, his aunt, where he believed appellant was sleeping. The police apprehended appellant
there and took him to the Ong Detachment for initial investigation. He was indorsed to the police
headquarters for further investigation in the evening of March 22, 1989. At 12:20 a.m. of the
following day, one of the injured children, Annabelle, died as a result of the gunshot wounds she
suffered. The other child, Samuel Jr., who was shot in the right forearm, was discharged from the
hospital one week after the incident, but needed 2 more weeks for healing. On March 26, 1989,
Arlyn Jimenez executed a sworn statement wherein she identified appellant as the culprit. On
March 30, 1989, a certain Ramon Bacnotan executed a sworn statement and turned over to the
police the sumpak allegedly used by appellant in the shooting of the two children. 2 Criminal
Cases were filed against Valencia, for Homicide with the use of an unlicensed firearm and Less
Serious Physical Injuries. When arraigned, the accused-appellant pleaded "Not Guilty." Trial
then proceeded resulting in accused-appellant’s conviction.

ISSUE/S: Whether or not the finding of the court of Accused-appellant’s guilt beyond
reasonable doubt is correct in spite of the fact that there was allegedly no preliminary
investigation, and that no sufficient evidence exists proving his guilt.

61
ERDIE E. AMBROCIO Remedial Law Review 1
2014-0308 Criminal Procedure – Case Digests

HELD: Yes. A person who is lawfully arrested, without a warrant pursuant to paragraph 1(b),
Section 5, Rule 113, Rules of Court should be delivered to the nearest police station and
proceeded against in accordance with Rule 112, Section 7.

Under said Section 7, Rule 112, 25 the prosecuting officer can file the Information in court
without a preliminary investigation, which was done in the accused-appellant’s case. Since the
records do not show whether the accused-appellant asked for a preliminary investigation after the
case had been filed in court, as in fact, the accused-appellant signified his readiness to be
arraigned, the Court can only conclude that he waived his right to have a preliminary
investigation, when he did, in fact, pleaded "Not Guilty" upon his arraignment.

62
ERDIE E. AMBROCIO Remedial Law Review 1
2014-0308 Criminal Procedure – Case Digests

30. PCGG VS NAVARRO-GUTIERREZ


773 SCRA

FACTS: PCGG filed against former officers/directors of the Development Bank of the
Philippines (DBP), namely, Ferry, Tengco, Zosa, Zalamea, Castell, and Sison, as well as former
officers/stockholders of National Galleon Shipping Corporation (Galleon), namely, Cuenca,
Tinio, and Roque charging them of violating Sections 3 (e) and (g) of RA 3019.

PCGG alleged that on October 8, 1992, then President Fidel V. Ramos (President Ramos) issued
Administrative Order No. 13, creating the Presidential Ad Hoc Fact-Finding Committee on
Behest Loans (Ad Hoc Committee) in order to identify various anomalous loans entered into by
the Philippine Government in the past. Thereafter, the Ad Hoc Committee, with the assistance of
a Technical Working Group (TWG) examined and studied documents relative to loan accounts
extended by GFIs to various corporations during the regime of the late President Ferdinand E.
Marcos (President Marcos) -one of which is the loan account granted by the DBP to Galleon.
TWG found anomalies after examining the loans of Galleon. PCGG then files a case against the
individual officers. Only Roque, Zalamea, Tengco, and Castell filed their counter-affidavits. In
their defense, Roque stated that he was only a minor stockholder and in no position to influence
such loan. Zamalea was only chairman of DBP after the transaction. Tenco argued that the
charges already prescribed, and Castell argued that his job is only supervision of employees.

Ombudsman found no probable cause against them hence the case was dismissed on the basis
that pieces of evidence attached to the case records were not sufficient to establish probable
cause against the individual respondents, considering that the documents presented by the PCGG
consisted mostly of executive summaries and technical reports, which are hearsay, self-serving,
and of little probative value. PCGG move for reconsideration but was also denied.

ISSUE/S: Whether or not there was a grave abuse of discretion on the part of the Ombudsman
in finding no probable cause to indict the respondents.

HELD: Yes. It must be stressed that the Court has consistently refrained from interfering with
the discretion of the Ombudsman to determine the existence of probable cause and to decide
whether or not an Information should be filed. Nonetheless, the Court is not precluded from
reviewing the Ombudsman's action when there is a charge of grave abuse of discretion. Grave
abuse of discretion implies a capricious and whimsical exercise of judgment tantamount to lack
of jurisdiction. The Ombudsman's exercise of power must have been done in an arbitrary or
despotic manner which must be so patent and gross as to amount to an evasion of a positive duty
or a virtual refusal to perform the duty enjoined or to act at all in contemplation of law. The
Court's pronouncement in Ciron v. Gutierrez is instructive on this matter, to wit:

xxx this Court's consistent policy has been to maintain noninterference in the determination of
the Ombudsman of the existence of probable cause, provided there is no grave abuse in the
exercise of such discretion. This observed policy is based not only on respect for the
investigatory and prosecutory powers granted by the Constitution to the Office of the
Ombudsman but upon practicality as well. Otherwise, the functions of the Court will be seriously

63
ERDIE E. AMBROCIO Remedial Law Review 1
2014-0308 Criminal Procedure – Case Digests

hampered by innumerable petitions assailing the dismissal of investigatory proceedings


conducted by the Office of the Ombudsman with regard to complaints filed before it, in much the
same way that the courts would be extremely swamped with cases if they could be compelled to
review the exercise of discretion on the part of the fiscals or prosecuting attorneys each time they
decide to file an information in court or dismiss a complaint by a private complainant.

In this regard, it is worthy to note that the conduct of preliminary investigation proceedings -
whether by the Ombudsman or by a public prosecutor - is geared only to determine whether or
not probable cause exists to hold an accused-respondent for trial for the supposed crime that he
committed. In Fenequito v. Vergara, Jr., the Court defined probable cause and the parameters in
finding the existence thereof in the following manner, to wit:

Probable cause, for the purpose of filing a criminal information, has been defined as such facts as
are sufficient to engender a well-founded belief that a crime has been committed and that
respondent is probably guilty thereof. The term does not mean "actual or positive cause" nor
does it import absolute certainty. It is merely based on opinion and reasonable belief. Probable
cause does not require an inquiry whether there is sufficient evidence to procure a conviction. It
is enough that it is believed that the act or omission complained of constitutes the offense
charged. A finding of probable cause needs only to rest on evidence showing that, more likely
than not, a crime has been committed by the suspects. It need not be based on clear and
convincing evidence of guilt, not on evidence establishing guilt beyond reasonable doubt, and
definitely not on evidence establishing absolute certainty of guilt. In determining probable cause,
the average man weighs facts and circumstances without resorting to the calibrations of the rules
of evidence of which he has no technical knowledge. He relies on common sense. What is
determined is whether there is sufficient ground to engender a well-founded belief that a crime
has been committed, and that the accused is probably guilty thereof and should be held for trial.
It does not require an inquiry as to whether there is sufficient evidence to secure a conviction.

Preliminary investigation is merely an inquisitorial mode of discovering whether or not there is


reasonable basis to believe that a crime has been committed and that the person charged should
be held responsible for it. Being merely based on opinion and belief, a finding of probable cause
does not require an inquiry as to whether there is sufficient evidence to secure a conviction. "[A
preliminary investigation] is not the occasion for the full and exhaustive display of [the
prosecution's] evidence. The presence and absence of the elements of the crime is evidentiary in
nature and is a matter of defense that may be passed upon after a full-blown trial on the merits."
Hence, "the validity and merits of a party's defense or accusation, as well as the admissibility of
testimonies and evidence, are better ventilated during trial proper than at the preliminary
investigation level."

Guided by the foregoing considerations, the Court finds that the Ombudsman gravely abused its
discretion in dismissing the criminal complaint against individual respondents for lack of
probable cause, as will be explained hereunder. As already stated, individual respondents were
accused of violating Section 3 (e) of RA 3019, the elements of which are as follows: (a) that the
accused must be a public officer discharging administrative, judicial, or official functions (or a
private individual acting in conspiracy with such public officers); (b) that he acted with manifest

64
ERDIE E. AMBROCIO Remedial Law Review 1
2014-0308 Criminal Procedure – Case Digests

partiality, evident bad faith, or inexcusable negligence; and (c) that his action caused any undue
injury to any party, including the government, or giving any private party unwarranted benefits,
advantage, or preference in the discharge of his functions. In the same vein, they were likewise
charged with violation of Section 3 (g) of the same law, which has the following elements: (a)
that the accused is a public officer; (b) that he entered into a contract or transaction on behalf of
the government; and (c) that such contract or transaction is grossly and manifestly
disadvantageous to the government. Notably, private individuals may also be charged with
violation of Section 3 (g) of RA 3019 if they conspired with public officers.

Finally, it was error for the Ombudsman to simply discredit the TWG's findings contained in the
Executive Summary which were adopted by the Ad Hoc Committee for being hearsay, self-
serving, and of little probative value. It is noteworthy to point out that owing to the initiatory
nature of preliminary investigations, the technical rules of evidence should not be applied in the
course of its proceedings. In the recent case of Estrada v. Ombudsman, the Court declared that
hearsay evidence is admissible in determining probable cause in preliminary investigations
because such investigation is merely preliminary, and does not finally adjudicate rights and
obligations of parties. Citing a case decided by the Supreme Court of the United States, it was
held that probable cause can be established with hearsay evidence, as long as there is substantial
basis for crediting the hearsay, viz.: Justice Brion's pronouncement in Unilever that "the
determination of probable cause does not depend on the validity or merits of a party's accusation
or defense or on the admissibility or veracity of testimonies presented" correctly recognizes the
doctrine in the United States that the determination of probable cause can rest partially, or even
entirely, on hearsay evidence, as long as the person making the hearsay statement is credible. In
United States v. Ventresca, the United States Supreme Court held: While a warrant may issue
only upon a finding of "probable cause," this Court has long held that "the term 'probable cause'
... means less than evidence which would justify condemnation," x x x and that a finding of
"probable cause" may rest upon evidence which is not legally competent in a criminal trial, x x x
As the Court stated in Brinegar v. United States x x x, "There is a large difference between two
things to be proved (guilt and probable cause), as well as between the tribunals which determine
them, and therefore a like difference in the quanta and modes of proof required to establish
them." Thus, hearsay may be the bases for issuance of the warrant "so long as there ... [is] a
substantial basis for crediting the hearsay." x xx And, in Aguilar, we recognized that "an
affidavit may be based on hearsay information and need not reflect the direct personal
observations of the affiant," so long as the magistrate is "informed of some of the underlying
circumstances" supporting the affiant's conclusions and his belief that any informant involved
"whose identity need not be disclosed..." was"credible" or his information "reliable." x x x. Thus,
probable cause can be established with hearsay evidence, as long as there is substantial basis for
crediting the hearsay. Hearsay evidence is admissible in determining probable cause in a
preliminary investigation because such investigation is merely preliminary, and does not finally
adjudicate rights and obligations of parties, x x x.

65
ERDIE E. AMBROCIO Remedial Law Review 1
2014-0308 Criminal Procedure – Case Digests

31. SARAUM VS PEOPLE


781 SCRA

FACTS: Saraum was charged with violation of Section 12, Article II (Possession of
Paraphernalia for Dangerous Drugs) of Republic Act (R.A.) No. 9165, or the Comprehensive
Dangerous Drugs Act of 2002. The accusatory portion of the Information reads:

That on or about the 17th day of August, 2006, at about 12:45 A.M., The accused
did then and there have in his possession the following: 1 = One (1) lighter, 2 =
One (1) rolled tissue paper, 3 = One (1) aluminum tin foil, which are instruments
and/or equipments fit or intended for smoking, consuming, administering,
ingesting, or introducing any dangerous drug into the body.

Saraum pleaded not guilty and was released due to his application of bail. According to the
prosecution,the police officers conducted a buy-bust operation and coordinated with the
Philippine Drug Enforcement Agency (PDEA) regarding the operation. During the operation,
"Pata" eluded arrest as he tried to run towards his shanty. Inside the house, which was divided
with a curtain as partition, the buy-bust team also saw Saraum and Peter Esperanza, who were
holding drug paraphernalia apparently in preparation to have a "shabu" pot session. They
recovered from Saraum’s possession a lighter, rolled tissue paper, and aluminum tin foil (tooter).
PO3 Larrobis confiscated the items, placed them in the plastic pack of misua wrapper, and made
initial markings ("A" for Saraum and "P" for Esperanza). At the police station, PO3 Larrobis
marked as "AIS-08-17-2006" the paraphernalia recovered from Saraum. After the case was filed,
the subject items were turned over to the property custodian of the Office of City Prosecutor.

Saraum denied the commission of the alleged offense. He testified that on the date and time in
question, he was passing by Lorega Cemetery on his way to the house of his parents-in-law when
he was held by men with firearms. They were already with "Antik" and "Pata," both of whom
were his neighbors. Believing that he had not committed anything illegal, he resisted the arrest.
He learned of the criminal charge only when he was brought to the court.

RTC ruled against Sarum.

CA sustained the RTCs decision.

ISSUE/S: Whether or not the Arrest of Saraum was valid

HELD: Yes. Considering that Saraum failed to show any arbitrariness, palpable error, or
capriciousness on the findings of fact of the trial and appellate courts, such findings deserve great
weight and are deemed conclusive and binding. Besides, a review of the records reveals that the
CA did not err in affirming his conviction.

The elements of illegal possession of equipment, instrument, apparatus and other paraphernalia
for dangerous drugs under Section 12, Article II of R.A. No. 9165 are: (1) possession or control
by the accused of any equipment, apparatus or other paraphernalia fit or intended for smoking,

66
ERDIE E. AMBROCIO Remedial Law Review 1
2014-0308 Criminal Procedure – Case Digests

consuming, administering, injecting, ingesting, or introducing any dangerous drug into the body;
and (2) such possession is not authorized by law. In this case, the prosecution has convincingly
established that Saraum was in possession of drug paraphernalia, particularly aluminum tin foil,
rolled tissue paper, and lighter, all of which were offered and admitted in evidence.

Saraum was arrested during the commission of a crime, which instance does not require a
warrant in accordance with Section 5 (a), Rule 113 of the Revised Rules on Criminal Procedure.
In arrest in flagrante delicto, the accused is apprehended at the very moment he is committing or
attempting to commit or has just committed an offense in the presence of the arresting officer. To
constitute a valid in flagrante delicto arrest, two requisites must concur: (1) the person to be
arrested must execute an overt act indicating that he has just committed, is actually committing,
or is attempting to commit a crime; and (2) such overt act is done in the presence or within the
view of the arresting officer.

Here, the Court is unconvinced with Saraum’s statement that he was not committing a crime at
the time of his arrest. PO3 Larrobis described in detail how they were able to apprehend him,
who was then holding a disposable lighter in his right hand and a tin foil and a rolled tissue paper
in his left hand, while they were in the course of arresting somebody. The case is clearly one of
hot pursuit of "Pata," who, in eluding arrest, entered the shanty where Saraum and Esperanza
were incidentally caught in possession of the illegal items. Saraum did not proffer any
satisfactory explanation with regard to his presence at the vicinity of the buy-bust operation and
his possession of the seized items that he claims to have "countless, lawful uses."

On the contrary, the prosecution witnesses have adequately explained the respective uses of the
items to prove that they were indeed drug paraphernalia. There is, thus, no necessity to make a
laboratory examination and finding as to the presence or absence of methamphetamine
hydrochloride or any illegal substances on said items since possession itself is the punishable act.

The valid warrantless arrest gave the officers the right to search the shanty for objects relating to
the crime and seize the drug paraphernalia they found. In the course of their lawful intrusion,
they inadvertently saw the various drug paraphernalia. As these items were plainly visible, the
police officers were justified in seizing them. Considering that Saraum’s arrest was legal, the
search and seizure that resulted from it were likewise lawful. The various drug paraphernalia that
the police officers found and seized in the shanty are, therefore, admissible in evidence for
having proceeded from a valid search and seizure. Since the confiscated drug paraphernalia are
the very corpus delicti of the crime charged, the Court has no choice but to sustain the judgment
of conviction. Even if We consider the arrest as invalid, Saraum is deemed to have waived any
objection thereto when he did not raise the issue before entering his plea. "The established rule is
that an accused may be estopped from assailing the legality of his arrest if he failed to move for
the quashing of the Information against him before his arraignment. Any objection involving the
arrest or the procedure in the court's acquisition of jurisdiction over the person of an accused
must be made before he enters his plea; otherwise the objection is deemed waived." In this case,
counsel for Saraum manifested its objection to the admission of the seized drug paraphernalia,
invoking illegal arrest and search, only during the formal offer of evidence by the prosecution.

67
ERDIE E. AMBROCIO Remedial Law Review 1
2014-0308 Criminal Procedure – Case Digests

32. COMERCIANTE VS PEOPLE


763 SCRA 587

FACTS: Comerciante was caught having in his possession, custody and control Two (2) heat-
sealed transparent plastic sachet (sic) each containing 0.15 gram (sic) and 0.28 gram (sic) of
white crystalline substance with a total of 0.43 grams which was found positive to the test for
Methamphetamine Hydrochloride commonly known as "shabu", a dangerous drug.

According to the prosecution, the Police were patrolling the area while on their way to visit a
friend at Private Road, Barangay Hulo, Mandaluyong City. when they spotted two (2) men - later
identified as Comerciante and a certain Erick Dasilla (Dasilla) - standing and showing "improper
and unpleasant movements," with one of them handing plastic sachets to the other. Thinking that
the sachets may contain shabu, they immediately stopped and approached Comerciante and
Dasilla.

The police officer arrested Comerciante and Dasilla, and confiscated two (2) plastic sachets
containing white crystalline substance from them. A laboratory examination later confirmed that
said sachets contained methamphetamine hydrochloride or shabu. After the prosecution rested its
case, Dasilla filed a demurrer to evidence, which was granted by the RTC, thus his acquittal.
However, due to Comerciante's failure to file his own demurrer to evidence, the RTC considered
his right to do so waived and ordered him to present his evidence.

In his defense, Comerciante averred that PO3 Calag was looking for a certain "Barok", who was
a notorious drug pusher in the area, when suddenly, he and Dasilla, who were just standing in
front of a jeepney along Private Road, were arrested and taken to a police station. There, the
police officers claimed to have confiscated illegal drugs from them and were asked money in
exchange for their release. When they failed to accede to the demand, they were brought to
another police station to undergo inquest proceedings, and thereafter, were charged with illegal
possession of dangerous drugs.

The RTC found that PO3 Calag conducted a valid warrantless arrest on Comerciante, which
yielded two (2) plastic sachets containing shabu. In this relation, the RTC opined that there was
probable cause to justify the warrantless arrest, considering that PO3 Calag saw, in plain view,
that Comerciante was carrying the said sachets when he decided to approach and apprehend the
latter. Further, the RTC found that absent any proof of intent that PO3 Calag was impelled by
any malicious motive, he must be presumed to have properly performed his duty when he
arrested Comerciante.

CA Affirmed the RTCs ruling.

ISSUE/S: Whether or not the there was a valid warrantless arrest and “stop and frisk” conducted
by the Police

HELD: No, the Court finds it highly implausible that PO3 Calag, even assuming that he has
perfect vision, would be able to identify with reasonable accuracy especially from a distance of

68
ERDIE E. AMBROCIO Remedial Law Review 1
2014-0308 Criminal Procedure – Case Digests

around 10 meters, and while aboard a motorcycle cruising at a speed of 30 kilometers per hour
miniscule amounts of white crystalline substance inside two (2) very small plastic sachets held
by Comerciante. The Court also notes that no other overt act could be properly attributed to
Comerciante as to rouse suspicion in the mind of PO3 Calag that the former had just committed,
was committing, or was about to commit a crime. Verily, the acts of standing around with a
companion and handing over something to the latter cannot in any way be considered criminal
acts. In fact, even if Comerciante and his companion were showing "improper and unpleasant
movements" as put by PO3 Calag, the same would not have been sufficient in order to effect a
lawful warrantless arrest under Section 5 (a), Rule 113 of the Revised Rules on Criminal
Procedure.31 That his reasonable suspicion bolstered by (a) the fact that he had seen his fellow
officers arrest persons in possession of shabu; and (b) his trainings and seminars on illegal drugs
when he was still assigned in the province are insufficient to create a conclusion that what he
purportedly saw in Comerciante was indeed shabu. Neither has the prosecution established that
the rigorous conditions set forth in Section 5 (b), Rule 113, have been complied with, i.e., that an
offense had in fact just been committed and the arresting officer had personal knowledge of facts
indicating that the accused had committed it. As already discussed, the factual backdrop of the
instant case failed to show that PO3 Calag had personal knowledge that a crime had been
indisputably committed by Comerciante. Verily, it is not enough that the arresting officer had
reasonable ground to believe that the accused had just committed a crime; a crime must, in fact,
have been committed first, which does not obtain in this case.

In this relation, the Court finds respondent's assertion that there was a valid "stop and frisk"
search made on Comerciante untenable. In People v. Cogaed, the Court had an opportunity to
exhaustively explain "stop and frisk" searches: "Stop and frisk" searches (sometimes referred to
as Terry searches) are necessary for law enforcement. That is, law enforcers should be given the
legal arsenal to prevent the commission of offenses. However, this should be balanced with the
need to protect the privacy of citizens in accordance with Article III, Section 2 of the
Constitution. The balance lies in the concept of "suspiciousness" present where the police officer
finds himself or herself in. This may be undoubtedly based on the experience of the police
officer. Experienced police officers have personal experience dealing with criminals and criminal
behavior. Hence, they should have the ability to discern - based on facts that they themselves
observe - whether an individual is acting in a suspicious manner. Clearly, a basic criterion would
be that the police officer, with his or her personal knowledge, must observe the facts leading to
the suspicion of an illicit act. x x x x Normally, "stop and frisk" searches do not give the law
enforcer an opportunity to confer with a judge to determine probable cause. In Posadas v. Court
of Appeals, one of the earliest cases adopting the "stop and frisk" doctrine in Philippine
jurisprudence, this court approximated the suspicious circumstances as probable cause: The
probable cause is that when the petitioner acted suspiciously and attempted to flee with the buri
bag there was a probable cause that he was concealing something illegal in the bag and it was the
right and duty of the police officers to inspect the same.

For warrantless searches, probable cause was defined as a reasonable ground of suspicion
supported by circumstances sufficiently strong in themselves to warrant a cautious man to
believe that the person accused is guilty of the offense with which he is charged. Malacat v.
Court of Appeals clarifies the requirement further. It does not have to be probable cause, but it

69
ERDIE E. AMBROCIO Remedial Law Review 1
2014-0308 Criminal Procedure – Case Digests

cannot be mere suspicion. It has to be a genuine reason to serve the purposes of the "stop and
frisk" exception: Other notable points of Terry are that while probable cause is not required to
conduct a "stop and frisk," it nevertheless holds that mere suspicion or a hunch will not validate a
"stop and frisk." A genuine reason must exist, in light of the police officer's experience and
surrounding conditions, to warrant the belief that the person detained has weapons concealed
about him. In his dissent for Esquillo v. People, Justice Bersamin reminds us that police officers
must not rely on a single suspicious circumstance. There should be "presence of more than one
seemingly innocent activity, which, taken together, warranted a reasonable inference of criminal
activity." The Constitution prohibits "unreasonable searches and seizures." Certainly, reliance on
only one suspicious circumstance or none at all will not result in a reasonable search.

70
ERDIE E. AMBROCIO Remedial Law Review 1
2014-0308 Criminal Procedure – Case Digests

33. ANTIQUERA VS PEOPLE


714 SCRA

FACTS: At around 4:45 A.M. of February 11, 2004, police officers Gregorio and Laurence
while onboard a patrol car, saw two unidentified men rush out of a house in David St., Pasay
City. Sensing something amiss, the police officers approached the house and peeked inside the
partially opened door, where they saw George holding an improvised tooter and a pink lighter,
and beside him, his live-in partner, Corazon. Because of this, they entered the house, and arrested
George and Corazon. A search of the immediate surroundings revealed a wooden box containing
improvised tooter, scoop 10 sachets of suspected shabu, and strips of aluminum oil. Because of
they, they were charged with illegal possession of drugs paraphernalia. Only George appealed
the decision rendered by the RTC convicting him as charged, since Corazon jumped bail. The
Court of Appeals denied his appeal, hence he elevated his case to the Supreme Court. Both lower
courts justified the conviction of George, citing his arrest was a valid warrantless arrest under
Section 5, Rule 113 of the Rules of Court.

ISSUE/S: Whether or not there was a valid warrantless arrest?

HELD: The prosecution’s theory, upheld by both the RTC and the CA, is that it was a case of
valid warrantless arrest in that the police officers saw accused Antiquera and Cruz through the
door of their house, in the act of having a pot session. That valid warrantless arrest gave the
officers the right as well to search the living room for objects relating to the crime and thus seize
the paraphernalia they found there.

The prosecution contends that, since the seized paraphernalia tested positive for shabu, they were
no doubt used for smoking, consuming, administering, injecting, ingesting, or introducing
dangerous drug into the body in violation of Section 12 of Republic Act 9165. That the accused
tested negative for shabu, said the prosecution, had no bearing on the crime charged which was
for illegal possession of drug paraphernalia, not for illegal use of dangerous drugs. The
prosecution added that even assuming that the arrest of the accused was irregular, he is already
considered to have waived his right to question the validity of his arrest when he voluntarily
submitted himself to the court’s jurisdiction by entering a plea of not guilty.

Section 5(a), Rule 113 of the Rules of Criminal Procedure provides that a “peace officer or a
private person may, without a warrant, arrest a person when, in his presence, the person to be
arrested has committed, is actually committing, or is attempting to commit an offense.” This is
an arrest in flagrante delicto. The overt act constituting the crime is done in the presence or
within the view of the arresting officer. But the circumstances here do not make out a case of
arrest made in flagrante delicto.

1. The police officers claim that they were alerted when they saw two unidentified men suddenly
rush out of 107 David Street, Pasay City. Since they suspected that a crime had been committed,
the natural thing for them to do was to give chase to the jeep that the two fleeing men boarded,
given that the officers were in a patrol car and a tricycle. Running after the fleeing suspects was
the more urgent task but the officers instead gave priority to the house even when they heard no

71
ERDIE E. AMBROCIO Remedial Law Review 1
2014-0308 Criminal Procedure – Case Digests

cry for help from it.

2. Admittedly, the police officers did not notice anything amiss going on in the house from the
street where they stood. Indeed, even as they peeked through its partially opened door, they saw
no activity that warranted their entering it.

Clearly, no crime was plainly exposed to the view of the arresting officers that authorized the
arrest of accused Antiquera without warrant under the above-mentioned rule. Considering that
his arrest was illegal, the search and seizure that resulted from it was likewise illegal.
Consequently, the various drug paraphernalia that the police officers allegedly found in the house
and seized are inadmissible, having proceeded from an invalid search and seizure. Since the
confiscated drug paraphernalia is the very corpus delicti of the crime charged, the Court has no
choice but to acquit the accused.

Finally, the failure of the accused to object to the irregularity of his arrest by itself is not enough
to sustain his conviction. A waiver of an illegal warrantless arrest does not carry with it a waiver
of the inadmissibility of evidence seized during the illegal warrantless arrest.” Accused is
therefore acquitted.

72
ERDIE E. AMBROCIO Remedial Law Review 1
2014-0308 Criminal Procedure – Case Digests

34. PEOPLE VS VASQUEZ


714 SCRA

FACTS: This is an appeal from the decision of CA which affirmed the joint decision of RTC in
a consolidated case, convicting the appellant Donald Vasquez y Sandigan (Don) of the crimes of
illegal sale and illegal possession of regulated drugs. Initially the case of illegal possession of
drugs was raffled but upon motion it was consolidated with the case of illegal sale of drugs. On
arraignment, the appellant pleaded not guilty to both charges. The pre-trial conference of the
cases was held, but the same was terminated without the parties entering into any stipulation of
facts.

During the trial of the case the prosecution stated the events. There was a confidential informant
reported to PO2 Trambulo about the illegal drug activities. Fajardo form a buy-bust team. It was
in the buy-bust operation that Don was arrested. RTC convicted the appellant of the crimes
charged. The RTC gave more credence to the prosecution’s evidence given that the presumption
of regularity in the performance of official duty on the part of the police officers was not
overcome. On appeal the Court of Appeals affirmedthe conviction of the appellant. Hence this
appeal. He argues that the police officers did not have a search warrant or a warrant of arrest at
the time he was arrested. This occurred despite the fact that the police officers allegedly had
ample time to secure a warrant of arrest against him. Inasmuch as his arrest was illegal, the
appellant avers that the evidence obtained as a result thereof was inadmissible in court.

ISSUE/S: Whether the appellant Don may assail the validity of arrest.

HELD: No. At the outset, the Court rules that the appellant can no longer assail the validity of
his arrest. We reiterated in People v. Tampis that "[a]ny objection, defect or irregularity
attending an arrest must be made before the accused enters his plea on arraignment. Having
failed to move for the quashing of the information against them before their arraignment,
appellants are now estopped from questioning the legality of their arrest. Any irregularity was
cured upon their voluntary submission to the trial court’s jurisdiction." Be that as it may, the fact
of the matter is that the appellant was caught in flagrante delicto of selling illegal drugs to an
undercover police officer in a buy-bust operation. His arrest, thus, falls within the ambit of
Section 5(a), Rule 11354 of the Revised Rules on Criminal Procedure when an arrest made
without warrant is deemed lawful. Having established the validity of the warrantless arrest in this
case, the Court holds that the warrantless seizure of the illegal drugs from the appellant is
likewise valid. In People v. Cabugatan, the Court held that the interdiction against warrantless
searches and seizures, however, is not absolute and such warrantless searches and seizures have
long been deemed permissible by jurisprudence in instances of (1) search of moving vehicles, (2)
seizure in plain view, (3) customs searches, (4) waiver or consented searches, (5) stop and frisk
situations (Terry search), and search incidental to a lawful arrest. The last includes a valid
warrantless arrest, for, while as a rule, an arrest is considered legitimate [if] effected with a valid
warrant of arrest, the Rules of Court recognize permissible warrantless arrest, to wit: (1) arrest in
flagrante delicto, (2) arrest effected in hot pursuit, and (3) arrest of escaped prisoners. Thus, the
appellant cannot seek exculpation by invoking belatedly the invalidity of his arrest and the
subsequent search upon his person.

73
ERDIE E. AMBROCIO Remedial Law Review 1
2014-0308 Criminal Procedure – Case Digests

35. ZUNO VS. CABEBE

FACTS: The instant administrative case stemmed from the sworn complaint of Chief State
Prosecutor Jovencito R. Zuño of the Department of Justice, against Judge Alejandrino C.
Cabebe, then Presiding Judge, Regional Trial Court. The charges are knowingly rendering an
unjust judgment, gross ignorance of the law and partiality. In his complaint, Chief State
Prosecutor Zuño alleged that Criminal Case for illegal possession of prohibited or regulated
drugs was filed with the Regional Trial Court, against Rey DaquepArcangel, Victorino Gamet
Malabed, William Roxas Villanueva, all police officers, Jocelyn Malabed Manuel and Pelagio
Valencia Manuel. Upon arraignment, all the accused, assisted by their counsel de parte, pleaded
not guilty to the crime charged. On May 6, 2002, the accused filed a motion to dismiss invoking
as ground the right of the accused to a speedy trial. On November 5, 2002, respondent judge
motu propio issued an Order granting bail to the accused, fixing the bail for each at P70,000.00
in cash or property bond at P120,000.00, except for accused Evelyn Manuel whose bail was
fixed at P20,000.00 in cash. Respondent judge issued the Order without the accused’s application
or motion for bail. The prosecution then filed a motion for reconsideration. Instead of acting
thereon, respondent judge issued an order inhibiting himself from further proceeding with the
case, realizing that what he did was patently irregular. Complainant thus prays that respondent
judge be dismissed from the service with forfeiture of all benefits and be disbarred from the
practice of law. In his comment, respondent denied the charges. While admitting that he issued
the Order granting bail to the accused without any hearing, “the same was premised on the
constitutional right of the accused to a speedy trial.” The prosecution did not object to the grant
of bail to the accused. On March 26, 2003, respondent judge compulsorily retired. In his Report
dated July 7, 2003, Deputy Court Administrator Jose P. Perez found respondent judge liable for
gross ignorance of the law and recommended that a fine ofP20,000.00 be imposed upon him,
with a stern warning that a repetition of the same or similar offense will be dealt with more
severely.

ISSUE/S: W/N the judge is guilty of gross ignorance of the law by granting bail without hearing.

HELD: YES. In Docena-Caspe vs. Judge Arnulfo O. Bugtas, we held that jurisprudence is
replete with decisions on the procedural necessity of a hearing, whether summary or otherwise,
relative to the grant of bail, especially in cases involving offenses punishable by death, reclusion
perpetua, or life imprisonment, where bail is a matter of discretion. Under the present Rules, a
hearing is mandatory in granting bail whether it is a matter of right or discretion. It must be
stressed that the grant or the denial of bail in cases where bail is a matter of discretion, hinges on
the issue of whether or not the evidence of guilt of the accused is strong, and the determination
of whether or not the evidence is strong is a matter of judicial discretion which remains with the
judge. In order for the latter to properly exercise his discretion, he must first conduct a hearing to
determine whether the evidence of guilt is strong. In fact, even in cases where there is no petition
for bail, a hearing should still be held. There is no question that respondent judge granted bail to
the accused without conducting a hearing, in violation of Sections 8 and 18, Rule 114 of the
Revised Rules of Criminal Procedure. In Cortes vs. Catral, we laid down the following rules
outlining the duties of the judge in case an application for bail is filed:

74
ERDIE E. AMBROCIO Remedial Law Review 1
2014-0308 Criminal Procedure – Case Digests

1. In all cases whether bail is a matter of right or discretion, notify the prosecutor of the
hearing of the application for bail or require him to submit his recommendation.
2. Where bail is a matter of discretion, conduct a hearing of the application for bail
regardless of whether or not the prosecution refuses to present evidence to show that the
guilt of the accused is strong for the purpose of enabling the court to exercise its sound
discretion (Section 7 and 8, id.);
3. Decide whether the guilt of the accused is strong based on the summary of evidence of
the prosecution;
4. If the guilt of the accused is not strong, discharge the accused upon the approval of the
bail bond (Section 19, id.); otherwise the petition should be denied.

Based on the above-cited procedure, after the hearing, the court’s order granting or refusing
bail must contain a summary of the evidence of the prosecution and based thereon, the judge
should formulate his own conclusion as to whether the evidence so presented is strong
enough to indicate the guilt of the accused. Respondent judge did not follow the above Rules
and procedure enumerated in Cortes. He did not conduct a hearing before he granted bail to
the accused, thus depriving the prosecution of an opportunity to interpose objections to the
grant of bail. Irrespective of his opinion on the strength or weakness of evidence to prove the
guilt of the accused, he should have conducted a hearing and thereafter made a summary of
the evidence of the prosecution. The importance of a bail hearing and a summary of evidence
cannot be downplayed, these are considered aspects of procedural due process for both the
prosecution and the defense; its absence will invalidate the grant or denial of bail.
Respondent’s contention is bereft of merit. There is no indication in the records of the
criminal case that the prosecution has intentionally delayed the trial of the case. Even
assuming there was delay, this does not justify the grant of bail without a hearing. This is
utter disregard of the Rules. The requirement of a bail hearing has been incessantly stressed
by this Court. In the same vein, the Code of Judicial Conduct enjoins judges to be conversant
with the law and the Rules and maintain professional competence; and by the very nature of
his office, should be circumspect in the performance of his duties. He must render justice
without resorting to shortcuts clearly uncalled for. Obviously, respondent failed to live up to
these standards.

75
ERDIE E. AMBROCIO Remedial Law Review 1
2014-0308 Criminal Procedure – Case Digests

36. GOVERNMENT OF HONG KONG SPEC. ADM. REGION VS OLALIA

FACTS: Private respondent Muñoz was charged before Hong Kong Court. Warrants of arrest
were issued and by virtue of a final decree the validity of the Order of Arrest was upheld. The
petitioner Hong Kong Administrative Region filed a petition for the extradition of the private
respondent. In the same case, a petition for bail was filed by the private respondent.

The petition for bail was denied by reason that there was no Philippine law granting the same in
extradition cases and that the respondent was a high “flight risk”. Private respondent filed a
motion for reconsideration and was granted by the respondent judge subject to the following
conditions:
1. Bail is set at Php750,000.00 in cash with the condition that accused hereby undertakes
that he will appear and answer the issues raised in these proceedings and will at all times
hold himself amenable to orders and processes of this Court, will further appear for
judgment. If accused fails in this undertaking, the cash bond will be forfeited in favor of
the government;
2. Accused must surrender his valid passport to this Court;
3. The Department of Justice is given immediate notice and discretion of filing its own
motion for hold departure order before this Court even in extradition proceeding; and
4. Accused is required to report to the government prosecutors handling this case or if they
so desire to the nearest office, at any time and day of the week; and if they further desire,
manifest before this Court to require that all the assets of accused, real and personal, be
filed with this Court soonest, with the condition that if the accused flees from his
undertaking, said assets be forfeited in favor of the government and that the
corresponding lien/annotation be noted therein accordingly.

ISSUE/S: WON a potential extraditee is entitled to post bail

HELD: Petitioner alleged that the trial court committed grave abuse of discretion amounting to
lack or excess of jurisdiction in admitting private respondent to bail; that there is nothing in the
Constitution or statutory law providing that a potential extraditee has a right to bail, the right
being limited solely to criminal proceedings. On the other hand, private respondent maintained
that the right to bail guaranteed under the Bill of Rights extends to a prospective extraditee; and
that extradition is a harsh process resulting in a prolonged deprivation of one’s liberty.
In this case, the Court reviewed what was held in Government of United States of America v.
Hon. Guillermo G. Purganan, Presiding Judge, RTC of Manila, Branch 42, and Mark B. Jimenez,
a.k.a. Mario Batacan Crespo GR No. 153675 April 2007, that the constitutional provision on bail
does not apply to extradition proceedings, the same being available only in criminal proceedings.

The Court took cognizance of the following trends in international law:


1. the growing importance of the individual person in public international;
2. the higher value now being given to human rights;
3. the corresponding duty of countries to observe these universal human rights in fulfilling
their treaty obligations; and

76
ERDIE E. AMBROCIO Remedial Law Review 1
2014-0308 Criminal Procedure – Case Digests

4. the duty of this Court to balance the rights of the individual under our fundamental law,
on one hand, and the law on extradition, on the other. In light of the recent developments
in international law, where emphasis is given to the worth of the individual and the
sanctity of human rights, the Court departed from the ruling in Purganan, and held that an
extraditee may be allowed to post bail.

77
ERDIE E. AMBROCIO Remedial Law Review 1
2014-0308 Criminal Procedure – Case Digests

37. PEOPLE VS SANDIGANBAYAN

FACTS: On November 2000, as an offshoot of the impeachment proceedings against Joseph


Ejercito Estrada, then President of the Republic of the Philippines, five criminal complaints
against the former President and members of his family, his associates, friends and conspirators
were filed with the Office of the Ombudsman.

On April 4, 2001, the Ombudsman issued a Joint Resolution finding probable cause warranting
the filing with the Sandiganbayan of several criminal Information against the former President
and the other respondents therein. One of the Informations was for the crime of plunder under
Republic Act [RA] No. 7080 and among the respondents was herein petitioner Jose Jinggoy
Estrada, then mayor of San Juan, Metro Manila.

The Information was amended and filed on April 18, 2001. Docketed as Criminal Case No.
26558, the case was assigned to [the] respondent Third Division of the Sandiganbayan. The
amended information referred to, like the original, charged respondent Jinggoy, together with the
former President and several others, with plunder, defined and penalized under RA No. 7080, as
amended by Section 12 of RA No. 7659. From the denial action of the Sandiganbayan
immediately adverted to, Jinggoy interposed a petition for certiorari before this Court claiming
that the respondent Sandiganbayan committed grave abuse of discretion in, inter alia, (a)
sustaining the charge against him for alleged offenses and with alleged conspirators with whom
he is not even connected, and (b) in not fixing bail for him. Pending resolution of this petition,
docketed as G.R. No. 148965, Jinggoy filed with the Sandiganbayan an „Urgent Second Motion
for Bail for Medical Reasons. The Ombudsman opposed the motion. For three (3) days in
September 2001, the Sandiganbayan conducted hearings on the motion for bail, with one Dr.
Roberto Anastacio of the Makati Medical Center appearing as sole witness for Jinggoy.

On December 18, 2001, Jinggoy filed with the Court an Urgent Motion praying for early
resolution of his Petition for Bail on Medical/Humanitarian Considerations. He reiterated his
earlier plea for bail filed with the Sandiganbayan. On the same day, the Court referred the motion
to the Sandiganbayan for resolution and directed said court to make a report, not later than 8:30
in the morning of December 21, 2001. The report was submitted as directed. Attached to the
Report was a copy of the Sandiganbayan Resolution dated December 20, 2001 denying
Jinggoy’s motion for bail for lack of factual basis. According to the graft court, basing its
findings on the earlier testimony of Dr. Anastacio, Jinggoy failed to submit sufficient evidence to
convince the court that the medical condition of the accused requires that he be confined at home
and for that purpose that he be allowed to post bail.

On February 26, 2002, the Court dismissed Jinggoy’ petition. On April 17, 2002, Jinggoy filed
before the Sandiganbayan an Omnibus Application for Bail against which the prosecution filed
its comment and opposition. Bail hearings were then conducted, followed by the submission by
the parties of their respective memoranda. In the herein assailed Resolution of March 6, 2003,
respondent Sandiganbayan (Special Division) granted the omnibus application for bail. Petitioner
filed a motion for reconsideration thereto which the respondent court denied via the herein
equally assailed May 30, 2003 Resolution. Hence, the present petition on the submission that

78
ERDIE E. AMBROCIO Remedial Law Review 1
2014-0308 Criminal Procedure – Case Digests

respondent Special Division of the Sandiganbayan acted with grave abuse of discretion
amounting to lack or excess of jurisdiction

ISSUE/S: Whether the grant of bail was proper.

HELD: Even if the capital offense charged is bailable owing to the weakness of the evidence of
guilt, the right to bail may justifiably still be denied if the probability of escape is great. Here,
ever since the promulgation of the assailed Resolutions a little more than four (4) years ago,
Jinggoy does not, as determined by Sandiganbayan, seem to be a flight risk.

As we see it, the rulings in Castelo and Ty Sui Wong are not on all-fours applicable to and of
governing sway to the issue of the propriety of revoking Jinggoy’s release on bail. As it were, the
petitioner erroneously equates the provisional grant of bail to respondent Jinggoy to his virtual
acquittal in Criminal Case No. 26558. Petitioner is wrong. Castelo and Ty Sui Wong
contextually dealt with the guilt of culprits therein for the crimes of murder after all the evidence
had been adduced. Unlike in this proceeding, the propriety of a grant of bail, given the evidence
for or against the bail application, was not an issue in Castelo and Ty Sui Wong. And in the
present case, respondent Sandiganbayan is still in the process of determining the facts and merits
of the main case.

The respondent court did not commit grave abuse of discretion in issuing its assailed resolutions,
because the grant of bail therein is predicated only on its preliminary appreciation of the
evidence.

79
ERDIE E. AMBROCIO Remedial Law Review 1
2014-0308 Criminal Procedure – Case Digests

38. OKABE VS GUTIERREZ

FACTS: Charged for Estafa, Petitioner filed a verified motion for judicial determination of
probable cause and to defer proceedings/arraignment, alleging that the only documents appended
to the Information submitted by the investigating prosecutor were respondent Maruyamas
affidavit-complaint for estafa and the resolution of the investigating prosecutor; the affidavits of
the witnesses of the complainant, the respondents counter-affidavit and the other evidence
adduced by the parties were not attached thereto. The petitioner further alleged that the
documents submitted by the investigating prosecutor were not enough on which the trial court
could base a finding of probable cause for estafa against her.

The court denied the petitioners motions based on its personal examination and consideration of
the Information, the affidavit-complaint of respondent Maruyama and the resolution of the
investigating prosecutor duly approved by the city prosecutor, the court found probable cause for
the petitioner’s arrest. Since the petitioners motion for a determination of probable cause was
made after the court had already found probable cause and issued a warrant for the petitioner’s
arrest, and after the latter filed a personal bail bond for her provisional liberty, such motion was a
mere surplusage;

In denying her motion for a determination of probable cause, she posits that the respondent judge
acted with grave abuse of discretion amounting to excess or lack of jurisdiction.

ISSUE/S: If the RTC judge may rely on investigating prosecutor’s resolution in the
determination of probable cause for the arrest of the accused

HELD: NO. In determining the existence or non-existence of probable cause for the arrest of the
accused, the judge should not rely solely on the said report.[ The judge should consider not only
the report of the investigating prosecutor but also the affidavit/affidavits and the documentary
evidence of the parties, the counter-affidavit of the accused and his witnesses, as well as the
transcript of stenographic notes taken during the preliminary investigation, if any, submitted to
the court by the investigating prosecutor upon the filing of the Information. The duty to make
such determination is personal and exclusive to the issuing judge. He cannot abdicate his duty
and rely on the certification of the investigating prosecutor that he had conducted a preliminary
investigation in accordance with law and the Rules of Court, as amended, and found probable
cause for the filing of the Information.

The task of the presiding judge when the Information is filed with the court is first and foremost
to determine the existence or non-existence of probable cause for the arrest of the accused.
Probable cause is meant such set of facts and circumstances which would lead a reasonably
discreet and prudent man to believe that the offense charged in the Information or any offense
included therein has been committed by the person sought to be arrested. The purpose of the
mandate of the judge to first determine probable cause for the arrest of the accused is to insulate
from the very start those falsely charged of crimes from the tribulations, expenses and anxiety of
a public trial.

80
ERDIE E. AMBROCIO Remedial Law Review 1
2014-0308 Criminal Procedure – Case Digests

Under Section 6, Rule 112 of the Rules of Court in relation to Section 2, Article III of the 1987
Constitution, the judge must make a personal determination of the existence or non-existence of
probable cause for the arrest of the accused. Under Section 1, Rule 112 of the Rules on Criminal
Procedure, the investigating prosecutor, in conducting a preliminary investigation of a case
cognizable by the RTC, is tasked to determine whether there is sufficient ground to engender a
well-founded belief that a crime has been committed and the respondent therein is probably
guilty thereof and should be held for trial. A preliminary investigation is for the purpose of
securing the innocent against hasty, malicious and oppressive prosecution, and to protect him
from an open and public accusation of a crime, from the trouble, expense and anxiety of a public
trial.

If the investigating prosecutor finds probable cause for the filing of the Information against the
respondent, he executes a certification at the bottom of the Information that from the evidence
presented, there is a reasonable ground to believe that the offense charged has been committed
and that the accused is probably guilty thereof. Such certification of the investigating prosecutor
is, by itself, ineffective. It is not binding on the trial court. Nor may the RTC rely on the said
certification as basis for a finding of the existence of probable cause for the arrest of the accused.

81
ERDIE E. AMBROCIO Remedial Law Review 1
2014-0308 Criminal Procedure – Case Digests

39. LEVISTE VS CA

FACTS: Jose Antonio Leviste was charged with the crime of murder but was convicted by the
RTC for the lesser crime of homicide. He appealed the RTC's decision to the CA then he field an
application for admission to bail pending appeal, due to his advanced age and health condition,
and claiming the absence of any risk or possibility of flight on his part.

The CA denied his application on the ground that the discretion to extend bail during the course
of appeal should be exercised with grave caution and only for strong reasons. That bail is not a
sick pass for an ailing or aged detainee or a prisoner needing medical care outside the prison
facility.

On this matter, Levisete questioned the ruling of the CA and averred that the CA committed
grave abuse of discretion in the denial of his application for bail considering that none of the
conditions justifying denial of bail under the Sec. 5 (3) Rule 114 of the Rules of Court was
present. That when the penalty imposed by the trial court is more than six years but not more
than 20 years and the circumstances in the above-mentioned provision are absent, bail must be
granted to an appellant pending appeal.

ISSUE/S: Whether or not the CA committed grave abuse of discretion in denying the application
for bail of Leviste.

HELD: No, under Sec 5 of Rule 114 bail is discretionary, upon conviction by the RTC of an
offense not punishable by death, reclusion perpetua, or life imprisonment. Under par. 3 of the
same rule if the penalty impose is more than 6 years the accused shall be denied bail, or his bail
be cancelled upon a showing by the prosecution, with notice to the accused, of the following or
other circumstances:

1. that he is a recidivist, quasi-recidivist, or habitual delinquent, or has committed the crime


aggravated by the circumstance of reiteration;
2. that he has previously escaped from legal confinement, evaded sentence, or violated the
conditions of his bail without a valid justification;
3. that he committed the offense while under probation, parole, or conditional pardon;
4. that the circumstances of his case indicate the probability of flight if released on bail; or
5. that there is undue risk that he may commit another crime during the pendency of the
appeal.
That bail is expressly declared to be discretionary pending appeal and it cannot be said
that CA committed grave abuse of discretion. After conviction by the trial court, the
presumption of innocence terminates and, accordingly, the constitutional right to bail
ends, from then on the grant of bail is subject to judicial discretion.

82
ERDIE E. AMBROCIO Remedial Law Review 1
2014-0308 Criminal Procedure – Case Digests

40. ENRILE VS SANDIGANBAYAN

FACTS: On June 5, 2014, Petitioner Juan Ponce Enrile was charged with plunder in the
Sandiganbayan on the basis of his purported involvement in the Priority Development Assistance
Fund (PDAF) Scam. Initially, Enrile in an Omnibus Motion requested to post bail, which the
Sandiganbayan denied. On July 3, 2014, a warrant for Enrile's arrest was issued, leading to
Petitioner's voluntary surrender. Petitioner again asked the Sandiganbayan in a Motion to Fix
Bail which was heard by the Sandiganbayan. Petitioner argued that: (a) Prosecution had not yet
established that the evidence of his guilt was strong; (b) that, because of his advanced age and
voluntary surrender, the penalty would only be reclusion temporal, thus allowing for bail and; (c)
he is not a flight risk due to his age and physical condition. Sandiganbayan denied this in its
assailed resolution. Motion for Reconsideration was likewise denied.

ISSUE/S:
1. Whether or not bail may be granted as a matter of right unless the crime charged is
punishable by reclusion perpetua where the evidence of guilt is strong.
2. Whether or not petitioner is bailable because he is not a flight risk.

HELD:
1. YES. Bail as a matter of right – due process and presumption of innocence.

Article III, Sec. 14 (2) of the 1987 Constitution provides that in all criminal prosecutions, the
accused shall be presumed innocent until the contrary is proved. This right is safeguarded by the
constitutional right to be released on bail.

The purpose of bail is to guarantee the appearance of the accused at trial and so the amount of
bail should be high enough to assure the presence of the accused when so required, but no higher
than what may be reasonably calculated to fulfill this purpose.

Bail as a matter of discretion

Right to bail is afforded in Sec. 13, Art III of the 1987 Constitution and repeted in Sec. 7, Rule
114 of the Rules of Criminal Procedure to wit:

Capital offense of an offense punishable by reclusion perpetua or life imprisonment, not bailable.
— No person charged with a capital offense, or an offense punishable by reclusion perpetua or
life imprisonment, shall be admitted to bail when evidence of guilt is strong, regardless of the
stage of the criminal prosecution.

The general rule: Any person, before conviction of any criminal offense, shall be bailable.

Exception: Unless he is charged with an offense punishable with reclusion perpetua [or life
imprisonment] and the evidence of his guilt is strong.

83
ERDIE E. AMBROCIO Remedial Law Review 1
2014-0308 Criminal Procedure – Case Digests

Thus, denial of bail should only follow once it has been established that the evidence of guilt is
strong. Where evidence of guilt is not strong, bail may be granted according to the discretion of
the court.

2. YES. Petitioner's poor health justifies his admission to bail

The Supreme Court took note of the Philippine's responsibility to the international community
arising from its commitment to the Universal Declaration of Human Rights. We therefore have
the responsibility of protecting and promoting the right of every person to liberty and due
process and for detainees to avail of such remedies which safeguard their fundamental right to
liberty.

Sandiganbayan committed grave abuse of discretion. They arbitrarily ignored the objective of
bail to ensure the appearance of the accused during the trial and unwarrantedly disregarded the
clear showing of the fragile health and advanced age of Petitioner. As such the Sandiganbayan
gravely abused its discretion in denying the Motion to Fix Bail. It acted whimsically and
capriciously and was so patent and gross as to amount to an evasion of a positive duty [to allow
petitioner to post bail].

84
ERDIE E. AMBROCIO Remedial Law Review 1
2014-0308 Criminal Procedure – Case Digests

41. DEL CASTILLO VS PEOPLE

FACTS: Pursuant to a confidential information that petitioner Del Castillo was engaged in
selling shabu, police officers headed by SPO3 Bienvenido Masnayon, after conducting
surveillance and test-buy operation at the house of petitioner, secured a search warrant from the
RTC. Upon arrival to the residence of Del Castillo to implement the search warrant, SPO3
Masnayon claimed that he saw petitioner run towards a small structure, a nipa hut, in front of his
house. Masnayon chased him but to no avail, because he and his men were not familiar with the
entrances and exits of the place. They all went back to the residence of Del Castillo and
requested his men to get a barangay tanod and a few minutes thereafter, his men returned with
two barangay tanods who searched the house of petitioner including the nipa hut where the
petitioner allegedly ran for cover. His men who searched the residence of the petitioner found
nothing, but one of the barangay tanods was able to confiscate from the nipa hut several articles,
including four (4) plastic packs containing white crystalline substance.

Thus, an information was filed against Del Castillo for violation of Section 16, Article III of R.A.
6425 and was found guilty by the RTC and affirmed by the Court of Appeals. Petitioner filed
with the Supreme Court the petition for certiorari contending among others that CA erred in
finding him guilty beyond reasonable doubt of illegal possession of prohibited drugs, because he
could not be presumed to be in possession of the same just because they were found inside the
nipa hut.

ISSUE/S: Whether the search made in the nipa hut is valid.

HELD: No, The records are void of any evidence to show that petitioner owns the nipa hut in
question nor was it established that he used the said structure as a shop. The RTC, as well as the
CA, merely presumed that petitioner used the said structure due to the presence of electrical
materials, the petitioner being an electrician by profession. The prosecution must prove that the
petitioner had knowledge of the existence and presence of the drugs in the place under his
control and dominion and the character of the drugs. With the prosecution’s failure to prove that
the nipa hut was under petitioner’s control and dominion, there casts a reasonable doubt as to his
guilt. In considering a criminal case, it is critical to start with the law’s own starting perspective
on the status of the accused — in all criminal prosecutions, he is presumed innocent of the
charge laid unless the contrary is proven beyond reasonable doubt. Proof beyond reasonable
doubt, or that quantum of proof sufficient to produce a moral certainty that would convince and
satisfy the conscience of those who act in judgment, is indispensable to overcome the
constitutional presumption of innocence.

85
ERDIE E. AMBROCIO Remedial Law Review 1
2014-0308 Criminal Procedure – Case Digests

42. MIGUEL VS SANDIGANBAYAN

FACTS: Then Vice Mayor Mercelita M. Lucido and other local officials of Koronadal City,
South Cotabato filed a lettercomplaint with the Office of the Ombudsman-Mindanao
(Ombudsman) charging the petitioner, among others, with violation of Republic Act (R.A.) No.
3019, in connection with the consultancy services for the architectural aspect, the engineering
design, and the construction supervision and management of the proposed Koronadal City public
market (project).

The Sandiganbayan ordered the Office of the Special Prosecutor (OSP) to conduct a
reinvestigation. On August 21, 2000, the petitioner, through counsel, followed suit and orally
moved for a reinvestigation, which the Sandiganbayan likewise granted. The Sandiganbayan
gave the petitioner ten (10) days within which to file his counter-affidavit with the OSP.

The petitioner asked for extension twice and still failed to file his counter-affidavit. Despite the
extension period asked and given, the petitioner failed to file his counter-affidavit, prompting
Prosecutor Norberto B. Ruiz to declare that the petitioner had waived his right to submit
countervailing evidence (April 25, 2001 resolution). On July 31, 2001, then Ombudsman Aniano
Desierto approved the resolution.

The OSP filed a Motion to Suspend [the petitioner] Pendente Lite. The petitioner filed his
Vigorous Opposition based on the obvious and fatal defect of the information in failing to allege
that the giving of unwarranted benefits and advantages was done through manifest partiality,
evident bad faith or gross inexcusable negligence, since he was not accorded a pre-suspension
hearing

ISSUE/S: Whether the information against petitioner and petitioner's suspension valid.

HELD: Yes. In deference to the constitutional right of an accused to be informed of the nature
and the cause of the accusation against him,[31] Section 6, Rule 110 of the Revised Rules of
Criminal Procedure (Rules) requires, inter alia, that the information shall state the designation of
the offense given by the statute and the acts or omissions imputed which constitute the offense
charged. Additionally, the Rules requires that these acts or omissions and its attendant
circumstances must be stated in ordinary and concise language and in terms sufficient to enable a
person of common understanding to know what offense is being charged x x x and for the court
to pronounce judgment.

The test of the informations sufficiency is whether the crime is described in intelligible terms and
with such particularity with reasonable certainty so that the accused is duly informed of the
offense charged. In particular, whether an information validly charges an offense depends on
whether the material facts alleged in the complaint or information shall establish the essential
elements of the offense charged as defined in the law. The raison detre of the requirement in the
Rules is to enable the accused to suitably prepare his defense.

86
ERDIE E. AMBROCIO Remedial Law Review 1
2014-0308 Criminal Procedure – Case Digests

In arguing against the validity of the information, the petitioner appears to go beyond the
standard of a person of common understanding in appreciating the import of the phrase acting
with evident bad faith and manifest partiality. A reading of the information clearly reveals that
the phrase acting with evident bad faith and manifest partiality was merely a continuation of the
prior allegation of the acts of the petitioner, and that he ultimately acted with evident bad faith
and manifest partiality in giving unwarranted benefits and advantages to his co-accused private
individuals. This is what a plain and non-legalistic reading of the information would yield.

Suspension is valid. Section 13. Suspension and loss of benefits. Any public officer against
whom any criminal prosecution under a valid information under this Act or under the provisions
of the Revised Penal Code on bribery is pending in court, shall be suspended from office. Should
he be convicted by final judgment, he shall lose all retirement or gratuity benefits under any law,
but if he is acquitted, he shall be entitled to reinstatement and to the salaries and benefits which
he failed to receive during suspension, unless in the meantime administrative proceedings have
been filed against him.

Since a pre-suspension hearing is basically a due process requirement, when an accused public
official is given an adequate opportunity to be heard on his possible defenses against the
mandatory suspension under R.A. No. 3019, then an accused would have no reason to complain
that no actual hearing was conducted. It is well settled that to be heard does not only mean oral
arguments in court; one may be heard also through pleadings. Where opportunity to be heard,
either through oral arguments or pleadings, has been accorded, no denial of procedural due
process exists.

87
ERDIE E. AMBROCIO Remedial Law Review 1
2014-0308 Criminal Procedure – Case Digests

43. PEOPLE VS LARA

ISSUE/S: Whether the identification made by Sumulong, Atie and Manacob in the police line-up
is inadmissible because Lara stood therein without the assistance of counsel? (NO, identification
not custodial investigation)

HELD: NO. There was no legal compulsion to afford him a counsel during a police line-up
since the latter is not part of custodial investigation and this does not constitute a violation of his
right to counsel.

That he stood at the police line-up without the assistance of counsel did not render Sumulong's
identification of Lara inadmissible. The right to counsel is deemed to have arisen at the precise
moment custodial investigation begins and being made to stand in a police line-up is not the
starting point or a part of custodial investigation.

People v. Amestuzo: The guarantees of Sec. 12 (1), Art. III of the 1987 Constitution, or the so-
called Miranda rights, may be invoked only by a person while he is under custodial investigation.
Custodial investigation starts when the police investigation is no longer a general inquiry into an
unsolved crime but has begun to focus on a particular suspect taken into custody by the police
who starts the interrogation and propounds questions to the person to elicit incriminating
statements.

Police line-up is not part of the custodial investigation; hence, the right to counsel guaranteed by
the Constitution cannot yet be invoked at this stage.

The right to be assisted by counsel attaches only during custodial investigation and cannot be
claimed by the accused during identification in a police line-up because it is not part of the
custodial investigation process. This is because during a police line-up, the process has not yet
shifted from the investigatory to the accusatory and it is usually the witness or the complainant
who is interrogated and who gives a statement in the course of the line-up.

An exception to this rule is when the accused had been the focus of police attention at the start of
the investigation. In the case at bench, appellant was identified in a police line-up by prosecution
witnesses from a group of persons gathered for the purpose. However, there was no proof that
appellant was interrogated at all or that a statement or confession was extracted from him.
During the police line-up, the accusatory process had not yet commenced.

Assuming there was interrogation, any allegation of violation of rights during custodial
investigation is relevant and material only to cases in which an extrajudicial admission or
confession extracted from the accused becomes the basis of their conviction. Here, appellant was
convicted based on the testimony of a prosecution witness and not on his alleged uncounseled
confession or admission.

88
ERDIE E. AMBROCIO Remedial Law Review 1
2014-0308 Criminal Procedure – Case Digests

44. SANICO VS PEOPLE

FACTS: The petitioner Sanico and Marsito Batiquin were criminally charged for trespassing and
theft of minerals in the Municipal Circuit Trial Court in Cebu. The MCTC found both accused
guilty beyond reasonable doubt.

Sanico’s counsel filed a notice of appeal in the MCTC. Consequently, the RTC, Branch 25, in
Danao City ordered Sanico to file his memorandum on appeal. Sanico did not comply; hence, the
RTC ruled for the dismissal of the appeal.

Later on, one Atty. Dennis Cañete, another lawyer acting for Sanico, filed a motion for
reconsideration vis-à-vis the dismissal of the appeal, stating that Sanico had not filed the
memorandum on appeal because he had been beset with problems due to his wife’s debilitating
illness which eventually claimed her life, as well as his counsel, Atty. Baring’s own medical
condition which caused her to forget how she got this case and whom to contact as principal
counsel hereof.

The RTC denied the motion for reconsideration because of its lack of verification and affidavit
of merit; and because the supposed sickness of Sanico’s wife and the lapses of Atty. Baring were
not justifiable reasons.

Sanico, through Atty. Cañete, filed a petition for review in the CA, contesting his conviction, and
assailing the dismissal of his appeal for failure to file the memorandum on appeal. CA denied the
petition for review.

The petitioner moved for reconsideration, but his motion was denied.

The counsel for respondent Jennifer S. Tenio filed an Ex Parte Motion for Entry of Judgment,
which the RTC authorized the issuance of. Sanico filed an omnibus motion to recall the order
and to quash the entry of judgment, but the RTC denied the omnibus motion, noting that Sanico
did not provide the RTC with a copy of his petition for review; hence, the RTC had no way of
knowing about the pendency of his petition for review in the CA; and that in any case, the CA
had already denied his petition for review, while his motion for reconsideration had yet to be
acted upon by the CA. Sanico’s motion for reconsideration was denied by the RTC.

ISSUE/S: Whether or not the negligence of the petitioner’s counsel should be binding on the
petitioner.

HELD: No, without doubt, the petitioner could reasonably expect that his counsel would afford
to him competent legal representation. The mere failure of the counsel to observe a modicum of
care and vigilance in the protection of the interests of the petitioner as the client — as manifested
in the multiple defects and shortcomings discovered in the petition for review — was gross
negligence in any language because the defects were plainly avoidable by the simple application
of the relevant guidelines existing in the Rules of Court. If the incompetence of counsel was so
great and the error committed as a result was so serious that the client was prejudiced by a denial

89
ERDIE E. AMBROCIO Remedial Law Review 1
2014-0308 Criminal Procedure – Case Digests

of his day in court, the litigation ought to be reopened to give to the client another chance to
present his case. The legitimate interests of the petitioner, particularly the right to have his
conviction reviewed by the RTC as the superior tribunal, should not be sacrificed in the altar of
technicalities.

90
ERDIE E. AMBROCIO Remedial Law Review 1
2014-0308 Criminal Procedure – Case Digests

45. PEOPLE VS AYSON

FACTS: Felipe Ramos was a ticket freight clerk of the Philippine Airlines, assigned at its
Baguio City station. It was alleged that he was involved in irregularities in the sales of plane
tickets, the PAL management notified him of an investigation to be conducted. That
investigation was scheduled in accordance with PAL's Code of Conduct and Discipline, and the
Collective Bargaining Agreement signed by it with the Philippine Airlines Employees'
Association (PALEA) to which Ramos pertained. A letter was sent by Ramos stating his
willingness to settle the amount of P76,000. The findings of the Audit team were given to him,
and he refuted that he misused proceeds of tickets also stating that he was prevented from
settling said amounts. He proffered a compromise however this did not ensue. Two months after
a crime of estafa was charged against Ramos. Ramos pleaded not guilty. Evidence by the
prosecution contained Ramos’ written admission and statement, to which defendants argued that
the confession was taken without the accused being represented by a lawyer. Respondent Judge
did not admit those stating that accused was not reminded of his constitutional rights to remain
silent and to have counsel. A motion for reconsideration filed by the prosecutors was denied.
Hence this appeal.

ISSUE/S: Whether or Not the respondent Judge correct in making inadmissible as evidence the
admission and statement of accused.

HELD: No, the handwritten admission and statement of accused were declared to be admissible.

Right Against Self-Incrimination

The first right, against self-incrimination, mentioned in Section 20, Article IV of the 1973
Constitution, is accorded to every person who gives evidence, whether voluntarily or under
compulsion of subpoena, in any civil, criminal, or administrative proceeding. The right is NOT
to "be compelled to be a witness against himself.”

The precept set out in that first sentence has a settled meaning.It prescribes an "option of refusal
to answer incriminating questions and not a prohibition of inquiry.” It simply secures to a
witness, whether he be a party or not, the right to refue to answer any particular incriminatory
question, i.e., one the answer to which has a tendency to incriminate him for some crime.
However, the right can be claimed only when the specific question, incriminatory in character, is
actually put to the witness. It cannot be claimed at any other time. It does not give a witness the
right to disregard a subpoena, to decline to appear before the court at the time appointed, or to
refuse to testify altogether. The witness receiving a subpoena must obey it, appear as required,
take the stand, be sworn and answer questions. It is only when a particular question is addressed
to him, the answer to which may incriminate him for some offense, that he may refuse to answer
on the strength of the constitutional guaranty.

That first sentence of Section 20, Article IV of the 1973 Constitution does not impose on the
judge, or other officer presiding over a trial, hearing or investigation, any affirmative obligation
to advise a witness of his right against self-incrimination. It is a right that a witness knows or

91
ERDIE E. AMBROCIO Remedial Law Review 1
2014-0308 Criminal Procedure – Case Digests

should know, in accordance with the well-known axiom that everyone is presumed to know the
law, that ignorance of the law excuses no one. Furthermore, in the very nature of things, neither
the judge nor the witness can be expected to know in advance the character or effect of a
question to be put to the latter.

The right against self-incrimination is not self- executing or automatically operational. It must be
claimed. If not claimed by or in behalf of the witness, the protection does not come into play. It
follows that the right may be waived, expressly, or impliedly, as by a failure to claim it at the
appropriate time.

92
ERDIE E. AMBROCIO Remedial Law Review 1
2014-0308 Criminal Procedure – Case Digests

46. VILLAREAL VS PEOPLE

FACTS: In February 1991, seven freshmen law students of the Ateneo de Manila University
School of Law signified their intention to join the Aquila Legis Juris Fraternity (Aquila
Fraternity).

The neophytes, including victim, Lenny Villa, were subjected to initiation rites. After the second
day of initiation rites has ended, accused non-resident or alumni fraternity members Fidelito
Dizon (Dizon) and Artemio Villareal (Villareal) demanded that the rites be reopened. The head
of initiation rites, Nelson Victorino (Victorino), initially refused. Upon the insistence of Dizon
and Villareal, however, he reopened the initiation rites. The fraternity members, including Dizon
and Villareal, then subjected the neophytes to "paddling" and to additional rounds of physical
pain. Lenny received several paddle blows, one of which was so strong it sent him sprawling to
the ground. The neophytes heard him complaining of intense pain and difficulty in breathing.
After their last session of physical beatings, Lenny could no longer walk. He had to be carried by
the auxiliaries to the carport. Again, the initiation for the day was officially ended, and the
neophytes started eating dinner. They then slept at the carport.

After an hour of sleep, the neophytes were suddenly roused by Lennys shivering and incoherent
mumblings. Initially, Villareal and Dizon dismissed these rumblings, as they thought he was just
overacting. When they realized, though, that Lenny was really feeling cold, some of the Aquilans
started helping him. They removed his clothes and helped him through a sleeping bag to keep
him warm. When his condition worsened, the Aquilans rushed him to the hospital. Lenny was
pronounced dead on arrival.

The case however did not proceed until 2002. The unexplained length of time in which the case
remained dormant caused the dismissal of the case for being in violation of the accused’s’
constitutionally guaranteed right to a speedy trial.

ISSUE/S: Whether the CA correctly dismissed the case for violation of the accused’s’ right to
speedy trial

HELD: Yes. The SC did not see any grave abuse of discretion in the CAs dismissal of the case
against accused Escalona, Ramos, Saruca, and Adriano on the basis of the violation of their right
to speedy trial.

The absence of the records in the trial court [was] due to the fact that the records of the case were
elevated to the Court of Appeals, and the prosecutions failure to comply with the order of the
court a quo requiring it to secure certified true copies of the same. What is glaring from the
records is the fact that as early as September 21, 1995, the court a quo already issued an Order
requiring the prosecution, through the Department of Justice, to secure the complete records of
the case from the Court of Appeals. The prosecution did not comply with the said Order as in
fact, the same directive was repeated by the court a quo in an Order dated December 27, 1995.
Still, there was no compliance on the part of the prosecution. It is not stated when such order was
complied with. It appears, however, that even until August 5, 2002, the said records were still not

93
ERDIE E. AMBROCIO Remedial Law Review 1
2014-0308 Criminal Procedure – Case Digests

at the disposal of the trial court because the lack of it was made the basis of the said court in
granting the motion to dismiss filed by co-accused Concepcion.

It is likewise noticeable that from December 27, 1995, until August 5, 2002, or for a period of
almost seven years, there was no action at all on the part of the court a quo. Except for the
pleadings filed by both the prosecution and the petitioners, the latest of which was on January 29,
1996, followed by petitioner Sarucas motion to set case for trial on August 17, 1998 which the
court did not act upon, the case remained dormant for a considerable length of time. This
prolonged inactivity whatsoever is precisely the kind of delay that the constitution frowns upon.

94
ERDIE E. AMBROCIO Remedial Law Review 1
2014-0308 Criminal Procedure – Case Digests

47. PEOPLE VS ESTOMACA

FACTS: Estomaca is an illiterate laborer accused of raping his own daughter, Estelita. 5
complaints were filed and 2 of them are being challenged. These instances happened sometime
in 1993 and 1994 which both took place in their residence in Iloilo. The court imposed the
penalty of Reclusion Perpetua for the sexual assault in 1993 and the penalty of death for the
alleged rape in 1994.

The records show, however, that there was an irregularity in his arraignment where the
complaints were supposed to have all been read to him in the Ilonggo/local dialect. In this said
arraignment, the accused pleaded guilty to all 5 complaints but subsequently stated the he wasn’t
guilty of the other 3 cases against him

ISSUE/S: Whether the arraignment was valid or not

HELD: No. The transcript of the arraignment shows that it merely consisted of the bare reading
of the 5 complaints. It was reported in the transcript that “Reading the information/complaint to
the accused in Ilonggo/local dialect”. Since it was stated in the singular, the court speculates
whether all 5 complaints were actually ready, translated or explained to him on a level within his
comprehension considering his limited education. Parenthetically, there was no statement of
record that appellant fully understood that medium of expression. This assumes added
significance since Ilonggo, or properly called Hiligaynon, is a regional language, spoken in a
major part of Iloilo province, Negros Occidental and, with variations, in Capiz. Within a
province or major geographical area using a basic regional language, there may be other local
dialects spoken in certain parts thereof. If said indication in the aforequoted portion of the
transcript intended to convey that Ilonggo is merely a local dialect and was also the idiom
referred to, the same is egregious error; it would be different if local dialect was used to denote
an alternative and different medium but, inexplicably, without identifying what it was. The
significance of this distinction is found right in the provision of Section 1(a) of Rule 116 which,
cognizant of the aforestated linguistic variations, deliberately required that the complaint or
information be read to the accused in the language or the dialect known to him, to ensure his
comprehension of the charges. The Court takes judicial notice, because it is either of public
knowledge or readily capable of unquestionable demonstration, that in the central and
northwestern part of Iloilo province and all the way up to and throughout Antique, including
necessarily San Joaquin where the offenses were committed and of which appellant and his
family are natives, the local dialect is known as kinaray-a. Barring previous exposure to or as a
consequence of extended social or commercial intercourse, kinaray-a is not readily
understandable to nor spoken by those born to the Hiligaynon regional language or who have
lived in the areas under its sway for an appreciable period of time. The converse is true for those
whose native tongue is the dialect of kinaray-a, since they are generally not well-versed in
Ilonggo, or Hiligaynon. Since all the complaints are not only in English but in technical legal
language, we are again at sea as to whether and how the indictments were translated to Ilonggo
and/or to kinaray-a, or that the appellant was truly and honestly made aware of the charges and,
especially, the consequences of his guilty plea thereto. The records are silent and do not reveal
anything on this point, nor how the dialogue between the presiding judge and appellant was

95
ERDIE E. AMBROCIO Remedial Law Review 1
2014-0308 Criminal Procedure – Case Digests

translated. Yet a mans life is at stake while this Court wrestles with that dilemma created by an
omission of official duty.

Since no valid judgments can be had upon an invalid arraignment, the cases were set aside and
were remanded to the trial court.

96
ERDIE E. AMBROCIO Remedial Law Review 1
2014-0308 Criminal Procedure – Case Digests

48. PEOPLE VS PANGILINAN

FACTS: Alfredo Pangilinan was charged with 2 informations for the rape of his daughter, AAA.
He was arrested and detained with no bail recommended.

He then filed a petition for bail. The petition for bail was heard and prosecution presented
evidence. The Regional Trial Court denied petition Defense then presented its evidence.

The RTC after having discovered that Pangilinan was not yet arraigned, scheduled his
arraignment. Pangilinan pleaded not guilty. RTC convicted him and sentenced him to death.

ISSUE/S: Whether or not the arraignment was valid

HELD: YES. Pangilinan assails his conviction because he was not properly arraigned. Since he
was arraigned only after the case was submitted for decision, said irregularity, he argues, is a
procedural error which is prejudicial to the appellant and is tantamount to denial of his
constitutional right to be informed of the accusation against him. He claims that his subsequent
arraignment did not cure the defect in the trial proceedings because at the time the petition for
bail was heard, the trial court had not yet acquired jurisdiction over his person.

The accused is mistaken. When the hearings for his petition for bail were conducted, the trial
court had already acquired jurisdiction over his person. Settled is the rule that jurisdiction over
the person of the accused is acquired upon his arrest or voluntary appearance.19 In the case at
bar, the trial court acquired jurisdiction over the person of the appellant when he was arrested on
19 March 1997. His arrest, not his arraignment, conferred on the trial court jurisdiction over his
person.

Arraignment is the formal mode and manner of implementing the constitutional right of an
accused to be informed of the nature and cause of the accusation against him. The purpose of
arraignment is, thus, to apprise the accused of the possible loss of freedom, even of his life,
depending on the nature of the crime imputed to him, or at the very least to inform him of why
the prosecuting arm of the State is mobilized against him.

Admittedly, appellant was arraigned after the case was submitted for decision. The question is:
Were appellant’s rights and interests prejudiced by the fact that he was arraigned only at this
stage of the proceedings? We do not think so. Appellant’s belated arraignment did not prejudice
him. This procedural defect was cured when his counsel participated in the trial without raising
any objection that his client had yet to be arraigned. In fact, his counsel even cross-examined the
prosecution witnesses. His counsel’s active participation in the hearings is a clear indication that
he was fully aware of the charges against him; otherwise, his counsel would have objected and
informed the court of this blunder. Moreover, no protest was made when appellant was
subsequently arraigned. The parties did not question the procedure undertaken by the trial court.
It is only now, after being convicted and sentenced to two death sentences, that appellant cries
that his constitutional right has been violated. It is already too late to raise this procedural defect.
This Court will not allow it.

97
ERDIE E. AMBROCIO Remedial Law Review 1
2014-0308 Criminal Procedure – Case Digests

In People v. Cabale and People v. Atienza where the same issue was raised under similar
circumstances, we held that while the arraignment of appellant was conducted after the cases had
been submitted for decision, the error is non- prejudicial and has been fully cured. Since
appellant’s rights and interests were not prejudiced by this lapse in procedure, it only follows that
his constitutional right to be informed of the nature and cause of the accusation against him was
not violated.

98
ERDIE E. AMBROCIO Remedial Law Review 1
2014-0308 Criminal Procedure – Case Digests

49. DAAN VS SANDIGANBAYAN

FACTS: Daan (together with other co-accused) was charged for three counts of malversation of
public funds and three counts of falsification of public document by a public officer or employee
before the Sandiganbayan.

In the falsification cases, the accused offered to withdraw their plea of "not guilty" and substitute
the same with a plea of "guilty", provided, the mitigating circumstances of confession or plea of
guilt and voluntary surrender will be appreciated in their favor. In the alternative, if such
proposal is not acceptable, said accused proposed instead to substitute their plea of "not guilty"
to the crime of falsification of public document by a public officer or employee with a plea of
"guilty", but to the lesser crime of falsification of a public document by a private individual. On
the other hand, in the malversation cases, the accused offered to substitute their plea of "not
guilty" thereto with a plea of "guilty", but to the lesser crime of failure of an accountable officer
to render accounts.

Prosecution found this acceptable. Insofar as the malversation cases are concerned, the
prosecution was likewise amenable to the offer of said accused to plead "guilty" to the lesser
crime of failure of an accountable officer to render accounts.

The Sandiganbayan, however, denied petitioner’s Motion to Plea Bargain, despite favorable
recommendation by the prosecution, on the main ground that no cogent reason was presented to
justify its approval. The Sandiganbayan likewise denied the ensuing Motion for Reconsideration.

ISSUE/S: Whether or not The Sandiganbayan committed grave abuse of discretion in denying
Daan’s motion to plea bargain.

HELD: YES. Plea bargaining in criminal cases is a process whereby the accused and the
prosecution work out a mutually satisfactory disposition of the case subject to court approval. It
usually involves the defendant's pleading guilty to a lesser offense or to only one or some of the
counts of a multi-count indictment in return for a lighter sentence than that for the graver charge.

Plea bargaining is authorized under Section 2, Rule 116 of the Revised Rules of Criminal
Procedure, to wit:
 SEC. 2. Plea of guilty to a lesser offense. — At arraignment, the accused, with the
consent of the offended party and the prosecutor, may be allowed by the trial court to
plead guilty to a lesser offense which is necessarily included in the offense charged.
After arraignment but before trial, the accused may still be allowed to plead guilty to
said lesser offense after withdrawing his plea of not guilty. No amendment of the
complaint or information is necessary.

Ordinarily, plea bargaining is made during the pre-trial stage of the proceedings. Sections 1 and
2, Rule 118 of the Rules of Court, require plea bargaining to be considered by the trial court at
the pre-trial conference. But it may also be made during the trial proper and even after the
prosecution has finished presenting its evidence and rested its case. Thus, the Court has held that

99
ERDIE E. AMBROCIO Remedial Law Review 1
2014-0308 Criminal Procedure – Case Digests

it is immaterial that plea bargaining was not made during the pre-trial stage or that it was made
only after the prosecution already presented several witnesses.

In People of the Philippines v. Villarama, the Court ruled that the acceptance of an offer to plead
guilty to a lesser offense is not demandable by the accused as a matter of right but is a matter that
is addressed entirely to the sound discretion of the trial court. However, Villarama involved plea
bargaining after the prosecution had already rested its case.

As regards plea bargaining during the pre-trial stage, as in the present case, the trial court's
exercise of its discretion should neither be arbitrary nor should it amount to a capricious and
whimsical exercise of discretion. Grave abuse of discretion implies such capricious and
whimsical exercise of judgment as is equivalent to lack of jurisdiction or, in other words, where
the power is exercised in an arbitrary manner by reason of passion, prejudice, or personal
hostility; and it must be so patent or gross as to amount to an evasion of a positive duty or to a
virtual refusal to perform the duty enjoined by law, or to act at all in contemplation of law.

In the present case, the Sandiganbayan rejected petitioner's plea offer on the ground that
petitioner and the prosecution failed to demonstrate that the proposal would redound to the
benefit of the public. The Sandiganbayan believes that approving the proposal would "only serve
to trivialize the seriousness of the charges against them and send the wrong signal to potential
grafters in public office that the penalties they are likely to face would be lighter than what their
criminal acts would have merited or that the economic benefits they are likely to derive from
their criminal activities far outweigh the risks they face in committing them; thus, setting to
naught the deterrent value of the laws intended to curb graft and corruption in government."

Apparently, the Sandiganbayan has proffered valid reasons in rejecting petitioner's plea offer.
However, subsequent events and higher interests of justice and fair play dictate that petitioner's
plea offer should be accepted. The present case calls for the judicious exercise of this Court's
equity jurisdiction.

In the cases at bar, there is no dispute that JOSELITO RANIERO J. DAAN has already restituted
the total amount of P18,860.00 as per official receipt issued by the provincial government of
Leyte dated February 26, 2002. In short, the damage caused to the government has already been
restituted by the accused.

There is also no dispute that accused DAAN voluntarily surrendered in the instant cases.
Moreover, the accused is also willing to plead guilty to a lesser offense which to our mind,
merits consideration.

100
ERDIE E. AMBROCIO Remedial Law Review 1
2014-0308 Criminal Procedure – Case Digests

50. PEOPLE VS JANJALANI

FACTS: An RRCG bus was plying its usual route, from its Navotas bus terminal towards its
Alabang bus terminal via EDSA. Around 6:30 to 7:30 in the evening, while they were about to
move out of the Guadalupe-EDSA southbound bus stop, the bus conductor noticed two men
running after the bus. The two insisted on getting on the bus, so the conductor obliged and let
them in.

According to Elmer Andales, the bus conductor, he immediately became wary of the two men,
because, even if they got on the bus together, the two sat away from each other—one sat two
seats behind the driver, while the other sat at the back of the bus. At the time, there were only 15
passengers inside the bus. He also noticed that the eyes of one of the men were reddish. When he
approached the person near the driver and asked him whether he was paying for two passengers,
the latter looked dumb struck by the question. He then stuttered and said he was paying for two
and gave PhP20. Andales grew more concerned when the other man seated at the back also paid
for both passengers. At this point, Andales said he became more certain that the two were up to
no good, and that there might be a holdup.

Andales said he became more suspicious because both men kept on asking him if the bus was
going to stop at Ayala Avenue.

As soon as the bus reached the stoplight at the corner of Ayala Avenue and EDSA, the two men
insisted on getting off the bus. According to Andales, the bus driver initially did not want to let
them off the bus, because a Makati ordinance prohibited unloading anywhere except at
designated bus stops. Eventually, the bus driver gave in and allowed the two passengers to alight.
The two immediately got off the bus and ran towards Ayala Avenue. Moments after, Andales felt
an explosion. He then saw fire quickly engulfing the bus. He ran out of the bus towards a nearby
mall. After a while, he went back to where the bus was. He saw their bus passengers either lying
on the ground or looking traumatized. A few hours after, he made a statement before the Makati
Police Station narrating the whole incident.

Shortly before the explosion, the spokesperson of the Abu Sayyaf Group—Abu Solaiman—
announced over radio station DZBB that the group had a Valentine’s Day “gift” for former
President Gloria Macapagal-Arroyo. After the bombing, he again went on radio and warned of
more bomb attacks.

As stipulated during pretrial, accused Trinidad gave ABS-CBN News Network an exclusive
interview sometime after the incident, confessing his participation in the Valentine’s Day
bombing incident. In another exclusive interview on the network, accused Baharan likewise
admitted his role in the bombing incident. Finally, accused Asali gave a television interview,
confessing that he had supplied the explosive devices for the 14 February 2005 bombing. The
bus conductor identified the accused Baharan and Trinidad, and confirmed that they were the
two men who had entered the RRCG bus on the evening of 14 February.

101
ERDIE E. AMBROCIO Remedial Law Review 1
2014-0308 Criminal Procedure – Case Digests

Members of the Abu Sayyaf Group—namely Khaddafy Janjalani, Gamal B. Baharan, Angelo
Trinidad, Gappal Bannah Asali, Jainal Asali, Rohmat Abdurrohim a.k.a. Abu Jackie or Zaky, and
other “John” and “Jane Does”—were then charged with multiple murder and multiple frustrated
murder. Only Baharan, Trinidad, Asali, and Rohmat were arrested, while the other accused
remain at-large.

On their arraignment for the multiple murder charge, Baharan, Trinidad, and Asali all entered a
plea of guilty. On the other hand, upon arraignment for the multiple frustrated murder charge,
accused Asali pled guilty. Accused Trinidad and Baharan pled not guilty. Rohmat pled not guilty
to both charges.

In the light of the pretrial stipulations, the trial court asked whether accused Baharan and
Trinidad were amenable to changing their “not guilty” pleas to the charge of multiple frustrated
murder, considering that they pled “guilty” to the heavier charge of multiple murder, creating an
apparent inconsistency in their pleas. Defense counsel conferred with accused Baharan and
Trinidad and explained to them the consequences of the pleas. The two accused acknowledged
the inconsistencies and manifested their readiness for re-arraignment. After the Information was
read to them, Baharan and Trinidad pled guilty to the charge of multiple frustrated murder.

ISSUE/S: Whether or not the trial court gravely erred in accepting accused-appellants’ plea of
guilt despite insufficiency of searching inquiry into the voluntariness and full comprehension of
the consequences of the said plea.

HELD: Yes. The SC ruled that the trial court judges are required to observe the following
procedure under Section 3, Rule 116 of the Rules of Court

The requirement to conduct a searching inquiry applies more so in cases of re-arraignment. In


People v. Galvez, the Court noted that since accused-appellant’s original plea was “not guilty,”
the trial court should have exerted careful effort in inquiring into why he changed his plea to
“guilty.”

Likewise, the requirement to conduct a searching inquiry should not be deemed satisfied in cases
in which it was the defense counsel who explained the consequences of a “guilty” plea to the
accused, as it appears in this case. In People v. Alborida, this Court found that there was still an
improvident plea of guilty, even if the accused had already signified in open court that his
counsel had explained the consequences of the guilty plea; that he understood the explanation of
his counsel; that the accused understood that the penalty of death would still be meted out to
him; and that he had not been intimidated, bribed, or threatened.

The conduct of a searching inquiry remains the duty of judges, as they are mandated by the rules
to satisfy themselves that the accused had not been under coercion or duress; mistaken
impressions; or a misunderstanding of the significance, effects, and consequences of their guilty
plea. This requirement is stringent and mandatory. Nevertheless, the High Court is not unmindful
of the context under which the re-arraignment was conducted or of the factual milieu
surrounding the finding of guilt against the accused. The Court observes that accused Baharan

102
ERDIE E. AMBROCIO Remedial Law Review 1
2014-0308 Criminal Procedure – Case Digests

and Trinidad previously pled guilty to another charge— multiple murder—based on the same act
relied upon in the multiple frustrated murder charge. The Court further notes that prior to the
change of plea to one of guilt, accused Baharan and Trinidad made two other confessions of
guilt—one through an extrajudicial confession (exclusive television interviews, as stipulated by
both accused during pretrial), and the other via judicial admission (pretrial stipulation).
Considering the foregoing circumstances, we deem it unnecessary to rule on the sufficiency of
the “searching inquiry” in this instance. Remanding the case for re-arraignment is not warranted,
as the accused’s plea of guilt was not the sole basis of the condemnatory judgment under
consideration.

103
ERDIE E. AMBROCIO Remedial Law Review 1
2014-0308 Criminal Procedure – Case Digests

51. ABS-CBN CORP. VS GOZON

FACTS: The controversy arose from GMA-7’s news coverage on the homecoming of Filipino
overseas worker and hostage victim Angelo dela Cruz.

As summarized by the CA: Overseas Filipino worker Angelo dela Cruz was kidnapped by Iraqi
militants and as a condition for his release, a demand was made for the withdrawal of Filipino
troops in Iraq. After negotiations, he was released by his captors and was scheduled to return to
the country in the afternoon of 22 July 2004. Occasioned by said homecoming and the public
interest it generated, both GMA Network, Inc. and petitioner made their respective broadcasts
and coverage of the live event.

Allegedly, GMA-7 did not receive any notice or was not aware that Reuters was airing footages
of ABS-CBN. GMA-7’s news control room staff saw neither the “No Access Philippines” notice
nor a notice that the video feed was under embargo in favor of ABS-CBN.

ABS-CBN filed the Complaint for copyright infringement against Dela Pena-Reyes and
Manalastas to which the prosecutor found probable cause to indict the latter. Respondents filed
the Petition for Review before the Department of Justice. DOJ Secretary Raul Gonzalez ruled in
favor of respondents and held that good faith may be raised as a defense in the case. Both parties
moved for reconsideration of the Gonzalez Resolution. The trial court granted the Motion to
Suspend Proceedings filed earlier by Dela Peña-Reyes and Manalastas. The Motion to Suspend
was granted.

ISSUE/S: Whether or not the Trial Court is correct in granting the Motion to Suspend.

HELD: The trial court should have proceeded with respondents Dela Peña-Reyes and
Manalastas’ arraignment after the 60-day period from the filing of the Petition for Review before
the Department of Justice.

The trial court erred when it did not act on the criminal case during the interim period. It had full
control and direction of the case.

The suspension of the arraignment should always be within the limits allowed by law. The
doctrine in Crespo v. Judge Mogul was reiterated in Mayor Balindong v. Court of Appeals where
this court reminded the Department of Justice Secretary to refrain from entertaining petitions for
review when the case is already pending with this court: In order to avoid a situation where the
opinion of the Secretary of Justice who reviewed the action of the fiscal may be disregarded by
the trial court, the Secretary of Justice should, as far as practicable, refrain from entertaining a
petition for review or appeal from the action of the fiscal, when the complaint or information has
already been filed in the Court. The matter should be left entirely for the determination of the
Court. The trial court should have proceeded with respondents Dela Peña-Reyes and Manalastas’
arraignment after the 60-day period from the filing of the Petition for Review before the
Department of Justice on March 8, 2005. It was only on September 13, 2010 that the temporary
restraining order was issued by the Court of Appeals. The trial court erred when it did not act on

104
ERDIE E. AMBROCIO Remedial Law Review 1
2014-0308 Criminal Procedure – Case Digests

the criminal case during the interim period. It had full control and direction of the case. As Judge
Mogul reasoned in denying the motion to dismiss in Crespo, failure to proceed with the
arraignment “disregards the requirements of due process and erodes the Court’s independence
and integrity.”

105
ERDIE E. AMBROCIO Remedial Law Review 1
2014-0308 Criminal Procedure – Case Digests

52. ENRILE VS PEOPLE

FACTS: The Office of the Ombudsman filed an Information for plunder against Enrile, Napoles,
et. al before the Sandiganbayan. Enrile responded by filing before the Sandiganbayan (1) an
urgent omnibus motion (motion to dismiss for lack of evidence on record to establish probable
cause and ad cautelam motion for bail), and (2) a supplemental opposition to issuance of warrant
of arrest and for dismissal of Information,. The Sandiganbayan heard both motions. Thereafter,
the prosecution filed a consolidated opposition to both motions.

The Sandiganbayan denied Enrile’s motions and ordered the issuance of warrants of arrest on the
plunder case against the accused. Soon after, Enrile received a notice of hearing informing him
that his arraignment would be held before the Sandiganbayan. The day before his arraignment,
Enrile filed a motion for bill of particular as well as a motion for deferment of arraignment since
he was to undergo medical examination at the PGH.

ISSUE/S: Whether or not the Sandiganbayan abused its discretion in denying Enrile’s motion
for bill of particulars;

HELD: YES. The grant or denial of a motion for bill of particulars is discretionary on the court
where the Information is filed. As usual in matters of discretion, the ruling of the trial court will
not be reversed unless grave abuse of discretion or a manifestly erroneous order amounting to
grave abuse of discretion is shown. In the light of all these considerations, we hold that the
Sandiganbayan’s denial of the petitioner’s motion for a bill of particulars, on the ground that the
details sought to be itemized or specified are all evidentiary without any explanation supporting
this conclusion constitutes grave abuse of discretion. Some of the desired details are material
facts that must be alleged to enable the petitioner to properly plead and prepare his defense. The
Sandiganbayan should have diligently sifted through each detail sought to be specified, and made
the necessary determination of whether each detail was an ultimate or evidentiary fact,
particularly after Enrile stated in his Reply that the “desired details” could not be found in the
bundle of documents marked by the prosecution. We cannot insist or speculate that he is feigning
ignorance of the presence of these desired details; neither can we put on him the burden of
unearthing from these voluminous documents what the desired details are. The remedy of a bill
of particulars is precisely made available by the Rules to enable an accused to positively respond
and make an intelligent defense.

A bill of particulars guards against the taking of an accused by surprise by restricting the scope
of the proof; it limits the evidence to be presented by the parties to the matters alleged in the
Information as supplemented by the bill. It is for this reason that the failure of an accused to
move for a bill of particulars deprives him of the right to object to evidence which could be
lawfully introduced and admitted under an information of more or less general terms which
sufficiently charges the defendants with a definite crime.

Sec, 9 Rule 116, Bill of particulars.—The accused may, before arraignment, move for a bill of
particulars to enable him properly to plead and prepare for trial. The motion shall specify the
alleged defects of the complaint or information and the details desired.

106
ERDIE E. AMBROCIO Remedial Law Review 1
2014-0308 Criminal Procedure – Case Digests

In general, a bill of particulars is the further specification of the charges or claims in an action,
which an accused may avail of by motion before arraignment, to enable him to properly plead
and prepare for trial. In civil proceedings, a bill of particulars has been defined as a
complementary procedural document consisting of an amplification or more particularized
outline of a pleading, and is in the nature of a more specific allegation of the facts recited in the
pleading. The purpose of a motion for bill of particulars in civil cases is to enable a party to
prepare his responsive pleading properly. In criminal cases, a bill of particulars details items or
specific conduct not recited in the Information but nonetheless pertain to or are included in the
crime charged. Its purpose is to enable an accused: to know the theory of the government’s case;
to prepare his defense and to avoid surprise at the trial; to plead his acquittal or conviction in bar
of another prosecution for the same offense; and to compel the prosecution to observe certain
limitations in offering evidence.

In dealing with a motion for a bill of particulars in a criminal case, judges should observe that:
(a) the remedy is distinct from a motion to quash in the sense that it presupposes that the acts or
offenses constituting the offense (or the ultimate facts that comprise the crime’s component
elements) are already stated in the Information, albeit may be couched in vague language; (b) the
remedy is, as mentioned, not meant to supply evidential information (or evidentiary facts); and
(c) the particulars to be allowed are only those details that would allow a man of ordinary
intelligence, as may be reasonable under the circumstances, to, again, properly plead during his
arraignment and to prepare his defense for trial. Accordingly, the analysis involved in motion for
bill of particulars should go beyond a simple ultimate facts-evidentiary facts dichotomy.

If the Information is lacking, a court should take a liberal attitude towards its granting and order
the government to file a bill of particulars elaborating on the charges. Doubts should be resolved
in favor of granting the bill to give full meaning to the accused’s Constitutionally guaranteed
rights. Notably, the government cannot put the accused in the position of disclosing certain overt
acts through the Information and withholding others subsequently discovered, all of which it
intends to prove at the trial. This is the type of surprise a bill of particulars is designed to avoid.
The accused is entitled to the observance of all the rules designated to bring about a fair verdict.
This becomes more relevant in the present case where the crime charged carries with it the
severe penalty of capital punishment and entails the commission of several predicate criminal
acts involving a great number of transactions spread over a considerable period of time.

107
ERDIE E. AMBROCIO Remedial Law Review 1
2014-0308 Criminal Procedure – Case Digests

53. PEOPLE VS LACSON

FACTS: Before the Court is the petitioners’ Motion for Reconsideration of the Resolution dated
May 28, 2002, remanding this case to the Regional Trial Court (RTC) of Quezon City, Branch
81, for the determination of several factual issues relative to the application of Section 8 of Rule
117 on the dismissal of Criminal Cases Nos. Q-99-81679 to Q-99-81689 filed against the
respondent and his co-accused with the said court. The respondent and his co-accused were
charged with multiple murder for the shooting and killing of eleven persons bandied as members
of the Kuratong Baleleng Gang. The respondent opposed petitioners’ motion for reconsideration.

The Court ruled in the Resolution sought to be reconsidered that the provisional dismissal of
Criminal Cases Nos. Q-99- 81679 to Q-99-81689 were with the express consent of the
respondent as he himself moved for said provisional dismissal when he filed his motion for
judicial determination of probable cause and for examination of witnesses. The Court also held
therein that although Section 8, Rule 117 of the Revised Rules of Criminal Procedure could be
given retroactive effect, there is still a need to determine whether the requirements for its
application are attendant.

ISSUE/S: Whether or not the requirements for valid motion to quash were attendant;

HELD: NO. The provisional dismissal of offenses punishable by imprisonment not exceeding
six (6) years or a fine of any amount, or both, shall become permanent one (1) year after issuance
of the order without the case having been revived. With respect to offenses punishable by
imprisonment of more than six (6) years, their provisional dismissal shall become permanent two
(2) years after issuance of the order without the case having been revived.

Having invoked said rule before the petitioners-panel of prosecutors and before the Court of
Appeals, the respondent is burdened to establish the essential requisites of the first paragraph
thereof, namely:
1. the prosecution with the express conformity of the accused or the accused moves for a
provisional (sin perjuicio) dismissal of the case; or both the prosecution and the accused
move for a provisional dismissal of the case;
2. the offended party is notified of the motion for a provisional dismissal of the case;
3. the court issues an order granting the motion and dismissing the case provisionally;
4. the public prosecutor is served with a copy of the order of provisional dismissal of the
case.

The foregoing requirements are conditions sine qua non to the application of the time-bar in the
second paragraph of the new rule. The raison d’ etre for the requirement of the express consent
of the accused to a provisional dismissal of a criminal case is to bar him from subsequently
asserting that the revival of the criminal case will place him in double jeopardy for the same
offense or for an offense necessarily included therein.

Although the second paragraph of the new rule states that the order of dismissal shall become
permanent one year after the issuance thereof without the case having been revived, the

108
ERDIE E. AMBROCIO Remedial Law Review 1
2014-0308 Criminal Procedure – Case Digests

provision should be construed to mean that the order of dismissal shall become permanent one
year after service of the order of dismissal on the public prosecutor who has control of the
prosecution6 without the criminal case having been revived. The public prosecutor cannot be
expected to comply with the timeline unless he is served with a copy of the order of dismissal

Express consent to a provisional dismissal is given either viva voce or in writing. It is a positive,
direct, unequivocal consent requiring no inference or implication to supply its meaning. Where
the accused writes on the motion of a prosecutor for a provisional dismissal of the case No
objection or With my conformity, the writing amounts to express consent of the accused to a
provisional dismissal of the case. The mere inaction or silence of the accused to a motion for a
provisional dismissal of the case or his failure to.

A motion of the accused for a provisional dismissal of a case is an express consent to such
provisional dismissal. If a criminal case is provisionally dismissed with the express consent of
the accused, the case may be revived only within the periods provided in the new rule. On the
other hand, if a criminal case is provisionally dismissed without the express consent of the
accused or over his objection, the new rule would not apply. The case may be revived or refiled
even beyond the prescribed periods subject to the right of the accused to oppose the same on the
ground of double jeopardy or that such revival or refiling is barred by the statute of limitations.

The case may be revived by the State within the time-bar either by the refiling of the Information
or by the filing of a new Information for the same offense or an offense necessarily included
therein. There would be no need of a new preliminary investigation. However, in a case wherein
after the provisional dismissal of a criminal case, the original witnesses of the prosecution or
some of them may have recanted their testimonies or may have died or may no longer be
available and new witnesses for the State have emerged, a new preliminary investigation must be
conducted before an Information is refiled or a new Information is filed.

In this case, the respondent has failed to prove that the first and second requisites of the first
paragraph of the new rule were present when Judge Agnir, Jr. dismissed Criminal Cases Nos. Q-
99-81679 to Q-99-81689. Irrefragably, the prosecution did not file any motion for the provisional
dismissal of the said criminal cases. For his part, the respondent merely filed a motion for
judicial determination of probable cause and for examination of prosecution witnesses alleging
that under Article III, Section 2 of the Constitution and the decision of this Court in Allado v.
Diokno, among other cases, there was a need for the trial court to conduct a personal
determination of probable cause for the issuance of a warrant of arrest against respondent and to
have the prosecution’s witnesses summoned before the court for its examination.

109
ERDIE E. AMBROCIO Remedial Law Review 1
2014-0308 Criminal Procedure – Case Digests

54. PANAGUITON VS DOJ

FACTS: Cawili borrowed money from petitioner. As payment for the loan, Cawili and Tongson
jointly issued three checks in favor of petitioner. The checks were dishonored, either for
insufficiency of funds or by closure of the account. Petitioner filed a complaint for violation of
BP 22. The prosecutor found probable cause only with respect to Cawili as Tongson’s defense
that his signatures on the checks had been falsified. Petitioner then filed a partial appeal before
the DOJ even while the case against Cawili was already filed in court. The Chief State
Prosecutor directed the city prosecutor to conduct a reinvestigation and to refer the falsified
document to the NBI. After reinvestigation, still only probable cause with respect to Cawili was
sustained. In the city prosecutor’s resolution, it was held that the case with respect to Tongson
had already prescribed pursuant to Act No. 3326 which provides that violations penalized by
B.P. Blg. 22 shall prescribe after four (4) years. n this case, the four (4)-year period started on the
date the checks were dishonored, or on 20 January 1993 and 18 March 1993. The filing of the
complaint before the Quezon City Prosecutor on 24 August 1995 did not interrupt the running of
the prescriptive period, as the law contemplates judicial, and not administrative proceedings.
Thus, considering that from 1993 to 1998, more than four (4) years had already elapsed and no
information had as yet been filed against Tongson, the alleged violation of B.P. Blg. 22 imputed
to him had already prescribed. Ultimately, the DOJ held that the action on the crime has
prescribed.

In justifying its resolution, the DOJ explained that Act No. 3326 applies to violations of special
acts that do not provide for a prescriptive period for the offenses thereunder. Since B.P. Blg. 22,
as a special act, does not provide for the prescription of the offense it defines and punishes, Act
No. 3326 applies to it, and not Art. 90 of the Revised Penal Code which governs the prescription
of offenses penalized thereunder.

ISSUE/S: WON the running of the prescriptive period tolled upon the filing of the information
in court or upon the filing of the complaint with the prosecutor for preliminary investigation.

HELD: Act No. 3326 is the law applicable to offenses under special laws which do not provide
their own prescriptive periods.

It must be pointed out that when Act No. 3326 was passed on 4 December 1926, preliminary
investigation of criminal offenses was conducted by justices of the peace, thus, the phraseology
in the law, "institution of judicial proceedings for its investigation and punishment," and the
prevailing rule at the time was that once a complaint is filed with the justice of the peace for
preliminary investigation, the prescription of the offense is halted. “the term ‘proceedings’
should now be understood either executive or judicial in character”. To rule otherwise would
deprive the injured party the right to obtain vindication on account of delays that are not under
his control.

110
ERDIE E. AMBROCIO Remedial Law Review 1
2014-0308 Criminal Procedure – Case Digests

55. PEOPLE VS DUMLAO

FACTS: An Amended Information was filed before the Sandiganbayan charging respondents
Dumlao and Lao, Aber P. Canlas, Jacobo C. Clave, Roman A. Cruz, Jr. and Fabian C. Ver with
violation of Section 3(g) of Republic Act No. 3019, as amended, otherwise known as the Anti-
Graft and Corrupt Practices Act.

Respondent Dumlao, with the assistance of counsel de parte, pleaded not guilty to the offense
charged. A Joint Stipulation of Facts and Admission of Exhibits was submitted to the court on 10
January 2005; after which the court issued Pre-Trial Order; Respondent Dumlao then filed a
Motion to Dismiss/Quash on the ground that the facts charged do not constitute an offense. He
argued that the allegedly approved Board Resolution was not in fact approved by the GSIS
Board of Trustees, contrary to the allegations in the information. Since the signatures of Fabian
Ver, Roman Cruz, Aber Canlas and Jacobo Clave did not appear in the minutes of the meeting
held on 23 April 1982, he said it was safe to conclude that these people did not participate in the
alleged approval of the LeasePurchase Agreement. This being the case, he maintained that there
was no quorum of the board to approve the supposed resolution authorizing the sale of the GSIS
property. There being no approval by the majority of the Board of Trustees, there can be no
resolution approving the Lease-Purchase Agreement. The unapproved resolution, he added,
proved his innocence. He further contended that the person to be charged should be Atty. Luis
Javellana, who sold the subject property to respondent Lao without the proper authority.
Sandiganbayan ruled in favor of respondent; the People through the Office of the Special
Prosecutor, filed a petition for certiorari under Rule 45 seeking the reversal and setting aside of
the Sandiganbayan Resolution dismissing the case against respondent Dumlao, arguing that it
was denied its right to due process when the court a quo dismissed the case against respondent
Dumlao after pre-trial and before it could present its witnesses and formally offer its exhibits.

ISSUE/S: WON facts alleged, if hypothetically admitted, would establish the essentials elements
of the crime defined by law.

HELD: Insufficiency of evidence is not one of the grounds of a Motion to Quash. Insufficiency
of evidence is a ground for dismissal of an action only after the prosecution rests its case. Section
23, Rule 119 of the Revised Rules of Criminal Procedure.

In the case under consideration, the Sandiganbayan dismissed the case against respondent for
insufficiency of evidence, even without giving the prosecution the opportunity to present its
evidence. In so doing, it violated the prosecutions right to due process. It deprived the
prosecution of its opportunity to prosecute its case and to prove the culpability of the accused.

It was therefore erroneous for the Sandiganbayan to dismiss the case under the premises. Not
only did it not consider the ground invoked by respondent Dumlao; it even dismissed the case on
a ground not raised by him, and not at the appropriate time. The dismissal was thus without basis
and untimely.

111
ERDIE E. AMBROCIO Remedial Law Review 1
2014-0308 Criminal Procedure – Case Digests

56. SORIANO VS PEOPLE

FACTS: State Prosecutor Josefino A. Subia charged Soriano in the Regional Trial Court (RTC)
of Malolos, Bulacan, with violation of Section 83 of Republic Act No. 337 (R.A. No. 337) or the
General Banking Act, as amended by Presidential Decree No. 1795, or Violation of the Director,
Officer, Stockholder or Related Interest (DOSRI) Rules (DOSRI Rules)

An information for estafa thru falsification of commercial document was also filed against
Soriano and Ilagan. The informations were docketed as Criminal Case Nos. 1719-M-2000 and
1720-M-2000, respectively, and were raffled to Branch 14, presided by Judge Petrita Braga
Dime.

Another information for violation of Section 83 of R.A. No. 337, as amended, was filed against
Soriano, this time, covering the P15,000,000.00 loan obtained in the name of Rogelio Mañaol.

Soriano and Ilagan were also indicted for estafa thru falsification of commercial document for
obtaining said loan. The cases were docketed as 1980-M-2000 and 1981-M-2000, respectively,
and were raffled to Branch 77, presided by Judge Aurora Santiago-Lagman.

Petitioners Soriano and Ilagan filed an MQ before both salas. Petitioners argued that the
prosecutor charged more than one offense for a single act. Soriano was charged with violation of
DOSRI rules and estafa thru falsification of commercial document for allegedly securing
fictitious loans. They further argued that the facts as alleged in the information do not constitute
an offense.

ISSUE/S: Whether the two judges correctly denied the Motion to Quash

HELD: YES. Petitioners assail the validity of the informations against them on the ground that
more than one (1) offense is charged. They point that Soriano was charged with violation of
DOSRI Rules and with estafa thru falsification of commercial document for allegedly obtaining
loans from RBSM. Thus, they claim that the informations were duplicitous; hence, they should
be quashed.

Indisputably, duplicity of offenses in a single information is a ground to quash the Information


under Section 3(e), Rule 11713 of the 1985 Rules of Criminal Procedure. The Rules prohibit the
filing of a duplicitous information to avoid confusing the accused in preparing his defense. By
duplicity of charges is meant a single complaint or information that charges more than one
offense.

Otherwise stated, there is duplicity (or multiplicity) of charges when a single Information
charges more than one offense.

In this case, however, Soriano was faced not with one information charging more than one
offense, but with more than one information, each charging a different offense - violation of
DOSRI rules in one, and estafa thru falsification of commercial documents in the others. Ilagan,

112
ERDIE E. AMBROCIO Remedial Law Review 1
2014-0308 Criminal Procedure – Case Digests

on the other hand, was charged with estafa thru falsification of commercial documents in
separate informations. Thus, petitioners erroneously invoke duplicity of charges as a ground to
quash the Informations.

Petitioners also contend that Soriano should be charged with one offense only, because all the
charges filed against him proceed from and are based on a single act of obtaining fictitious loans.
Thus, Soriano argues that he cannot be charged with estafa thru falsification of commercial
document, considering that he is already being prosecuted for obtaining a DOSRI loan.

The contention has no merit.

Jurisprudence teems with pronouncements that a single act or incident might offend two or more
entirely distinct and unrelated provisions of law, thus justifying the filing of several charges
against the accused.

In Loney v. People, this Court, in upholding the filing of multiple charges against the accused,
held: As early as the start of the last century, this Court had ruled that a single act or incident
might offend against two or more entirely distinct and unrelated provisions of law thus justifying
the prosecution of the accused for more than one offense. The only limit to this rule is the
Constitutional prohibition that no person shall be twice put in jeopardy of punishment for "the
same offense." In People v. Doriquez, we held that two (or more) offenses arising from the same
act are not "the same"

x x x if one provision [of law] requires proof of an additional fact or element which the other
does not, x x x. Phrased elsewise, where two different laws (or articles of the same code) define
two crimes, prior jeopardy as to one of them is no obstacle to a prosecution of the other, although
both offenses arise from the same facts, if each crime involves some important act which is not
an essential element of the other.

Consequently, the filing of the multiple charges against petitioners, although based on the same
incident, is consistent with settled doctrine.

As aptly pointed out by the BSP in its memorandum, there are differences between the two (2)
offenses. A DOSRI violation consists in the failure to observe and comply with procedural,
reportorial or ceiling requirements prescribed by law in the grant of a loan to a director, officer,
stockholder and other related interests in the bank, i.e. lack of written approval of the majority of
the directors of the bank and failure to enter such approval into corporate records and to transmit
a copy thereof to the BSP supervising department. The elements of abuse of confidence, deceit,
fraud or false pretenses, and damage, which are essential to the prosecution for estafa, are not
elements of a DOSRI violation. The filing of several charges against Soriano was, therefore,
proper.

Petitioners next question the sufficiency of the allegations in the informations, contending that
the same do not constitute an offense.

113
ERDIE E. AMBROCIO Remedial Law Review 1
2014-0308 Criminal Procedure – Case Digests

The fundamental test in considering a motion to quash anchored on Section 3 (a), Rule 117 of
the1985 Rules on Criminal Procedure, is the sufficiency of the averments in the information; that
is, whether the facts alleged, if hypothetically admitted, would establish the essential elements of
the offense charged as defined by law. The trial court may not consider a situation contrary to
that set forth in the criminal complaint or information. Facts that constitute the defense of the
petitioners against the charge under the information must be proved by them during trial. Such
facts or circumstances do not constitute proper grounds for a motion to quash the information on
the ground that the material averments do not constitute the offense.

We have reviewed the informations and find that they contain material allegations charging
Soriano with violation of DOSRI rules and estafa thru falsification of commercial documents.

In Criminal Case Nos. 1719 & 1980 for violation of DOSRI rules, the informations alleged that
Soriano was the president of RBSMI, while Ilagan was then its general manager; that during
their tenure, Soriano, with the direct participation of Ilagan, and by using the names of Virgilio
Malang and Rogelio Mañaol, was able to indirectly obtain loans without complying with the
requisite board approval, reportorial and ceiling requirements, in violation of Section 83 of R.A.
No. 37722 as amended.

Similarly, the informations in Criminal Case Nos. 1720 & 1981 charge petitioners with estafa
thru falsification of commercial document. They allege that petitioners made it appear that
Virgilio J. Malang and Rogelio Mañaol obtained loans and received the proceeds thereof when
they did not in fact secure said loans or receive the amounts reflected in the promissory notes and
other bank records.

The information in Criminal Case No. 1720 further alleges the elements of estafa under Article
315 (1)(b)23 of the RPC to wit: (i) that money, goods or other personal property be received by
the offender in trust, or on commission, or for administration, or under any other obligation
involving the duty to make delivery of or to return the same; (ii) that there be misappropriation
or conversion of such money or property by the offender, or denial on his part of such receipt;
(iii) that such misappropriation or conversion or denial is to the prejudice of another; and (iv)
that there is demand made by the offended party to the offender.

The information in Criminal Case No. 1981, on the other hand, further alleged the following
essential elements of estafa under Article 315 (2) (a)24 of the RPC: (i) that there must be a false
pretense, fraudulent act or fraudulent means; (ii) that such false pretense, fraudulent act or
fraudulent means must be made or executed prior to or simultaneously with the commission of
the fraud; (iii) that the offended party must have relied on the false pretense, fraudulent act, or
fraudulent means—that is, he was induced to part with his money or property because of the
false pretense, fraudulent act, or fraudulent means; and (iv) that, as a result thereof, the offended
party suffered damage. The informations in Criminal Case Nos. 1720 & 1981, thus, charge
petitioners with the complex crime of estafa thru falsification of commercial documents.

Verily, there is no justification for the quashal of the Information filed against petitioners. The
RTC committed no grave abuse of discretion in denying the motions.

114
ERDIE E. AMBROCIO Remedial Law Review 1
2014-0308 Criminal Procedure – Case Digests

In fine, the Court has consistently held that a special civil action for certiorari is not the proper
remedy to assail the denial of a motion to quash an information. The proper procedure in such a
case is for the accused to enter a plea, go to trial without prejudice on his part to present the
special defenses he had invoked in his motion to quash and if after trial on the merits, an adverse
decision is rendered, to appeal therefrom in the manner authorized by law.25 Thus, petitioners
should not have forthwith filed a special civil action for certiorari with the CA and instead, they
should have gone to trial and reiterated the special defenses contained in their motion to quash.
There are no special or exceptional circumstances in the present case that would justify
immediate resort to a filing of a petition for certiorari. Clearly, the CA did not commit any
reversible error, much less, grave abuse of discretion in dismissing the petition.

115
ERDIE E. AMBROCIO Remedial Law Review 1
2014-0308 Criminal Procedure – Case Digests

57. CEREZO VS PEOPLE

FACTS: In 2002, Joseph Cerezo filed a complaint for libel against Juliet Yaneza, Pablo Abunda,
Jr., and Vicente Afulugencia, as well as Oscar Mapalo (Mapalo). Finding probable cause to
indict Yaneza, et al., the Quezon City Prosecutor’s Office (OP-QC) filed the corresponding
Information against them before the RTC. Yaneza, et al. thereafter filed a Motion for
Reconsideration and/or Motion to Re-evaluate Prosecution’s Evidence before the OP-QC. In its
resolution, the OP-QC reversed its earlier finding and recommended the withdrawal of the
Information. Consequently, a Motion to Dismiss and Withdraw Information was filed before the
RTC.During the intervening period, Yaneza, et al. were arraigned. All of them entered a "not
guilty" plea. In deference to the prosecutor’s last resolution, the RTC ordered the criminal case
dismissed. Aggrieved, Cerezo moved for reconsideration of the said Order, arguing that the OP-
QC resolution has not yet attained finality, considering that the same was the subject of a Petition
for Review filed before the Department of Justice (DOJ). The RTC deferred action on the said
motion to await the resolution of the DOJ. In 2006, the Secretary of Justice promulgated his
resolution reversing and setting aside the OP-QC’s resolution, and directing the latter to refile the
earlier Information for libel. The RTC issued its Order granting Cerezo's motion for
reconsideration, conformably with the resolution of the DOJ Secretary. Yaneza, et al. moved for
reconsideration, but the motion was denied by the RTC. Relentless, Yaneza, et al. elevated their
predicament to the CA through a Petition for Certiorari under Rule 65 of the Rules of Court. The
CA annulled the impugned RTC Orders. Cerezo interposed the appeal when his motion for
reconsideration of the CA Decision was denied.

ISSUE/S:
1. Whether the dismissal of the case by the RTC was valid.
2. Whether double jeopardy set in after the RTC's dismissal of the case.

HELD:
1. No, because there was independent assessment of the merits of the motion on the part of the
RTC.
2. No. The elements of double jeopardy are not attendant in the present case.

Well-entrenched is the rule that once a case is filed with the court, any disposition of it rests on
the sound discretion of the court. In thus resolving a motion to dismiss a case or to withdraw an
Information, the trial court should not rely solely and merely on the findings of the public
prosecutor or the Secretary of Justice. It is the court’s bounden duty to assess independently the
merits of the motion, and this assessment must be embodied in a written order disposing of the
motion. While the recommendation of the prosecutor or the ruling of the Secretary of Justice is
persuasive, it is not binding on courts.

By relying solely on the manifestation of the public prosecutor and the resolution of the DOJ
Secretary, the trial court abdicated its judicial power and refused to perform a positive duty
enjoined by law. The said Orders were thus stained with grave abuse of discretion and violated
the complainant’s right to due process. They were void, had no legal standing, and produced no
effect whatsoever.

116
ERDIE E. AMBROCIO Remedial Law Review 1
2014-0308 Criminal Procedure – Case Digests

This Court must therefore remand the case to the RTC, so that the latter can rule on the merits of
the case to determine if a prima facie case exists and consequently resolve the Motion to Dismiss
and Withdraw Information anew.

It is beyond cavil that double jeopardy did not set in. Double jeopardy exists when the following
requisites are present: (1) a first jeopardy attached prior to the second; (2) the first jeopardy has
been validly terminated; and (3) a second jeopardy is for the same offense as in the first. A first
jeopardy attaches only (a) after a valid indictment; (b) before a competent court; (c) after
arraignment; (d) when a valid plea has been entered; and (e) when the accused has been acquitted
or convicted, or the case dismissed or otherwise terminated without his express consent.

Since we have held that the Order granting the motion to dismiss was committed with grave
abuse of discretion, then respondents were not acquitted nor was there a valid and legal dismissal
or termination of the case. Ergo, the fifth requisite which requires the conviction and acquittal of
the accused, or the dismissal of the case without the approval of the accused, was not met. Thus,
double jeopardy has not set in.

117
ERDIE E. AMBROCIO Remedial Law Review 1
2014-0308 Criminal Procedure – Case Digests

58. CO VS NEW PROSPERITY PLASTIC PRODUCTS

FACTS: Respondent New Prosperity Plastic Products, represented by Elizabeth Uy (Uy), is the
private complainant in Criminal Case Nos. 206655-59, 206661-77 and 209634 for Violation of
Batas Pambansa (B.P.) Bilang 22 filed against petitioner William Co (Co), which were raffled to
the MeTC Branch. 49 of Caloocan City.

In the absence of Uy and the private counsel, the cases were provisionally dismissed on June 9,
2003 in open court pursuant to Section 8, Rule 117 of the Revised Rules of Criminal Procedure
(Rules).

On July 2, 2004, Uy, through counsel, filed a Motion to Revive the Criminal Cases.9 Hon. Belen
B. Ortiz, then Presiding Judge of the MeTC Branch 49, granted the motion on October 14, 2004
and denied Co’s motion for reconsideration.

On March 17, 2005, Co filed a petition for certiorari and prohibition with prayer for the issuance
of a temporary restraining order (TRO)/writ of preliminary injunction (WPI) before the RTC of
Caloocan City challenging the revival of the criminal cases.

It was, however, dismissed for lack of merit on May 23, 2005. Co’s motion for reconsideration
was, subsequently, denied on December 16, 2005. Co then filed a petition for review on
certiorari under Rule 45 before the Supreme Court, which was docketed as G.R. No. 171096. We
dismissed the petition per Resolution dated February 13, 2006.There being no motion for
reconsideration filed, the dismissal became final and executory on March 20, 2006.

Before the MeTC Branch 50 where Criminal Case Nos. 206655-59, 206661-77 and 209634 were
re-raffled after the inhibition of Judge Ortiz, Co filed a “Motion for Permanent Dismissal” on
July 13, 2006. Uy opposed the motion, contending that the motion raised the same issues already
resolved with finality by this Court in G.R. No. 171096. In spite of this, Judge Esteban V.
Gonzaga issued an Order dated September 4, 2006 granting Co’s motion. When the court
subsequently denied Uy’s motion for reconsideration on November 16, 2006, Uy filed a petition
for certiorari before the RTC of Caloocan City. On January 28, 2008, Hon. Judge Adoracion G.
Angeles of the RTC Branch 121 acted favorably on the petition, annulling and setting aside the
Orders dated September 4, 2006 and November 16, 2006 and directing the MeTC Branch 50 to
proceed with the trial of the criminal cases.Co then filed a petition for certiorari before the CA,
which, as aforesaid, dismissed the petition and denied his motion for reconsideration. Hence, this
present petition with prayer for TRO/WPI.

Petitioner’s contention: Co argues that the June 9, 2003 Order provisionally dismissing Criminal
Case Nos. 206655-59, 206661-77 and 209634 should be considered as a final dismissal on the
ground that his right to speedy trial was denied. He reasons out that from his arraignment on
March 4, 2002 until the initial trial on June 9, 2003, there was already a “vexatious, capricious
and oppressive” delay, which is in violation of Section 6 of Republic Act 8493 (Speedy Trial Act
of 1998) and Section 2, Paragraph 2, Rule 119 of the Revised Rules of Criminal Procedure
mandating that the entire trial period should not exceed 180 days from the first day of trial. As

118
ERDIE E. AMBROCIO Remedial Law Review 1
2014-0308 Criminal Procedure – Case Digests

the dismissal is deemed final, Co contends that the MeTC lost its jurisdiction over the cases and
cannot reacquire jurisdiction over the same based on a mere motion because its revival would
already put him in double jeopardy.

ISSUE/S: Whether or not the dismissal of the criminal cases against petitioner on the ground of
denial of his right to speedy trial constitutes final dismissal of these cases;

HELD: No. Co’s charge that his right to a speedy trial was violated is baseless. Obviously, he
failed to show any evidence that the alleged “vexatious, capricious and oppressive” delay in the
trial was attended with malice or that the same was made without good cause or justifiable
motive on the part of the prosecution. This Court has emphasized that “‘speedy trial’ is a relative
term and necessarily a flexible concept.”In determining whether the accused's right to speedy
trial was violated, the delay should be considered in view of the entirety of the proceedings.The
factors to balance are the following: (a) duration of the delay; (b) reason therefor; (c) assertion of
the right or failure to assert it; and (d) prejudice caused by such delay. Surely, mere mathematical
reckoning of the time involved would not suffice as the realities of everyday life must be
regarded in judicial proceedings which, after all, do not exist in a vacuum, and that particular
regard must be given to the facts and circumstances peculiar to each case. “While the Court
recognizes the accused's right to speedy trial and adheres to a policy of speedy administration of
justice, we cannot deprive the State of a reasonable opportunity to fairly prosecute criminals.
Unjustified postponements which prolong the trial for an unreasonable length of time are what
offend the right of the accused to speedy trial.”

119
ERDIE E. AMBROCIO Remedial Law Review 1
2014-0308 Criminal Procedure – Case Digests

59. PEOPLE VS DE LEON

FACTS: Rodante de Leon was charged with violation of RA 9165 (Comprehensive Dangerous
Drugs Act of 2002). A buy-bust team was formed to act on the illegal activities of De Leon upon
the report of a confidential informant at the AntiIllegal Drug Special Operation Task Force at the
Novaliches Police Station in Quezon City. De Leon was arrested upon selling to PO2
Magcalayo, posing as buyer, shabu for P200. The seized drug was identified as a prohibited drug
and subsequently presented in evidence.

The RTC and CA both found De Leon guilty of the crime charged.

ISSUE/S: Whether or not the buy-bust operation, allegedly full of irregularities was legal.

HELD: YES. A buy-bust operation is a form of entrapment whereby ways and means are
resorted to for the purpose of trapping and capturing the lawbreakers in the execution of their
criminal plan. In this jurisdiction, the operation is legal and has been proved to be an effective
method of apprehending drug peddlers, provided due regard to constitutional and legal
safeguards is undertaken.

The Court upheld the presumption of regularity in the performance of official duties and ruled
that the elements of the crime were sufficiently established, thereby affirming the decision of the
CA.

120
ERDIE E. AMBROCIO Remedial Law Review 1
2014-0308 Criminal Procedure – Case Digests

60. SALVANERA VS. PEOPLE

FACTS: Petitioner Rimberto Salvanera, together with Feliciano Abutin, Edgardo Lungcay and
Domingo Tampelix, is charged with the murder of Ruben Parane, by feloniously assaulting and
attacking Parane using a firearm causing his death. Petitioner was the alleged mastermind;
Lungcay, the hired hitman; Abutin, the driver of the motorcycle which carried Lungcay to the
place of the commission of the crime; while Tampelix delivered the blood money to the latter.
All the accused have been arrested and detained, except Edgardo Lungcay who remained at-
large. Respondent Lucita Parane is the spouse of victim Ruben Parane.

Petitioner applied for bail. The prosecution, moved for the discharge of accused Abutin and
Tampelix, to serve as state witnesses. In an order, the trial court granted petitioner’s application
for bail and denied the prosecution’s motion for the discharge of accused Abutin and Tampelix.
The prosecution moved for reconsideration but the motion was denied.

The prosecution then appealed to the Court of Appeals. It contended that the trial court
committed grave abuse of discretion when it denied the motion to discharge accused Abutin and
Tampelix to be state witnesses. It alleged that the testimonies of the two accused are absolutely
necessary to establish that petitioner masterminded the murder of Ruben Parane. The prosecution
likewise claimed that it was premature and baseless for the trial court to grant petitioner’s
application for bail because the prosecution had not yet rested its case in the hearing for the
discharge of the two accused.

The Court of Appeals sustained the prosecution. It discharged accused Abutin and Tampelix
from the Information to become state witnesses, and cancelled the bail bond of petitioner
Salvanera. The Court of Appeals denied the Motion for Reconsideration filed by the petitioner.

ISSUE/S: Whether or not there was substantial corroboration of Abutin and Tampelix’s
testimony to discharge them and serve as State Witnesses.

HELD: YES. In the discharge of an accused in order that he may be a state witness, the
following conditions must be present, namely:
1. Two or more accused are jointly charged with the commission of an offense;
2. The motion for discharge is filed by the prosecution before it rests its case;
3. The prosecution is required to present evidence and the sworn statement of each proposed
state witness at a hearing in support of the discharge;
4. The accused gives his consent to be a state witness; and
5. The trial court is satisfied that:
a. There is absolute necessity for the testimony of the accused whose discharge is
requested;
b. There is no other direct evidence available for the proper prosecution of the
offense committed, except the testimony of said accused;
c. The testimony of said accused can be substantially corroborated in its material
points;
d. Said accused does not appear to be the most guilty; and,

121
ERDIE E. AMBROCIO Remedial Law Review 1
2014-0308 Criminal Procedure – Case Digests

e. Said accused has not at any time been convicted of any offense involving moral
turpitude.

To require Parane and Salazar to corroborate the testimony of Abutin and Tampelix on the exact
same points is to render nugatory the other requisite that "there must be no other direct evidence
available for the proper prosecution of the offense committed, except the testimony of the state
witness." The corroborative evidence required by the Rules does not have to consist of the very
same evidence as will be testified on by the proposed state witnesses. The Supreme Court ruled
that a conspiracy is more readily proved by the acts of a fellow criminal than by any other
method. If it is shown that the statements of the conspirator are corroborated by other evidence,
then we have convincing proof of veracity. Even if the confirmatory testimony only applies to
some particulars, we can properly infer that the witness has told the truth in other respects. It is
enough that the testimony of a co-conspirator is corroborated by some other witness or evidence.
In the case at bar, we are satisfied from a reading of the records that the testimonies of Abutin
and Tampelix are corroborated on important points by each other’s testimonies and the
circumstances disclosed through the testimonies of the other prosecution witnesses, and "to such
extent that their trustworthiness becomes manifest."

As co-conspirators, Abutin and Tampelix can testify on the criminal plan of the conspirators.
Where a crime is contrived in secret, the discharge of one of the conspirators is essential because
only they have knowledge of the crime. The other prosecution witnesses are not eyewitnesses to
the crime, as, in fact, there is none. No one except the conspirators knew and witnessed the
murder. The testimonies of the accused and proposed state witnesses Abutin and Tampelix can
directly link petitioner to the commission of the crime.

122
ERDIE E. AMBROCIO Remedial Law Review 1
2014-0308 Criminal Procedure – Case Digests

61. VDA. DE MANGUERRA VS. RISOS

FACTS: Respondents, Risos et.al. were charged with Estafa before the RTC of Cebu City which
arose from the falsification of a deed of real estate mortgage allegedly committed by respondents
where they made it appear that petitioner De Manguerra, the owner of the mortgaged property
affixed her signature to the document. Petitioner was then confined in a hospital in Manila for
gastro-intestinal bleeding. Respondents filed a motion for suspension of criminal proceedings on
the ground of a prejudicial question alleging that the action for the declaration of nullity of
mortgage should first be resolved. The RTC granted the motion. Petitioner filed a special civil
action for certiorari with the CA to nullify the RTC orders.

Petitioner’s counsel filed a motion to take her deposition due to petitioner’s weak physical
condition. The RTC granted the motion and ordered petitioner’s deposition to be taken before the
Clerk of Court of Makati. Respondents assailed the order of the RTC granting the motion to take
petitioner’s deposition but the RTC denied the MR stating that “procedural technicalities should
be brushed aside due to the urgency of the situation.” Respondents assailed the RTC orders
before the CA, with the CA ruling in their favor stating that Rule 119, Section 15 and not Rule
23 applies in this case.

ISSUE/S: Whether or not Rule 23 applies to the deposition of De Manguerra.

HELD: NO. It is basic that all witnesses shall give their testimonies at the trial of the case in the
presence of the judge. This is especially true in criminal cases in order that the accused may be
afforded the opportunity to cross-examine the witnesses pursuant to his constitutional right to
confront the witnesses face to face. It also gives the parties and their counsel the chance to
propound such questions as they deem material and necessary to support their position or to test
the credibility of said witnesses. Lastly, this rule enables the judge to observe the witnesses’
demeanor.

This rule, however, is not absolute. As exceptions, Rules 23 to 28 of the Rules of Court provide
for the different modes of discovery that may be resorted to by a party to an action. These rules
are adopted either to perpetuate the testimonies of witnesses or as modes of discovery. In
criminal proceedings, Sections 12, 13 and 15, Rule 119 of the Revised Rules of Criminal
Procedure, which took effect on December 1, 2000, allow the conditional examination of both
the defense and prosecution witnesses.

It is thus required that the conditional examination be made before the court where the case is
pending. It is also necessary that the accused be notified, so that he can attend the examination,
subject to his right to waive the same after reasonable notice. As to the manner of examination,
the Rules mandate that it be conducted in the same manner as an examination during trial, that is,
through question and answer.

123
ERDIE E. AMBROCIO Remedial Law Review 1
2014-0308 Criminal Procedure – Case Digests

62. PEOPLE VS. JIMENEZ

FACTS: Montero, a former employee of the BSJ company owned by the Jimenezes executed
sworn statements confessing his participation in the killing of Ruby Rose Barrameda and naming
petitioner Jimenez, Lope Jimenez, Descalso, Ponce and Fernandez as his coconspirators. The
statements of Montero led to the recovery of a cadaver encased in a drum near or practically at
the place Montero pointed to.

The state prosecutors charged the accused of murder for the killing of Barrameda. Montero
thereafter filed a motion for his discharge as state witness. Petitioner opposed the motion. The
RTC denied the motion to discharge Montero as state witness ruling that the prosecution failed to
comply with the requirements of Sec. 19, Rule 119 of the ROC. This was however reversed by
the newly-appointed regular Judge Docena ruling that the prosecution complied with the
requisites of Sec. 17, Rule 119 further ruling that Montero is qualified to be discharged as state
witness. Petitioner filed with the CA petition for certiorari seeking the annulment of Judge
Docena’s orders. The CA initially ruled in favor of petitioner but on MR, reversed its ruling in
favor of respondents agreeing with Judge Docena’s decision. Upon denial of petitioner’s MR,
petitioners filed the consolidated petitions for review on certiorari before the SC.

ISSUE/S: Whether or not Montero’s discharge as a state witness complied with the requirements
of Sec. 17, Rule 119 of the ROC.

HELD: YES. The requirement is that a state witness does not need to be found to be the least
guilty; he or she should not only "appear to be the most guilty."

By jurisprudence, "most guilty" refers to the highest degree of culpability in terms of


participation in the commission of the offense and does not necessarily mean the severity of the
penalty imposed. While all the accused may be given the same penalty due to conspiracy, yet one
may be considered to have lesser or the least guilt taking into account his degree of participation
in the commission of the offense. What the rule avoids is the possibility that the most guilty
would be set free while his co-accused who are less guilty in terms of participation would be
penalized.

Thus, as a rule, for purposes of resolving a motion to discharge an accused as a state witness,
what are controlling are the specific acts of the accused in relation to the crime committed.

The SC ruled that from the evidence submitted by the prosecution in support of its motion to
discharge Montero, it appears that while Montero was part of the planning, preparation, and
execution stage as most of his co-accused had been, he had no direct participation in the actual
killing of Ruby Rose. While Lope allegedly assigned to him the execution of the killing, the
records do not indicate that he had active participation in hatching the plan to kill Ruby Rose,
which allegedly came from accused Lope and Jimenez, and in the actual killing of Ruby Rose
which was executed by accused Lennard. Montero’s participation was limited to providing the
steel box where the drum containing the victim’s body was placed, welding the steel box to seal

124
ERDIE E. AMBROCIO Remedial Law Review 1
2014-0308 Criminal Procedure – Case Digests

the cadaver inside, operating the skip or tug boat, and, together with his co-accused, dropping the
steel box containing the cadaver into the sea.

The discharge of an accused to be utilized as a state witness because he does not appear to be the
most guilty is highly factual in nature as it largely depends on the appreciation of who had the
most participation in the commission of the crime. The appellate courts do not interfere in the
discretionary judgment of the trial court on this factual issue except when grave abuse of
discretion intervenes.

125
ERDIE E. AMBROCIO Remedial Law Review 1
2014-0308 Criminal Procedure – Case Digests

63. PEOPLE VS. DE GRANO

FACTS: An Information for the murder of Emmanuel Mendoza was filed with the RTC of
Tanauan Batangas against Joven de Grano, Armando de Grano, and Estanislao Lacaba, together
with their co-accused Leonides Landicho, Domingo Landicho, and Leonardo Genil, who were
at-large. Duly arraigned, Joven, Armando, and Estanislao pleaded “not guilty.”

Respondents filed a motion for bail contending that the prosecution’s evidence was not strong.
RTC found the accused guilty of the offenses charged. An order was issued that modified the
previous decision, which downgraded the charge of murder to homicide.

However, Joven, Armando, and Domingo were not present during promulgation. Petitioner
maintained that while they were not present during the promulgation of the RTC Decision,
Estanislao, who was under police custody, attended the promulgation. Therefore, when they filed
their Joint Motion for Reconsideration which included that of Estanislao, the RTC was not
deprived of its authority to resolve the joint motion.

ISSUE/S: Whether or not the RTC erred in taking cognizance of the joint motion for
reconsideration despite the absence of the other accused during the promulgation of judgment.

HELD: The RTC erred in taking cognizance of the joint motion for reconsideration despite the
absence of the other accused during the promulgation of judgment. Section 14(2), Article III of
the Constitution, authorizing trials in absentia, allows the accused to be absent at the trial but not
at certain stages of the proceedings, to wit: (a) at arraignment and plea, whether of innocence or
of guilt; (b) during trial, whenever necessary for identification purposes; and (c) at the
promulgation of sentence, unless it is for a light offense, in which case, the accused may appear
by counsel or representative. At such stages of the proceedings, his presence is required and
cannot be waived.

When the Decision was promulgated, only Estanislao was present. Subsequently thereafter,
without surrendering and explaining the reasons for their absence, Joven, Armando, and
Domingo joined Estanislao in their Joint Motion for Reconsideration. In blatant disregard of the
Rules, RTC not only failed to cause the arrest of the respondents who were at large, it also took
cognizance of the joint motion.

RTC clearly exceeded its jurisdiction when it entertained the joint MR with respect to the
respondents who were at large. It should have considered the joint motion as a MR that was
solely filed by Estanislao. Being at large, Joven and Domingo have not regained their standing in
court. Once an accused jumps bail or flees to a foreign country, or escapes from prison or
confinement, he loses his standing in court; and unless he surrenders or submits to the
jurisdiction of the court, he is deemed to have waived any right to seek relief from the court.

126
ERDIE E. AMBROCIO Remedial Law Review 1
2014-0308 Criminal Procedure – Case Digests

64. ASISTIO VS. PEOPLE

FACTS: Petitioner Jocelyn Asistio was charged with violation of Section 46 of the Cooperative
Code of the Philippines (Republic Act No. [RA] 6938). The accused being the Chairperson and
Managing Director of A. Mabini Elementary School Teachers Multi-Purpose Cooperative
entered into a contract with Coca Cola Products in her own personal capacity when in truth and
in fact as the said accused fully well knew, the sale of Coca-Cola products at A. Mabini
Elementary School Teachers Multi-Purpose Cooperative should have accrued to Cooperative to
the damage and prejudice of the said Cooperative.

The school principal directed petitioner to submit her financial reports during her tenure as
Chairperson. Instead, petitioner claimed that the principal had no business and authority to
require her to produce financial statements, and that the said reports had been posted on the
school bulletin board. The principal ordered to conduct an auditing. Based on the documents
obtained from Coca-Cola, including the records of actual deliveries and sales, and the financial
statements prepared by petitioner, the audit committee found that petitioner defrauded the
Cooperative and its members for 3 years.

After the presentation and offer of evidence by the prosecution, petitioner moved to dismiss the
case by way of Demurrer to Evidence with prior leave of court. She argued, among other
matters, that the Regional Trial Court (RTC) of Manila, Branch 40, does not have jurisdiction
over the case, as the crime charged (Violation of Section 46 of RA 6938) does not carry with it a
sanction for which she can be held criminally liable.

Subsequently, the RTC dismissed the case for lack of jurisdiction. The private respondent
appealed the decision of the RTC to the Court of Appeals which was granted. Hence this
petition.

ISSUE/S:
1. Whether or not the dismissal of the charge against petitioner on demurrer to evidence
amounts to an acquittal, hence final and cannot be appealed.
2. Whether or not the right of the accused against double jeopardy has been violated.

HELD: NO. In this case, the RTC granted the demurrer to evidence and dismissed the case not
for insufficiency of evidence, but for lack of jurisdiction over the offense charged. Notably, the
RTC did not decide the case on the merits, let alone resolve the issue of petitioner's guilt or
innocence based on the evidence proffered by the prosecution. This being the case the RTC order
of dismissal does not operate as an acquittal hence may still be subject to an appeal under Rule
41 of the ROC. “Acquittal is always based on the merits, that is, the defendant is acquitted
because the evidence does not show that the defendant's guilt is beyond reasonable doubt; but
dismissal does not decide the case on the merits or that the defendant is not guilty. Dismissal
terminates the proceeding, either because the court is not a court of competent jurisdiction, or the
evidence does not show that the offense was committed within the territorial jurisdiction of the
court, or the complaint or information is not valid or sufficient in form and substance, etc."

127
ERDIE E. AMBROCIO Remedial Law Review 1
2014-0308 Criminal Procedure – Case Digests

NO. The accused-appellee cannot also contend that she will be placed in double jeopardy upon
this appeal. It must be stressed that the dismissal of the case against her was premised upon her
filing of a demurrer to evidence, and the finding, albeit erroneous, of the trial court that it is
bereft of jurisdiction. The requisites that must be present for double jeopardy to attach are: (a) a
valid complaint or information; (b) a court of competent jurisdiction; (c) the accused has pleaded
to the charge; and (d) the accused has been convicted or acquitted or the case dismissed or
terminated without the express consent of the accused. There is no double jeopardy in this case
as the dismissal was with the accused-appellee's consent, that is, by moving for the dismissal of
the case through a demurrer to evidence he is deemed to have waived his protection against
double jeopardy.

128
ERDIE E. AMBROCIO Remedial Law Review 1
2014-0308 Criminal Procedure – Case Digests

65. CABRADOR VS. PEOPLE

FACTS: The public prosecutor accused Cabador of murdering, in conspiracy with others, Atty.
Valerio. After 5 years of trial and 5 witnesses, RTC ended the prosecution's presentation of
witnesses and required the prosecution to make a written or formal offer of its documentary
evidence within 15 days from notice. The public prosecutor asked for 3 extensions of time but
was still not able to make the offer.

Cabador filed a motion to dismiss the case, invoking his right to a speedy trial. Moreover, he
claimed that the trial court could not consider any evidence against him that had not been
formally offered. 4 days before such filing, without the knowledge of Cabador, another extension
was asked for, and an offer was made on the day Cabador filed his MTD.

RTC issued an order treating Cabador's MTD as a demurrer to evidence. And since he filed his
motion without leave of court, he waived his right to present evidence in his defense. RTC
submitted the case for decision. Cabador filed a MR which the RTC denied. CA likewise denied
his petition and affirmed RTC. CA denied his MR.

ISSUE/S: Whether or not the motion filed by Cabador was demurrer to evidence or a motion to
dismiss.

HELD: The SC ruled that Cabador filed a motion to dismiss, not a demurrer to evidence. He did
not waive his right to present evidence. There are 2 stages in the trial proper of a criminal case:
(1) prosecution's presentation of evidence against the accused; and (2) accused's presentation of
evidence in his defense. If after the first stage, the evidence appears insufficient to support a
conviction, the trial court may at its own initiative or on motion of the accused dispense with the
second stage, and dismiss the criminal action. There is no point for the trial court to hear the
evidence of the accused in such a case since the prosecution bears the burden of proving his guilt
beyond reasonable doubt. The order of dismissal amounts to an acquittal.

But because some have in the past used the demurrer in order to delay the proceedings in the
case, the remedy now carries a caveat. When the accused files a demurrer without leave of court,
he shall be deemed to have waived the right to present evidence and the case shall be considered
submitted for judgment.

To determine whether the pleading filed is a demurer to evidence or a motion to dismiss, the
Court must consider the ff: the allegations in it were made in good faith; the stage of the
proceeding at which it is filed; the primary objective of the party filing it.

Cabador in his MTD pointed out how the trial dragged on for years; the gaps between hearings
were long, with hearings often postponed due to the absence of the prosecutor. And it was
compounded by the repeated motions for extension by the prosecutor to make the formal offer,
and its failure to make such offer. He invoked his right to speedy trial.

129
ERDIE E. AMBROCIO Remedial Law Review 1
2014-0308 Criminal Procedure – Case Digests

In criminal cases, a motion to dismiss may be filed on the ground of denial of the accused's right
to speedy trail. This denial is characterized by unreasonable, vexatious, and oppressive delays
without fault of the accused, or by unjustified postponements that unreasonably prolonged the
trial. This was the main thrust of Cabador’s motion to dismiss and he had the right to bring this
up for a ruling by the trial court.

Cabador did not state what evidence the prosecution had presented against him to show in what
respects such evidence failed to meet the elements of the crime, something that is fundamental in
ay demurrer. It did not touch on any particular testimony or documentary exhibit. He could not
do so because he did not know that the prosecution finally made its offer of exhibits on the same
date he filed his MTD.

A demurrer assumes that the prosecution has already rested its case. In this case, after the
prosecution filed its formal offer, the trial court still needed to give Cabador an opportunity to
object to the admission of those exhibits. It also needed to rule on the formal offer. And only
after such a ruling could the prosecution be deemed to have rested its case. The MTD was filed
before he could object to the offer, before the trial court could rule on the offer, and before the
prosecution could rest its case. It cannot be said that he intended his MTD to serve as a demurrer.
He cannot be declared to have waived his right to present evidence in his defense.

130
ERDIE E. AMBROCIO Remedial Law Review 1
2014-0308 Criminal Procedure – Case Digests

66. PEOPLE VS. TAN

FACTS: Information for violation of Rule 36 (a)-1, in relation to Sections 32 (a)-1 and 56 of the
Revised Securities Act were filed against Tan in the RTC. After arraignment, he pleaded not
guilty to both charges and trial commenced. He made his formal offer of evidence. RTC issued
an Order admitting some (Exhibits A, B, W and X) but denied admission of all the other exhibits.
Tan filed a MR but this was denied by the RTC. He filed an omnibus motion for leave to file
demurrer to evidence and to admit the attached demurrer to evidence. RTC granted the motion
for leave to file the demurrer and admitted the attached demurrer. The RTC issued an Order
granting the respondent’s demurrer to evidence.

Petitioner filed a petition for certiorari with the CA assailing the Orders of the RTC. CA denied
the petition, ruling that the dismissal of a criminal action by the grant of a demurrer to evidence
is one on the merits and operates as an acquittal, for which reason, the prosecution cannot appeal
therefrom as it would place the accused in double jeopardy.

ISSUE/S: Whether or not double jeopardy applies in this case.

HELD: Double jeopardy applies. In People v. Sandiganbayan, this Court explained the general
rule that the grant of a demurrer to evidence operates as an acquittal and is, thus, final and cannot
be appealed.

The elements of double jeopardy are (1) the complaint or information was sufficient in form and
substance to sustain a conviction; (2) the court had jurisdiction; (3) the accused had been
arraigned and had pleaded; and (4) the accused was convicted or acquitted, or the case was
dismissed without his express consent.

The rule on double jeopardy, however, is not without exceptions. In People v. Laguio, Jr., this
Court stated that the only instance when double jeopardy will not attach is when the RTC acted
with grave abuse of discretion.

This exception is inapplicable to the factual milieu herein. RTC did not abuse its discretion in the
manner it conducted the proceedings of the trial, as well as its grant of respondent’s demurrer to
evidence.

Thus, the question to be resolved, given the factual molding of herein petition, is "did the RTC
violate petitioner’s right to due process?" Petitioner was given more than ample opportunity to
present its case as gleaned from the factual antecedents which led to the grant of respondent’s
demurrer.

Lastly, even if this Court were to review the action taken by the RTC in granting the demurrer to
evidence, no grave abuse can be attributed to it as it appears that the 29-page Order granting the
demurrer was arrived at after due consideration of the merits thereto. As correctly observed by
the CA, RTC extensively discussed its position on the various issues brought to contention by
petitioner. One of the main reasons for the RTC’s decision to grant the demurrer was the absence

131
ERDIE E. AMBROCIO Remedial Law Review 1
2014-0308 Criminal Procedure – Case Digests

of evidence to prove the classes of shares that the Best World Resources Corporation stocks were
divided into, whether there are preferred shares as well as common shares, or even which type of
shares respondent had acquired.

Withal, it bears to stress that the fundamental philosophy behind the constitutional proscription
against double jeopardy is to afford the defendant, who has been acquitted, final repose and
safeguard him from government oppression through the abuse of criminal processes.

132
ERDIE E. AMBROCIO Remedial Law Review 1
2014-0308 Criminal Procedure – Case Digests

67. IMPERIAL VS. JOSON

FACTS: A truck, owned by petitioner Imperial and being driven by petitioner Francisco,
collided with a truck, owned by respondent. After colliding with respondent’s truck, Imperial’s
truck rammed into a KIA Besta Van killing the driver and 7 of its passengers, and inflicting
serious physical injuries upon 3 other passengers. A criminal complaint for Reckless.

Imprudence resulting to Multiple Homicide, Multiple Serious Physical Injuries and Damages to
Property was filed against petitioners with the MTC.

Francisco proposed a series of facts for stipulation with the prosecution. Prosecutor Zabella
refused to stipulate. Court issued pre-trial order to that effect. Francisco filed motion to “compel
and disqualify Prosecutor Zabella and to correct the pre-trial order”, on the ground, among
others, that the latter cannot refuse to stipulate on matters which he has personal knowledge.
MTC denied motion. However, it directed another pre-trial conference in view of the
reassignment of the case to another prosecutor, and appearance of a new private prosecutor.

Francisco filed petition for certiorari, prohibition and mandamus with the RTC. He further
moved for the dismissal of the case on the ground that his right to speedy trial was violated in
view of the 9 postponements of the pre-trial conference. This was denied, so again he filed a
CPM petition with the RTC, which was consolidated with the previous CPM petition. Both CPM
petitions were dismissed for lack of merit. Francisco elevated this to the CA. CA upheld the
RTC.

ISSUE/S: Whether or not double jeopardy applies in this case.

HELD: The accused’s right to speedy trial was not violated. Designed to prevent the oppression
of the citizen by holding criminal prosecution suspended over him for an indefinite time and to
prevent delays in the administration of justice, said right is considered violated only when the
proceeding is attended by vexatious, capricious and oppressive delays.

133
ERDIE E. AMBROCIO Remedial Law Review 1
2014-0308 Criminal Procedure – Case Digests

68. PEOPLE VS. SANDIGANBAYAN

FACTS: Vice Mayor Manuel Barcenas of Toledo City in Cebu failed to liquidate cash advances
P62, 765.00 despite demands to the damage and prejudice of government. He was charged with
violation of Sec. 89 PD 1445 before Sandiganbayan. He was arraigned for which he pleaded not
guilty. Prosecution presented lone witness COA State Auditor Villad. Thereafter, prosecution
filed its formal offer of evidence and rested its case. Barcenas filed motion for leave to file
demurrer to evidence. Sandiganbayan granted the motion on ground that prosecution failed to
prove that government suffered any damage from Barcenas’ non-liquidation of subject cash
advance.

ISSUE: W/N Sandiganbayan acted with grave abuse of discretion.

HELD: No. Actual damage to government arising from non-liquidation of cash advance is not
an essential element. Instead, mere failure to timely liquidate is the gravamen of the offense.
Even if Sandiganbayan proceeded from an erroneous interpretation of law, the error committed
was an error of judgment and not of jurisdiction. The error committed is of such nature that can
no longer be rectified on appeal by prosecution because it would place accussed in double
jeopardy. Such error cannot be corrected because double jeopardy had already set in.

134
ERDIE E. AMBROCIO Remedial Law Review 1
2014-0308 Criminal Procedure – Case Digests

69. BANGAYAN, JR. VS. BANGAYAN

FACTS: Sally Go filed complaint accusing Bangayan, Jr and Resally having committed bigamy.
Sally Go learned that Bangayan, Jr had taken Resally as is concubine. Sally Go further
discovered that Bangayan Jr also married a certain Azucena in Caloocan City. After arraignment,
both Bangayan Jr and Resally pleaded not guilty. Prosecution presented and offered evidence.
Both Bangayan Jr and Resally separately filed their respective motions for leave to file demurrer
to evidence. RTC dismissed case for insufficiency of evidence. Sally Go elevated case to CA via
petition for certiorari. CA held that the pieces of evidence presented by prosecution were
sufficient to deny demurrer to evidence.

ISSUE: W/N Petitioners’ right against double jeopardy was violated by CA when it reversed
RTC order dismissing criminal case against them.

HELD: Yes. Double jeopardy already set in. All 4 elements of double jeopardy are present. A
valid information for bigamy was filed against petitioners. They pleaded not guilty to charges
against them and subsequently case was dismissed after the prosecution rested its case. If the
court finds that the evidence is not sufficient and grants demurrer, such dismissal of the case is
one on the merits, which is equivalent to acquittal of the accused. Even if trial court had
incorrectly overlooked the evidence against petitioners, it only committed an error of judgment
and not one of jurisdiction, which could not be rectified by certiorari. It must be shown that a
party was deprived of his opportunity to be heard.

135
ERDIE E. AMBROCIO Remedial Law Review 1
2014-0308 Criminal Procedure – Case Digests

70. PEOPLE VS. JOSE GO

FACTS: Fictitious loans in favor of two entities – Timmy’s Inc and Asia Textile Mills, Inc –
were approved, after which two manager’s checks representing the supposed proceeds of these
fictitious loans were issued but made payable to two different entities – Philippine Recycler’s Inc
and Zeta International – without any documents issed by the supposed borrowers Timmy’s Inc
and Asia Textile Mills, Inc assigning the supposed loan proceeds to the two payees. Thereafter,
these two manager’s checks – together with several others totaling P 120, 819, 475. 00 – were
encashed, and then deposited in OCBC Savings Account of Go. Then several automatic transfer
deposits were made from Go’s savings account to his OCBC Current Account which were then
used to fund Go’s previously dishonoured personal checks. PDIC, designated as OCBC receiver,
came to the conclusion that the signatures on Timmy’s Inc and Asia Textile Mills, Inc loan
documents were indeed falsified. Two counts of estafa thru falsification of commercial
documents were filed by PDIC against Go. CA affirmed RTC Order granting Go’s demurrer.

ISSUE/S: W/N there’s sufficient evidence not to warrant demurrer to evidence.

HELD: Yes. Trial court effectively failed and /or refused to weigh the prosecution’s evidence
against Go, which it was dutybound to do as a trier of facts; considering that the case involved
hundreds of millions of pesos of OCBC depositors’ money – not to mention that banking
industry is impressed with public interest, the trial court should have conducted itself with
circumspection and engaged in intelligent reflection in resolving issues. Demand is not an
element of the felony. Furthermore, as the beneficiary of the proceeds, Go is presumed to be the
author of the falsification. It is irrelevant that the proceeds of supposed loans were made payable
to entities other than the alleged borrowers. The bottom line is that they are encashed using bank
funds, and the proceeds were deposited in Go’s bank savings and current accounts and used to
fund personal checks.

136
ERDIE E. AMBROCIO Remedial Law Review 1
2014-0308 Criminal Procedure – Case Digests

71. PEOPLE VS. PEPINO

FACTS: Two men and a woman entered the office of Edward Tan at Kilton Motors Corporation
in Sucat, Paraniaque City, and pretended to be customers. Pepino pulled out a gun. Thinking that
it was a holdup, Edward told Pepino that the money was inside the cashier's box. Pepino and the
other man looted the "'cashier's box, handcuffed Edward, and forced him to go with them.
Kidnappers called Edward's father and demanded a P40 million ransom for his release. Edward's
father told the kidnappers that he did not have that amount. The abductors negotiated with
Jocelyn who eventually agreed to a P700,000.00 ransom. When Edward removed his blindfold,
he found himself inside his own car parked at the UP Diliman Campus. He drove home and
reported his kidnapping to Teresita Ang See, a known anti-crime crusader. Edward was invited
to identify some suspects from a lineup consisting of seven persons: five males and two females.
Edward positively identified Pepino, Gomez, and one Mario Galgo. Jocelyn likewise identified
Pepino. Pepino and Gomez did not testify for their defense. Only Pepino only Gomez were
arraigned while others remained at large.

RTC convicted Pepino and Gomez of kidnapping and serious illegal detention under Article 267
of the Revised Penal Code (as amended) and sentenced them to suffer the death penalty. RTC
held that they were positively identified in police lineup. The CA held that Pepino and Gomez
were deemed to have waived any objection to the illegality of their arrests when they did not
move to quash the information before entering their plea, and when they participated at the trial.
Only Gomez appealed.

ISSUE/S: W/N Edward's identification of her during trial might have been preconditioned by the
"suggestive identification" made during the police lineup

HELD: No. The illegality of the warrantless arrest cannot deprive the State of its right to
prosecute the guilty when all other facts on record point to their culpability. It is much too late in
the day to complain about the warrantless arrest after a valid information had been filed, the
accused had been arraigned, the trial had commenced and had been completed, and a judgment
of conviction had been rendered against her.

Applying the totality-of-circumstances test , there was thus ample opportunity for Edward -
before and after the gun had been pointed at him - to view the faces of the three persons who
entered his office. In addition, Edward stated that Pepino had talked to him "[a]t least once a
day" during the four days that he was detained. Edward also saw Gomez seated at the front seat
of the getaway metallic green Toyota Corolla vehicle. In addition, the abductors removed the
tape from Edward's eyes when they arrived at the apartment, and among those whom he saw
there was Gomez. According to Edward, he was able to take a good look at the occupants of the
car when he was about to be released.

On the part of Jocelyn, she was firm and unyielding in her identification of Pepino as the person
who pointed a gun at her husband while going down the stairs, and who brought him outside the
premises of Kilton Motors. She maintained that she was very near when Pepino was taking away
her husband; and that she could not forget Pepino's face.

137
ERDIE E. AMBROCIO Remedial Law Review 1
2014-0308 Criminal Procedure – Case Digests

Police lineup conducted at the NBI was not suggestive. We note that there were seven people in
the lineup; Edward was not compelled to focus his attention on any specific person or persons.
While it might have been ideal if there had been more women included in the lineup instead of
only two, or if there had been a separate lineup for Pepino and for Gomez, the fact alone that
there were five males and two females in the lineup did not render the procedure irregular. There
was no evidence that the police had supplied or even suggested to Edward that the appellants
were the suspected perpetrators.

Defense witness Reynaldo, however, maintained that Pepino and Gomez were among those
already presented to the media as kidnapping suspects by the DOJ a day before the police lineup
was made. In this sense, the appellants were already the focus of the police and were thus
deemed to be already under custodial investigation when the out-of-court identification was
conducted.

Nonetheless, the defense did not object to the in-court identification for having been tainted by
an irregular out-of-court identification in a police lineup. They focused, instead, on the legality
of the appellants' arrests.

The reduced penalty shall likewise apply to the non-appealing party, Pepino, since it is more
favorable to him.

138

Vous aimerez peut-être aussi